You are on page 1of 199

Ultra Practice Bundle PDF

IBPS PO Prelims - Quantitative Aptitude

Page 1 of 199

Subscribe The Xpress Video Course & Mock Test Package for Bank & Insurance Exams
If there are any suggestions/ errors in our PDFs Feel Free to contact us via this email: admin@exampundit.in
Ultra Practice Bundle PDF
IBPS PO Prelims - Quantitative Aptitude

S.No Title Page No


1 Simplification 03

2 Approximation 13

3 Missing Number 26

4 Wrong Number 38

5 Quadratic Equation 50

6 Table DI 71

7 Bar DI 91

8 Line DI 110

9 Pie DI 132

10 Caselet DI 158

11 Mixed DI 176

Page 2 of 199

Subscribe The Xpress Video Course & Mock Test Package for Bank & Insurance Exams
If there are any suggestions/ errors in our PDFs Feel Free to contact us via this email: admin@exampundit.in
Ultra Practice Bundle PDF
IBPS PO Prelims - Quantitative Aptitude
Simplification

1. (360 – 152) × 6 – 210 = ? × 12 5. 50% of 1200 – 320 = ? + 12% of 1500


a) 100 a) 100
b) 50 b) 120
c) 80 c) 150
d) 20 d) 80
e) 40 e) 140
2. (45 × 8) – 16% of 2000 = ? – 260 6. √484 + ? = of 102
a) 100
a) 453
b) 250
b) 463
c) 400
c) 471
d) 450
d) 477
e) 300
e) 501
2
3. 24% of 500 + 30 = ? × 51
7. 27 x 27 - 9 = ? x 40
a) 35
a) 30
b) 20
b) 0
c) 15
c) -2
d) 25
d) 18
e) 30
e) 1
4. (3/8) × 560 + (2/5) × 350 – (3/4) × 200 = ?
8. 192 - 302 + 212 = ?
a) 100
a) -108
b) 180
b) -68
c) 150
c) -98
d) 240
d) -168
e) 200
e) -88
Page 3 of 199

Subscribe The Xpress Video Course & Mock Test Package for Bank & Insurance Exams
If there are any suggestions/ errors in our PDFs Feel Free to contact us via this email: admin@exampundit.in
Ultra Practice Bundle PDF
IBPS PO Prelims - Quantitative Aptitude
9. 8 x 46 - 30 x 22 = ? b) 2795
a) -314 c) 2840
b) -277 d) 2680
c) -288 e) 2720
d) -294 14. (262 + 122) ÷ 4 + 42 % of 1200 = ?
e) -292 a) 750
10. 80% of 370 + 10% of 510 = ? b) 712
a) 347 c) 709
b) 447 d) 728
c) 267 e) 743
d) 377 15. 12 × 7 × 132 ÷ 13 + 45 × 27 ÷ 9 =?
e) 367 a) 1325
11. 150 % of 300 – 70 % of 180 = 10 × ? b) 1430
a) 26.2 c) 1268
b) 32.4 d) 1320
c) 34.5 e) 1227
d) 36.7 16. 1500 + 2000 ÷ 40 × 20 = ?
e) 28.6 a) 1500
12. 0.06 + 6.66 + 0.606 + 0.6 + 66.006 + 6.666 = ? – b) 2300
2.402 c) 2500
a) 78 d) 2000
b) 82 e) 1800
c) 83 17. (81)1/4 + (32)2 – 150 × 2 + (5)2= ?
d) 80 a) 746
e) 85 b) 620
13. √[ (2613 ‚ 13) × (6622 ‚ 11) + 38 ] = 2812 - ? c) 572
a) 2783 d) 780
Page 4 of 199

Subscribe The Xpress Video Course & Mock Test Package for Bank & Insurance Exams
If there are any suggestions/ errors in our PDFs Feel Free to contact us via this email: admin@exampundit.in
Ultra Practice Bundle PDF
IBPS PO Prelims - Quantitative Aptitude
e) 752 b) 25
18. 7500 ÷ 3 + 270 - ? = 102 c) 29
a) 2670 d) 27
b) 2720 e) 26
c) 3250 23. 67.5% of 600 + 22.5 % of 200 = ?
d) 2865 a) 435
e) 6270 b) 415
19. –(6)3 + (30)2 – (2)5 = ? c) 430
a) 764 d) 450
b) 652 e) 425
c) 678 24. (7/22 of 792) x 12 ÷ 63 = ?
d) 256 a) 40
e) 562 b) 45
20. √5476 ‚ √1369 × √1024 = 4 × 2? c) 42
a) 8 d) 49
b) 2 e) 48
c) 6 25. (√529 + 2)3 = 7528 + ?
d) 4 a) 8056
e) 5 b) 8050
21. 272 × 33 ÷ 51 + 32 = ? c) 8000
a) 208 d) 8097
b) 200 e) 8100
c) 198 26. (444 ÷ 37.5 × 57) ÷ 37 = ?
d) 206 a) 19.26
e) 205 b) 17.65
22. 152 - ?2 + 195 + 212 = 132 c) 18.24
a) 28 d) 20.84
Page 5 of 199

Subscribe The Xpress Video Course & Mock Test Package for Bank & Insurance Exams
If there are any suggestions/ errors in our PDFs Feel Free to contact us via this email: admin@exampundit.in
Ultra Practice Bundle PDF
IBPS PO Prelims - Quantitative Aptitude
e) 17.64 b) 11
27. 292 + 63 = (?)2 - 387 c) 24
a) 32 d) 12
b) 42 e) 34
c) 33 32. 3√5832 + √2916 -30% 0f 600 =? - 180
d) 38 a) 78
e) 34 b) 77
28. √ 676 × √289 – ? = 202 c) 72
a) 230 d) 80
b) 250 e) 60
c) 240 33. of 186 + 820 = 4200 -?
d) 260
a) 1830
e) 270
b) 1640
3
29. 468 × 114 ÷ 6 + 53 = ?
c) 1770
a) 300
d) 1685
b) 306
e) 1845
c) 320
34. (256 + 123 + 519 + 372) ÷ ( 48 + 29 +35 + ? ) = 10
d) 315
a) 12
e) 315
b) 18
30.( √1156+ √676 + 30 )× 80% = ?
c) 16
a) 77
d) 25
b) 80
e) 15
c) 72
35. ( 16 × 11 – 26) ÷ ( ? + 14 × 8 ) = 15/14
d) 76
a) 10
e) 75
b) 28
31. (36 × 6)3 ÷ (6 × 6)3 × (1296)2 = 6?
c) 18
a) 22
Page 6 of 199

Subscribe The Xpress Video Course & Mock Test Package for Bank & Insurance Exams
If there are any suggestions/ errors in our PDFs Feel Free to contact us via this email: admin@exampundit.in
Ultra Practice Bundle PDF
IBPS PO Prelims - Quantitative Aptitude
d) 20 40. 250% of 3650 + 155% of 8600 = ?
e) 14 a) 28069
36. 125% of 420 + 15 % of 450 = ? + 68 % of 720 b) 32467
a) 165.9 c) 47400
b) 102.9 d) 22455
c) 490.5 e) 22501
d) 280.8 41.7168 ÷ 16 × 11 + 256 = ?2
e) 150.0 A. 72
37.15% of 300 + 252= ? × 67 B. 68
a) 35 C. 62
b) 10 D. 78
c) 15 E. None of these
d) 25 42. x + ? = 25% of 232
e) 30
A.49
38. B. 43
a) 10/15 C. 41
b) 23/15 D. 48
c) 15/10 E. 46
d) 24/15 43. 34343 – 3432 – 12644 + 14135 = ? + 18373
e) 20/10 A. 300
39. 4/5 of 18/24 of 35/10 of ? = 1512 B. 240
a) 780 C. 280
b) 850 D.220
c) 750 E.200
d) 800 47. 32423 – 7474 –9134 + 22123 = ? + 13124
e) 720 A. 24714

Page 7 of 199

Subscribe The Xpress Video Course & Mock Test Package for Bank & Insurance Exams
If there are any suggestions/ errors in our PDFs Feel Free to contact us via this email: admin@exampundit.in
Ultra Practice Bundle PDF
IBPS PO Prelims - Quantitative Aptitude
B. 24874 A. 2
C. 24634 B. 8
D.24814 C. 6
E.24572 D.3
48. 67 ×4071 ÷ 69 = ?2 – 16 E. None of these
A. 73 50. 3.6 x 4.5 x 2.8 x 50 = ?
B. 77 A.2318
C. 85 B.2268
D.67 C.2348
E.63 D.2258
49. 41× ? × 15 = 702 + √400 E.2168

Simplification - Answers and Explanations

1. Answer: B) 24% of 500 + 302= ? × 51


(360 – 152) × 6 – 210 = ? × 12 0.24 × 500 + 900 = ? × 51
(360 – 225) × 6 – 210 = ? × 12 ? = (120 + 900) ÷ 51
? = 600 ÷ 12 = 20
? = 50 4. Answer: E)
2. Answer: E) (3/8) × 560 + (2/5) × 350 – (3/4) × 200 = ?
(45 × 8) – 16% of 2000 = ? – 260 (3 × 70) + (2 × 70) – (3 × 50) = ?
360 – 0.16 × 2000 = ? – 260 ? = 210 + 140 – 150
? = 360 – 320 + 260 ? = 200
? = 300 5. Answer: A)
3. Answer: B) 50% of 1200 – 320 = ? + 12% of 1500

Page 8 of 199

Subscribe The Xpress Video Course & Mock Test Package for Bank & Insurance Exams
If there are any suggestions/ errors in our PDFs Feel Free to contact us via this email: admin@exampundit.in
Ultra Practice Bundle PDF
IBPS PO Prelims - Quantitative Aptitude
0.50 × 1200 – 320 = ? + 0.12 × 1500 150 % of 300 – 70 % of 180 = 10 × ?
? = 600 – 320 – 180 450 – 126 = 10 × ?
? = 100 324 = 10 × ?
6. Answer: C) 32.4 = ?
? = 102 x (29/6) - √484 12. Answer: c)
= 17 x 29 - 22 0.06 + 6.66 + 0.606 + 0.6 + 66.006 + 6.666 = ? – 2.402
= 493 - 22 80.598 = ? – 2.402
= 471 80.598 + 2.402 = ?
7. Answer: D) 83 = ?
? = (27 x 27 - 9)/40 13. Answer: a)
= (729 - 9)/40 √[ (2613 ÷ 13) + (6622 ÷ 11) + 38 ]= 2812 - ?
= 720/40 √[ 201 + 602 + 38 ] = 2812 - ?
= 18 √841 = 2812 - ?
8. Answer: C) 29 = 2812 - ?
? = 192 - 302 + 212 2783 = ?
= 361 - 900 + 441 14. Answer: c)
= - 98 (262 + 122) ÷ 4 + 42 % of 1200 = ?
9. Answer: E) (676 + 144) ÷ 4 + 42 % of 1200 = ?
? = 8 x 46 - 30 x 22 820 ÷ 4 + 42 % of 1200 = ?
= 368 - 660 205 + 42 % of 1200 = ?
= -292 205 + 504 = ?
10. Answer: A) 709 = ?
? = (80 x 370/100) + (10 x 510/100) 15. Answer: e)
= (29600/100) + (5100/100) 12 × 7 × 132 ÷ 13 + 45 × 27 ÷ 9 =?
= 296 + 51 12 × 7 × 13 + 45 × 3 = ?
= 347 1092 + 135 = ?
11. Answer: b) 1227 = ?
Page 9 of 199

Subscribe The Xpress Video Course & Mock Test Package for Bank & Insurance Exams
If there are any suggestions/ errors in our PDFs Feel Free to contact us via this email: admin@exampundit.in
Ultra Practice Bundle PDF
IBPS PO Prelims - Quantitative Aptitude
16. Answer: c) ?= 208
1500 + 2000 ÷ 40 × 20 = ? 22. Answer: d)
1500 + 50 × 20 = ? 152 - ?2 + 195 + 212 = 132
1500 + 1000 =? ?2 = 152 + 195 + 212 – 132
2500 = ? ?2 = 225 + 195 + 441 – 132 = 729
17. Answer: e) ? = 27
1/4 2 2
(81) + (32) – 150 × 2 + (5) = ? 23. Answer: d)
3 + 1024 – 300 + 25 = ? 67.5% of 600 + 22.5 % of 200 = ?
752 = ? ? = 405 + 45
18. Answer: a) = 450
7500 ÷ 3 + 270 - ? = 102 24. Answer: e)
2500 + 270 - ? = 100 (7/22 of 792) × 12 ÷ 63 = ?
2670 = ? (7 x 792 x 12) / (22 x 63) = ?
19. Answer: b) ? = 48
–(6)3 + (30)2 – (2)5 = ? 25. Answer: d)
-216 + 900 -32 = ? (√529 + 2)3 = 7528 + ?
652 = ? ? = (23 + 2)3 -7528 = 253 -7528
20. Answer: d) ? = 15625 – 7528
?
√5476 ÷ √1369 × √1024 = 4 × 2 ? = 8097
74 ÷ 37 × 32 = 4 × 2? 26. Answer: c)
2 × 32 = 4 × 2? (444 ÷ 37.5 × 57) ÷37 = ?
16 = 2? ? = (444 x 57) / (37.5 x 37)
24 = 2? ? = 228 / 12.5
4=? = 18.24
21. Answer: a) 27. Answer: d)
? = 272 × 33 ÷ 51 + 32 292 + 63 = (?)2 - 387
= 16 x 11 + 32 = 176 + 32 ?2= 841 + 216 + 387
Page 10 of 199

Subscribe The Xpress Video Course & Mock Test Package for Bank & Insurance Exams
If there are any suggestions/ errors in our PDFs Feel Free to contact us via this email: admin@exampundit.in
Ultra Practice Bundle PDF
IBPS PO Prelims - Quantitative Aptitude
?2 = 1444 33. Answer: a)
? = 38 50/6 of 186 + 820 = 4200 -?
28. Answer: c) 50/6 × 186 + 820 = 4200 – x
√ 676 × √289 – ? = 202 50 × 31 +820 = 4200 –x
? = 26 x 17 – 202 1550 + 820 = 4200 – x
? = 442 – 202 X=1830
? = 240 34. Answer: e)
29. Answer: a) (256 + 123 + 519 + 372) ÷ ( 48 + 29 +35 + ? ) = 10
468 × 114 ÷ 63 + 53 = ? 1270 ÷ (112 + ?) = 10
? = 13 x 19 + 53 = 247 + 53 = 300 1270 ÷ 10 = (112+?)
? = 300 127 = 112 + ?
30. Answer: c) 15 = ?
( √1156+ √676 + 30 )× 80% = ? 35. Answer: b)
(34 + 26 + 30) x 0.80 =? ( 16 × 11 – 26) ÷ ( ? + 14 × 8 ) = 15/14
? = 90 x 0.80 (176 – 26) ÷ ( ? + 112) = 15/14
= 72 150 ÷ ( ? +112 ) = 15/14
31. Answer: b) 150 ÷ 15/14 = ? +112
(36 × 6)3 ÷ (6 × 6)3 × (1296)2 = 6? 150 × 14 /15 = ? +112
3 3 2 3× 4 2 x
(6 ) ÷ (6 ) (6 ) =6 140 = ? +112
69 ÷ 66 × 68 =6x 28 = ?
69-6+8 =6x 36. Answer: b)
611 = 6x 125% of 420 + 15 % of 450 = ? + 68% of 720
X=11 1.25 × 420 + 0.15 × 450 - 0.68 ×720=?
32. Answer: c) 525 + 67.5 - 489.6=?
3
√5832 + √2916 -30% 0f 600 = ? - 180 102.9 = ?
18 + 54 – 180 = x -180 37. Answer: B)
72 = x 15% of 300 + 252= ? × 67
Page 11 of 199

Subscribe The Xpress Video Course & Mock Test Package for Bank & Insurance Exams
If there are any suggestions/ errors in our PDFs Feel Free to contact us via this email: admin@exampundit.in
Ultra Practice Bundle PDF
IBPS PO Prelims - Quantitative Aptitude
0.15 × 300 + 625 = ? × 67 x + ? = 25% of 232
? = (45 + 625) ÷ 67
27/11 x 44/9 + ? = 58
= 10
12 + ? = 58
38. Answer: b)
? = 58 -12
? = 46
43. Anwer: a)
(2-3+8-6) + ( 1/3 -4/5 +3/2 -1/2) =?
34343 – 3432 – 12644 + 14135 - 18373 = ?
1 + (10-24+45-15)/30 =?
? = 14029
1 + 16/30 = ?
44. Answer: c)
46/30 =?
(0.5)4 × 500 ÷ 5000 of (0.5)2= ?
23/15 =?
0.0625 x 500 / 1250 = ?
39. Answer: e)
31.25 / 1250 = ?
4/5 of 18/24 of 35/10 of ? = 1512
?= 0.025
4/5 × 18/24 × 35/10 × x = 1512
45. Answer: b)
21/10 × x = 1512
78% of 500 – 18% of 1950 = 0.5 × ?
X = 15120/21
390 – 351 = 0.5 x ?
X = 720
39 / 0.5 = ?
40. Answer: d)
? = 78
250% of 3650 + 155% of 8600 = ?
46. Answer: e)
9125 + 13330=?
178% of 450 + 72% of 250 = 39.05% of 2000 + ?
22455 =?
801 + 180 = 781 + ?
41. Answer: a)
? = 981 – 781
?2 = 7168 ÷ 16 × 11 +256
= 200
?2 = 448 × 11 + 256
47. Answer: d)
?2 = 4928 + 256 = 5184
? = 32423 – 7474 – 9134 + 22123 – 13124
? = √5184 = 72
? = 24814
42. Answer: e)
Page 12 of 199

Subscribe The Xpress Video Course & Mock Test Package for Bank & Insurance Exams
If there are any suggestions/ errors in our PDFs Feel Free to contact us via this email: admin@exampundit.in
Ultra Practice Bundle PDF
IBPS PO Prelims - Quantitative Aptitude
48. Answer: e) ? x 41 x 15 = 4900 + 20 = 4920
2
67 × 4071 ÷ 69 = ? – 16 ? = 4920 / (41 x 15)
?2 – 16 = 67 x 59 =8
?2 = 3953 + 16 = 3969 50. Answer: b)
? = 63 ? = 3.6 x 4.5 x 2.8 x 50
49. Answer: b) ? = 2268

Simplification Practice Questions PDF Download


Download Quantitative Aptitude Practice Questions PDF

Approximation
Direction (1 to 50): What approximate value should E.249
come in the place of (?) in the following questions? 3) 75.06 % of 399.97 + 45.09% of 179.89 = ?
1) 19.918% of 3600.019 – 45% of 6401.111 = ? – A.392
3660.103 B.376
A.1400 C.381
B.1500 D.360
C.1300 E.354
D.1200 4) (8.12)2 + (6.19)2 + (9.98)2 =?
E.1600 A.50
2) 54.31 * 19.98 - 65.02 * 12.879 =? B.100
A.235 C.150
B.274 D.200
C.228 E.300
D.208 5) √5475 × √2024 / (√1370) = x2 / (89.91)
Page 13 of 199

Subscribe The Xpress Video Course & Mock Test Package for Bank & Insurance Exams
If there are any suggestions/ errors in our PDFs Feel Free to contact us via this email: admin@exampundit.in
Ultra Practice Bundle PDF
IBPS PO Prelims - Quantitative Aptitude
A.18 B.300
B.27 C.250
C.81 D.200
D.45 E.360
E.90 10) (0.064) * (0.4)7 = (0.4)? * (0.0256)2
A.2
6) B.7
A.800 C.4
B.700 D.5
C.600 E.8
D.550 11) 60% of 549.86 + 2.5% of 801.22 = x% of 1051
E.450 A.20.66
7) 45.05% of 540.7 + 33.37% of 599.65 = ?2 + (6.911)3 B.25.44
A.11 C.33.33
B.-11 D.30.22
C.12 E.40.55
D.-12 12) √1938×√480 ‚√124 = 10x-10
E.None of these A.10
8) 2√783 × 3√3374 + (41.89×22.22) = ? + 123.65 B.20
A.920 C.25
B.1120 D.15
C.1320 E.30
D.1420 13) (2798÷55.97) ×12.11 = 30% of ?
E.1220 A.2500
9) 35.31% of 839.26 + 21.92×15.25 =? +299.39 B.2000
A.324 C.1500
D.2700
Page 14 of 199

Subscribe The Xpress Video Course & Mock Test Package for Bank & Insurance Exams
If there are any suggestions/ errors in our PDFs Feel Free to contact us via this email: admin@exampundit.in
Ultra Practice Bundle PDF
IBPS PO Prelims - Quantitative Aptitude
E.3000 D.105
E.84
18) (36.4 × 9-1.9) ÷ (81-1.5×94.3)= 3?
14) ?= (68.06-31.90+ ×8.02+ 24.99÷5.01)÷?
A.0.6
A.-9
B.1.2
B.12
C.1
C.10
D.0
D.-12
E.-2.1
E.None of these
19) 484.217 + 221.316 +? = (123.71 ×20.02)- (16.01)2
15) 196- 33.33% of [(-211-418.07)/√440.9] =?
A.1520
A.196
B.1440
B.180
C.1860
C.186
D.1630
D.200
E.2080
E.220
20) 2√(1220)+ (14.981)2 = (? +119.19)× 4.907
16) 23.99×15.071÷35.89+8499÷501.01 =? ×3.013
A.-103
A.15
B.-67
B.9
C.-84
C.12
D.-46
D.18
E.-32
E.21
21) ? × 15.27 + (29.99)2 – (16.19)2 + 411.77 = (30.99)2 +
17)
(11.88)3
A.111
17.69% of
B.109
A.63
C.123
B.21
D.131
C.42
E.120
Page 15 of 199

Subscribe The Xpress Video Course & Mock Test Package for Bank & Insurance Exams
If there are any suggestions/ errors in our PDFs Feel Free to contact us via this email: admin@exampundit.in
Ultra Practice Bundle PDF
IBPS PO Prelims - Quantitative Aptitude
22) 51.25% of 379.99 + x% of 599.99 = (12.79)2 + 26) √145 % of 350.091 + 12.518 % of 231.889 + √80.83
(5.89)2 + (7.89)3 * √840 = ?
A.82 A.312
B.79 B.350
C.85 C.348
D.71 D.332
E.87 E.369
23) 4/5 × (55.22) + 29.99% of 4569.99 + 364.83 = x3 + 27) 368.93 * √120 + √530 * 71.891 – 207.041 * √255= ?
51.99 * 89.12
A.12 A.27
B.17 B.23
C.24 C.30
D.10 D.36
E.7 E.45
24) 230.89 × 4/11 ‚ 20.95 + √x = 1001.111 ‚ 25 28) 66.667% of (26.98)² + 7³ + (155.90 – 133.87)²
A.1520 +17.9= (?)³
B.4984 A.12
C.1296 B.11
D.1561 C.10
E.4979 D.13
25) 35.91% of ? – 20.11 × 44.89 - √224.80 = (21.11)2 E.14
A.4639 29) 2.98 x 11.09 + 23.92 + 71.98 - 33.89 = 4? – 2
B.3863 A.4
C.3782 B.6
D.4521 C.5
E.3767 D.8
E.9
Page 16 of 199

Subscribe The Xpress Video Course & Mock Test Package for Bank & Insurance Exams
If there are any suggestions/ errors in our PDFs Feel Free to contact us via this email: admin@exampundit.in
Ultra Practice Bundle PDF
IBPS PO Prelims - Quantitative Aptitude
30) (496.20 + 592.65)1/2 ÷ 11.20 = (2.48 x ?) ÷ 4.99 E.22500
A.9 34)
B.12
C.14
A.2450
D.6
B.2140
E.8
C.2340
31) 236.60 + 247.90 - (?)² = - (31.87% of 599)
D.2250
A.26
E.2350
B.24
35)
C.29
D.28
E.27
A.1333
B.1310
C.1210
32)
D.1270
A.245000
E.1370
B.231000
36) 5 8/9 % of 99 + (6.114)3 – (3/8.1) of 239 = ? + 12.92
C.222000
A.120
D.245000
B.140
E.215000
C.100
D.90

33) E.135

A.6400
B.28900 37)
C.19600 A.6603
D.25600 B.5993

Page 17 of 199

Subscribe The Xpress Video Course & Mock Test Package for Bank & Insurance Exams
If there are any suggestions/ errors in our PDFs Feel Free to contact us via this email: admin@exampundit.in
Ultra Practice Bundle PDF
IBPS PO Prelims - Quantitative Aptitude
C.4448 E.55
D.2033 41) √63.62 * 24.93 + 32.01 ‚ √16.08 = ? + 128.07
E.None of these A.50
B.70

38) C.60

A.192 D.80

B.155 E.90

C.142 42) (1560.08 ‚ √170) * 4.98 = ? + 1679.901 ‚ 2.91

D.64 A.30

E.None of these B.40

39) C.45
D.35
E.25
43) (? ‚ 35.95) * √15 = (17.89 ‚ ?) * √65
A.16603
A.43
B.14993
B.41
C.15093
C.39
D.12033
D.36
E.None of these
E.32
40)
44) 47.95 * 49.92 + 13.05 * 17.02 = ?2 + 20
A.56
B.51
C.53
A.57
D.59
B.59
E.49
C.47
45) (6.98³ - 16.89²) + (27.99² - 21.98²) + (1016 ÷ 22) =
D.30
(?)
Page 18 of 199

Subscribe The Xpress Video Course & Mock Test Package for Bank & Insurance Exams
If there are any suggestions/ errors in our PDFs Feel Free to contact us via this email: admin@exampundit.in
Ultra Practice Bundle PDF
IBPS PO Prelims - Quantitative Aptitude
A.395 A.2
B.385 B.3
C.390 C.1.5
D.400 D.2.5
E.405 E.None of these
46) [(66.09² - 13.99²) ‚ 159] + (√8640 ‚ 30.99) + 10.89² 49) 1571.11% of 120.99 – (√63.99)3 – (15.99 x 10.98 x
= (?) 4.98) + 7.99² = (?)²
A.15 A.26
B.11 B.27
C.12 C.24
D.9 D.29
E.14 E.23
1/2
47) (64.99% of 258 + 14.89% of 578) + (1027 ÷ 50) (√2300 + 44.94 x 15.99) + (15.01 x 13.99) ‚ 29.89 =
23.99) + √7050 = (?) (?)
A.150 A.795
B.147 B.895
C.153 C.765
D.157 D.775
E.143 E.789
48) 239.999% of 16.66% of 1240 + (18.91² - 12.89²) =
95.99/? - 40.21

Approximation - Answer and Detailed Explanation

1) Answer: B 20% of 3600 – 45% of 6400 =? – 3660


19.918% of 3600.019 – 45% of 6401.111 = ? – 720 – 2880 = ? -3660
3660.103 1500 =?
Page 19 of 199

Subscribe The Xpress Video Course & Mock Test Package for Bank & Insurance Exams
If there are any suggestions/ errors in our PDFs Feel Free to contact us via this email: admin@exampundit.in
Ultra Practice Bundle PDF
IBPS PO Prelims - Quantitative Aptitude
2) Answer: A
54.31 * 19.98 - 65.02 * 12.879 =?
54*20-65*13=?
1080 – 845 = ?
? = 235
3) Answer: C
75.06 % of 399.97 + 45.09% of 179.89 = ? 7) Answer: E

75% of 400+45%180=? 45.05% of 540.7 + 33.37% of 599.65 = ?2 + (6.911)3

300 + 81 = ? 45% of 540 + 33.33% of 600 = ?2 + 73


243 +200 -343 = ?2
? = 381
+10 or -10 =?
4) Answer: D
(8.12)2 + (6.19)2 + (9.98)2 =? 8) Answer: E
2
=> 82 + 62 + 102 √783 × 3√3374 + (41.89×22.22) = ? + 123.65
2
√783 × 3√3374 + (42×22) = ? + 124
=> 64 + 36 + 100
=> 200 (28 ×15) + 924 = ? + 124
1220 =?
5) Answer: E
9) Answer: A
√5475×√2024/(√1370)=x2/(89.91)
√5476×√2025/√1369= x2/90 35.31% of 839.26 + 21.92×15.25 =? +299.39
35% of 840 + 22×15 = ? +300
74 × 45/37 = x2/90
902 =x2 294 +330 = ? +300
324=?
90 =x
6) Answer: C 10) Answer: A
(0.064) * (0.4)7 = (0.4)? * (0.0256)2
(0.4)3 + 7 = (0.4)? * (0.4)8
? = 10 – 8 = 2
11) Answer: C
60% of 549.86 + 2.5% of 801.22 = x% of 1051
Page 20 of 199

Subscribe The Xpress Video Course & Mock Test Package for Bank & Insurance Exams
If there are any suggestions/ errors in our PDFs Feel Free to contact us via this email: admin@exampundit.in
Ultra Practice Bundle PDF
IBPS PO Prelims - Quantitative Aptitude
60% of 550 + 2.5% of 800 = x% of 1050 196+10= 206
330 + 20 = x% of 1050 206=?
350 = (x/100) (1050) 16) Answer: B
x= 33.33 23.99×15.071÷35.89+8499÷501.01 =? ×3.013
12) Answer: A 24× (15/36) + (8500/500) =?×3
√1938×√480 ‚√124 = 10x-10 10 + 17 =?×3
44 × 22 ÷11 = 10x-10 ?=9
88 +10 =10x 17) Answer: B
10 =x
13) Answer: B
17.69% of
(2798÷55.97) ×12.11 = 30% of ?
18% of 2√(11025) + [(63/19)×(57/9)]÷10 =?
(2800/56) ×12 = 30% of ?
18% of 105 + 21÷10 =?
50 ×12 = 30% of ?
18.9 +2.1 =?
600= (30/100) of ?
21=?
2000 =?
18) Answer: D
14) Answer: A
(36.4 × 9-1.9) ÷ (81-1.5×94.3)= 3?
(36.4 × 3-3.8) ÷ (3-6 × 38.6) = 3?
?= (68.06-31.90+ ×8.02+ 24.99÷5.01)÷? 32.6 ÷ 32.6 =3?
?2= (68 – 32 + (15/3) ×8 + (25/5) ) 32.6-2.6 = 3?
?2= 81 30 = 3?
?= +9 or -9 0 =?
15) Answer: D 19) Answer: A
196- 33.33% of [(-211-418.07)/√440.9] =? 484.217 + 221.316 +? = (123.71 ×20.02)- (16.01)2
196 – 33.33% of [-629/21] =? 484 + 221 +? = (124×20) – 256
196 – 33.3% of (330)=? 705 + ? = 2224
196- (-10) =? ? = 1519 (Approx 1520)

Page 21 of 199

Subscribe The Xpress Video Course & Mock Test Package for Bank & Insurance Exams
If there are any suggestions/ errors in our PDFs Feel Free to contact us via this email: admin@exampundit.in
Ultra Practice Bundle PDF
IBPS PO Prelims - Quantitative Aptitude
20) Answer: B Hence, the required answer is = 87.
2
√(1220)+ (14.981)2 = (? +119.19)× 4.907 23) Answer: A
2
√(1220)+ 225 = (? +119) ×5 4/5 × (55.22) + 29.99% of 4569.99 + 364.83 = x3 + 51.99
35+ 225 = (?+119) ×5 4/5 × 55 + 30% of 4570 + 365 = x3 + 52
52 – 119 = ? 4/5 × 55 + 30/100 × 4570 + 365 = x3 + 52
-67 =? 4/5 × 55 + 1371 + 365 = x3 + 52
21) Answer: B 44 + 1371 + 365 – 52 = x3
? × 15.27 + (29.99)2 – (16.19)2 + 411.77 = (30.99)2 + 1728 = x3
(11.88)3 x = 12
? × 15 + (30)2 – (16)2 + 412 = (31)2 + (12)3 24) Answer: C
? × 15 + 900 –256 + 412 = 961 + 1728 230.89 × 4/11 ÷ 20.95 + √x = 1001.111 ÷ 25
? × 15 + 1312 – 256 = 961 + 1728 231 × 4/11 ÷ 21 + √x = 1001 ÷ 25
? × 15 + 1056 = 2689 231 × 4/11 × 1/21 + √x = 40
? × 15 = 2689 – 1056 4 + √x = 40
? × 15 = 1633 √x = 40 – 4
? = 1633/15 x = 1296
?= 109 25) Answer: E
22) Answer: E 35.91% of ? – 20.11 × 44.89 - √224.80 = (21.11)2
51.25% of 379.99 + x% of 599.99 = (12.79)2 + (5.89)2 + 36% of ? – 20 × 45 - √225 = (21)2
(7.89)3 36% of ? – 900 – 15 = 441
51.25% of 380 + x% of 600 = (13)2 + (6)2 + (8)3 36% of ? = 441 + 900 + 15
51.25/100 × 380 + x/100 × 600 = 169 + 36 + 512 36% of ? = 1356
195 + x/100 × 600 = 717 ? = 1356 × 100/36
195 + 6x = 717 ? = 3767
6x = 717 – 195 26) Answer: D
6x = 522 √145 % of 350.091 + 12.518 % of 231.889 + √80.83 *
x = 87 √840 = ?
Page 22 of 199

Subscribe The Xpress Video Course & Mock Test Package for Bank & Insurance Exams
If there are any suggestions/ errors in our PDFs Feel Free to contact us via this email: admin@exampundit.in
Ultra Practice Bundle PDF
IBPS PO Prelims - Quantitative Aptitude
42 + 29 + 261 = ? 236.60 + 247.90 - (?)² = - (31.87% of 599)
? = 332 237 + 248 - (?)² = - (32% of 600)
27) Answer: A 485 + 192 = (?)²
368.93 * √120 + √530 * 71.891 – 207.041 * √255 = ? * 677 = (?)²
89.12 ? = 26
4059 + 1656 – 3312 = ? * 89 32) Answer: B
? = 27
28) Answer: B
66.667% of (26.98)² + 7³ + (155.90 – 133.87)² +17.9=
(?)³
2/3 x 729 + 343 + (156 – 134)² +18= (?)³
? = 231000
486 + 343 + 484+18 = (?)³
33) Answer: D
(?)³ = 1331
? = 11
29) Answer: C
2.98 x 11.09 + 23.92 + 71.98 - 33.89 = 4? – 2
3 X 11 + 24 + 7² - 34 = 4? – 2
33 + 16 + 49 – 34 = 4? – 2
64 = 4? – 2
4³ = 4? – 2 ? = 25600
?=3+2=5 34) Answer: E
30) Answer: D 39.99% of 399.987 + 24.99% of 1240.001 = 0.2 x ?
(496.20 + 592.65)1/2 ÷ 11.20 = (2.48 x ?) ÷ 4.99
(496 + 593)1/2 ÷ 11 = (2.5 x ?) ÷ 4.99
3 x 5 = 2.5 x ?
160 + 310 = 0.2 x ?
?=6
31) Answer: A
Page 23 of 199

Subscribe The Xpress Video Course & Mock Test Package for Bank & Insurance Exams
If there are any suggestions/ errors in our PDFs Feel Free to contact us via this email: admin@exampundit.in
Ultra Practice Bundle PDF
IBPS PO Prelims - Quantitative Aptitude

?= 2350
35) Answer: B

? = 4 x 1 x 16 + 128 = 64 + 128 = 192 approx.


39) Answer: D

? = 1310
36) Answer: A
5 8/9 % of 99 + (6.114)3 – (3/8.1) of 239 = x + 12.92
(6/100) * 100 + 63 – (3/8) * 240 – 13 = x
24336 +? = 36369
x = 6 + 216 – 90 – 13
? = 12033 approx.
x = 119 ≈ 120
40) Answer: A
37) Answer: C

432 – 375 =?
? = 4448 approx.
? = 57 approx.
38) Answer: A
41) Answer: D
√63.62 * 24.93 + 32.01 ÷ √16.08 = ? + 128.07
200 + 8 = ? + 128
Page 24 of 199

Subscribe The Xpress Video Course & Mock Test Package for Bank & Insurance Exams
If there are any suggestions/ errors in our PDFs Feel Free to contact us via this email: admin@exampundit.in
Ultra Practice Bundle PDF
IBPS PO Prelims - Quantitative Aptitude
? = 80 (?) = 12
42) Answer: B 47) Answer: E
(1560.08 ÷ √170) * 4.98 = ? + 1679.901 ÷ 2.91 (65% of 260 + 15% of 580)1/2 + 43 + 84 =?
600 = ? + 560 √256 + 127 = (?)
? = 40 143 = (?)
43) Answer: D 48) Answer: A
(? ÷ 35.95) * √15 = (17.89 ÷ ?) * √65 240% of 1/6 of 1240 + (19² - 13²) + 40 = 96/?
(?/36) * 4 = (18/?) * 8 496 + 192 + 40 = 96/?
? = 36 96/? = 728
44) Answer: B 96/? = 9³
47.95 * 49.92 + 13.05 * 17.02 = ?2 + 20 6/? = 3
48*50+13*17= ?2 + 20 ?=2
2
2400 + 221 = ? + 20 49) Answer: C
? = 51 1900 - (8)3 – 880 + 64 = (?)²
45) Answer: D 1964 – (512 + 880) = (?)²
(343 – 289) + (784 – 484) + 46 = (?) (?)² = 572
54 + 300 + 46 = (?) ? = 24
(?) = 400 50) Answer: D
46) Answer: C (√2300 + 44.94 x 15.99) + (15.01 x 13.99) ‚ 29.89 = (?)
([(66 + 14) x (66 – 14)] ÷ 160) + (93 ÷ 31) + 121 = (?) (48 + 45 x 16) + (15 x 14) ÷ 30 = (?)
26 + 3 + 121 = (?)² 768 + 7 = (?)
(?)² = 150 (?) = 775

Approximation Practice Questions PDF Download


Download Quantitative Aptitude Practice Questions PDF

Page 25 of 199

Subscribe The Xpress Video Course & Mock Test Package for Bank & Insurance Exams
If there are any suggestions/ errors in our PDFs Feel Free to contact us via this email: admin@exampundit.in
Ultra Practice Bundle PDF
IBPS PO Prelims - Quantitative Aptitude
Missing Number
1. 5, 17, 37, 67, 109, ? a) 0.21
a) 165 b) 0.93
b) 138 c) 0.9
c) 168 d) 0.81
d) 182 e) 0.73
e) 146 6. 124, 194, 257, 313,?,404
2. 6, 24, 144, 1152, 11520, ? a) 484
a) 138240 b) 362
b) 74350 c) 396
c) 143800 d) 246
d) 114400 e) 256
e) 477800 7. 1, 2, 5, 10, 17, ?
3. 19, 57, 97, 139, 183, ? a) 40
a) 246 b) 45
b) 229 c) 26
c) 204 d) 29
d) 288 e) 30
e) 226 8. 160, ?, 480, 2400, 16800, 151200
4. 1105, 961, 840, 740, 659, ? a) 200
a) 691 b) 360
b) 641 c) 200
c) 671 d) 240
d) 619 e) 160
e) 595 9. 178, 159, 142, ?, 114, 103
5. 2268, 324, 54, 10.8, 2.7, ? a) 124
Page 26 of 199

Subscribe The Xpress Video Course & Mock Test Package for Bank & Insurance Exams
If there are any suggestions/ errors in our PDFs Feel Free to contact us via this email: admin@exampundit.in
Ultra Practice Bundle PDF
IBPS PO Prelims - Quantitative Aptitude
b) 127 E.600
c) 132 14. 225, 4096, ?, 5832, 361, 8000
d) 129 A.289
e) 135 B.256
10. 15, 9, 11, 19, ? , 106. C.3450
a) 20 D.1300
b) 41 E.540
c) 24 15. 14 , 21 , 52.5 ,? , 826.875
d) 27.5 A.105.25
e) 36.5 B.183.75
11. 2, 27, 63 , 112 ,176 ,? , 357 C.240.75
a).250 D.564.25
b).257 E.740.05
c).226 16. 8 , 10 , 20 ,? , 96 , 178
d).253 A.64
e).324 B.35
12. 4420, 2160, 1030, ?, 182.5, 41.25 C.46
a).475 D.40
b).360 E.50
c).650 17. 32, 41, 59, 86 , 122, ?
d).570 A.164
e).465 B.246
13. 8, 18 , 57, ?, 1165 , 6996 C.219
A.135 D.167
B.232 E.245
C.180 18. 27600 , 13800 , ? , 1150 , 230
D.545 A.8970
Page 27 of 199

Subscribe The Xpress Video Course & Mock Test Package for Bank & Insurance Exams
If there are any suggestions/ errors in our PDFs Feel Free to contact us via this email: admin@exampundit.in
Ultra Practice Bundle PDF
IBPS PO Prelims - Quantitative Aptitude
B.4600 e) 11
C.12600 23. 10 14 23 39 64 ? 149
D.5060 a) 114
E.2320 b) 119
19. 630, ? , 2030, 3030 , 4230 c) 104
A.1230 d) 100
B.1250 e) 109
C.1000 24. 18 36 90 ? 945 3780
D.1600 a) 360
E.2010 b) 270
20. 520, 259 , 251, ? , 34.5 , -107.75 c) 380
A.148.75 d) 290
B.164.05 e) 245
C.108.50 25. 183 120 71 36 15 ?
D.98.50 a) 13
E.120.25 b) 12
21. 8, 20, 44, ? , 188, 380 c) 7
a) 67 d) 6
b) 80 e) 8
c) 88 26. 12 13 ? 44 60 185
d) 70 a) 18
e) 92 b) 20
22. ? , 33, 95, 375, 1869, 11207 c) 17
a) 25 d) 19
b) 18 e) 16
c) 16 27. 4 3 4 ? 32 155
d) 15 a) 13
Page 28 of 199

Subscribe The Xpress Video Course & Mock Test Package for Bank & Insurance Exams
If there are any suggestions/ errors in our PDFs Feel Free to contact us via this email: admin@exampundit.in
Ultra Practice Bundle PDF
IBPS PO Prelims - Quantitative Aptitude
b) 3 D. 65.05
c) 14 E. 62.75
d) 9 32. 2 , 2 , 8 , 72 , ? , 28800
e) 15 A. 1152
28. 17 18 26 53 117 ? B. 2870
a) 242 C. 5885
b) 217 D. 3864
c) 253 E. 10500
d) 218 33. 2162 , 2167 , 2192 , ? , 2942 , 6067
e) 170 A. 2476
29. 8 ? 64 98 136 178 B. 2548
a) 38 C. 2280
b) 34 D. 2870
c) 39 E. 2317
d) 33 34. 45 , 90 , ? , 2160 , 17280 , 172800
e) 28 A. 180
30. 10000 2000 1000 200 ? B. 360
20 C. 450
a) 200 D. 540
b) 250 E. 90
c) 100 35. 2294 , ? , 1974 , 2935 , 1254 , 3855
d) 150 A. 2300
e) 180 B. 1875
31. 12 , 6 , 9 , 22.5 , ? , 354.375 C. 2415
A. 55.25 D. 1760
B. 72.65 E. 2870
C. 78.75 36. 8 , 9 , ? , 75 , 316 , 1605
Page 29 of 199

Subscribe The Xpress Video Course & Mock Test Package for Bank & Insurance Exams
If there are any suggestions/ errors in our PDFs Feel Free to contact us via this email: admin@exampundit.in
Ultra Practice Bundle PDF
IBPS PO Prelims - Quantitative Aptitude
A. 36 D. 34
B. 45 E. 42
C. 18 41. 936 , 315 , 108 , ? , 16
D. 22 A. 35
E. 68 B. 18
37. 27 , 42 , 60 , ? , 105 , 132 C. 39
A. 66 D. 26
B. 81 E. 36
C. 74 42. 4 , 16 , ? , 2304 , 57600
D. 95 A. 144
E. 102 B. 240
38. 16 , 8 , 12 , ? , 105 , 472.5 C. 160
A. 90 D. 230
B. 40 E. 260
C. 65 43. 6 , 4 , 6 , 12 , 28 , ?
D. 85 A. 40
E. 30 B. 56
39. 10 , 20 , 20 , ? , 40 , 5 C. 80
A. 15 D. 75
B. 10 E. None of these
C. 25 44. 625 , 624 , 632 , 605 , ? , 544
D. 50 A. 634
E. 30 B. 652
40. 5 , 8 , ? , 47 , 95 , 170 C. 695
A. 20 D. 624
B. 30 E. 669
C. 36 45. 2205 , 4410 , 2940 , 1176 , ? , 74.67
Page 30 of 199

Subscribe The Xpress Video Course & Mock Test Package for Bank & Insurance Exams
If there are any suggestions/ errors in our PDFs Feel Free to contact us via this email: admin@exampundit.in
Ultra Practice Bundle PDF
IBPS PO Prelims - Quantitative Aptitude
A. 336 A. 1277
B. 225 B. 1288
C. 441 C. 1273
D. 117 D. 1200
E. 467 E. 1245
46. 96 , 96 , 35 , 7 , ? 49. 11 , 22 , ? , 264 , 1320 , 7920
A. 3 A. 50
B. 1 B. 24
C. 6 C. 60
D. 0 D. 63
E. 11 E. 66
47. 256 , 207 , ? , 146 , 130 , 121 50. 32 , ? , 3.5 , -7.25 . -19.625
A. 177 A. 30
B. 171 B. 35
C. 170 C. 15
D. 160 D. 7
E. 150 E. 19
48. 1256 , 1257 , 1248 , ? , 1224 , 1305

Missing Number - Answer and Detailed Explanation


1. Answer: A) 109 + (7 x 8) 165
5 5 2. Answer: A)
5 + (3 x 4) 17 6 6
17 + (4 x 5) 37 6x4 24
37 + (5 x 6) 67 24x6 144
67 + (6 x 7) 109 144x8 1152
Page 31 of 199

Subscribe The Xpress Video Course & Mock Test Package for Bank & Insurance Exams
If there are any suggestions/ errors in our PDFs Feel Free to contact us via this email: admin@exampundit.in
Ultra Practice Bundle PDF
IBPS PO Prelims - Quantitative Aptitude
1152x10 11520 194 + 63 257 (Since, 70 – 7
11520x12 138240 = 63)
3. Answer: B) 257 + 56 313 (Since, 63 -7
19 19 = 56)
19 + 38 57 313 + 49 362 (Since, 56 -7
57 + 40 97 = 49)
97 + 42 139 362 + 42 404
139 + 44 183 7. Answer: C)
183 + 46 229 1 1
4. Answer: E) 1+1 2
1105 1105 2+3 5
1105 - 122 961 5+5 10
2
961 - 11 840 10 + 7 17
840 - 102 740 17 + 9 26
740 - 92 659 8. Answer: E)
659 - 82 595 160 160
5. Answer: C) 160 × 1 160
2268 2268 160 × 3 480
2268 ÷ 7 324 480 × 5 2400
324 ÷ 6 54 2400 × 7 16800
54 ÷ 5 10.8 16800 × 9 151200
10.8 ÷ 4 2.7 9. Answer: B)
2.7 ÷ 3 0.9 178 178
6. Answer: B) 178 – 19 159
124 124 159 – 17 142
124 + 70 194 142 – 15 127
127 – 13 114
Page 32 of 199

Subscribe The Xpress Video Course & Mock Test Package for Bank & Insurance Exams
If there are any suggestions/ errors in our PDFs Feel Free to contact us via this email: admin@exampundit.in
Ultra Practice Bundle PDF
IBPS PO Prelims - Quantitative Aptitude
114 – 11 103 57 x 4 + 4 232
10. Answer: B) 232 x 5 + 5 1165
15 15 1165 x 6 +6 6996
15 × 0.5 + 1.5 9 14. Answer: A)
9×1+2 11 225 (152) 225
11 x 1.5 + 2.5 19 4096 (163) 4096
2
19 × 2 + 3 41 289 (17 ) 289
41 × 2.5 + 3.5 106 5832 (183) 5832
11. Answer: B) 361 (192) 361
2 2 8000 (203) 8000
2 + 52 27 15. Answer: B)
27 + 62 63 14 14
2
63 +7 112 14 × 1.5 21
112 + 82 176 21 × 2.5 52.5
176 + 92 257 52.5 × 3.5 183.75
257 + 102 357 183.75 × 4.5 826.875
12. Answer: E) 16. Answer: C)
4420 4420 8 8
2
4420 ÷ 2 - 50 2160 8 + 1 +1 10
2160 ÷ 2 – 50 1030 10 + 32 + 1 20
1030 ÷ 2 – 50 465 20 + 52 + 1 46
465 ÷ 2 – 50 182.5 46 + 72 + 1 96
182.5 ÷ 2 – 50 41.25 96 + 92 + 1 178
13. Answer: B) 17. Answer: D)
8 8 32 32
8 x 2 +2 18 32 + (1 ×9 ) 41
18 x3+3 57 41 + (2×9) 59
Page 33 of 199

Subscribe The Xpress Video Course & Mock Test Package for Bank & Insurance Exams
If there are any suggestions/ errors in our PDFs Feel Free to contact us via this email: admin@exampundit.in
Ultra Practice Bundle PDF
IBPS PO Prelims - Quantitative Aptitude
59 + (3×9) 86 92×2+4 188
86 + (4×9) 122 188x2+4 380
122 +(5×9) 167 22. Answer: B)
18. Answer: B) 18 18
27600 27600 18×2-3 33
27600 ÷2 13800 33×3-4 95
13800 ÷3 4600 95×4-5 375
4600 ÷4 1150 375×5-6 1869
1150 ÷5 230 1869×6-7 11207
19. Answer: A) 23. Answer: D)
630 630 10 10
630 + 600 1230 10 + 2² 14
1230 + 800 2030 14 + 3² 23
2030 + 1000 3030 23 + 4² 39
3030 + 1200 4230 39 + 5² 64
20. Answer: D) 64 + 6² 100
520 520 100 + 72 149
520 × 0.5 – 13 259 24. Answer: B)
3
259 × 1 – 2 251 18 18
251 × 0.5 – 33 98.5 18 x 2 36
98. × 1 – 43 34.5 36 x 2.5 90
34.5 × 0.5 – 53 -107.75 90 x 3 270
21. Answer: E) 270 x 3.5 945
8 8 945 x 4 3780
8×2+4 20 25. Answer: E)
20×2+4 44 183 183
44×2+4 92 183 – 7 x 9 120
Page 34 of 199

Subscribe The Xpress Video Course & Mock Test Package for Bank & Insurance Exams
If there are any suggestions/ errors in our PDFs Feel Free to contact us via this email: admin@exampundit.in
Ultra Practice Bundle PDF
IBPS PO Prelims - Quantitative Aptitude
120 – 7 x 7 71 8 + 26 34
71 – 7 x 5 36 34 + 30 64
36 – 7 x 3 15 64 + 34 98
15 – 7 x 1 8 98 + 38 136
26. Answer: C) 136 + 42 178
12 12 30. Answer: C)
12 + 1³ 13 10000 10000
13 + 2² 17 10000/5 2000
17 + 3³ 44 2000/2 1000
44 + 4² 60 1000/5 200
60 + 5³ 185 200/2 100
27. Answer: D) 100/5 20
4 4 31. Answer: C)
4x1–1 3 12 12
3x2–2 4 12 × 0.5 6
4x3–3 9 6 × 1.5 9
9x4–4 32 9 × 2.5 22.5
32 x 5 – 5 155 22.5 × 3.5 78.75
28. Answer: A) 78.75 × 4.5 354.375
17 17 32. Answer: A)
17 + 1 (13) 18 2 2
18 + 8 (23) 26 2 × 12 2
26 + 27 (33) 53 2 × 22 8
3 2
53 + 64 (4 ) 117 8 ×3 72
117 + 125 (53) 242 72 × 42 1152
29. Answer: B) 1152 × 52 28800
8 8 33. Answer: E)
Page 35 of 199

Subscribe The Xpress Video Course & Mock Test Package for Bank & Insurance Exams
If there are any suggestions/ errors in our PDFs Feel Free to contact us via this email: admin@exampundit.in
Ultra Practice Bundle PDF
IBPS PO Prelims - Quantitative Aptitude
2162 2162 37. Answer: B)
1
2162 +5 2167 27 42 60 81 105 132 (Taking
2167 + 52 2192 Difference between Consecutive numbers)
2192 + 53 2317 15 18 21 24 27 (Taking
2317 + 54 2942 Difference Again)
2942 + 55 6067 3 3 3 3
34. Answer: B) 38. Answer: E)
45 45 16 16
45 ×2 90 16 × 0.5 8
90 ×4 360 8 × 1.5 12
360 ×6 2160 12 × 2.5 30
2160 ×8 17280 30 × 3.5 105
17280 × 10 172800 105 × 4.5 472.5
35. Answer: C) 39. Answer: B)
2294 2294 10 10
2294 + 112 2415 10 ÷ 0.5 20
2415 - 212 1974 20 ×1 20
1974 + 312 2935 20 ÷2 10
2
2935 - 41 1254 10 ×4 40
1254 + 512 3855 40 ÷8 5
36. Answer: D) 40. Answer: A)
8 8 5 5
8 × 1 + 12 9 5 + (1×3) 8
2
9 ×2+2 22 8 + (2×6) 20
2
22 ×3+3 75 20 + (3×9) 47
75 × 4 + 42 316 47 + (4×12) 95
316 × 5 + 52 1605 95 + (5×15) 170
Page 36 of 199

Subscribe The Xpress Video Course & Mock Test Package for Bank & Insurance Exams
If there are any suggestions/ errors in our PDFs Feel Free to contact us via this email: admin@exampundit.in
Ultra Practice Bundle PDF
IBPS PO Prelims - Quantitative Aptitude
41. Answer: C) 2205 2205
936 936 2205 ÷ 0.5 4410
936 ÷ 3 + 3 315 4410 ÷ 1.5 2940
315 ÷ 3 + 3 108 2940 ÷ 2.5 1176
108 ÷ 3 + 3 39 1176 ÷ 3.5 336
39 ÷3+3 16 336 ÷ 4.5 74.67
42. Answer: A) 46. Answer: B)
4 4 96 96
4 ×4 16 96 ÷1 96
16 ×9 144 96 ÷3 35
144 × 16 2304 35 ÷5 7
2304 × 25 57600 7 ÷7 1
43. Answer: D) 47. Answer: B)
6 6 256 207 171 146 130 121
6 × 0.5 + 1 4 49 36 25 16 9
4 ×1+2 6 The difference is 72 , 62 , 52 , 42 , 32
6 × 1.5 + 3 12 48. Answer: C)
12 ×2+4 28 1256 1256
2
28 × 2.5 + 5 75 1256 +1 1257
44. Answer: E) 1257 - 32 1248
625 625 1248 + 52 1273
625 - 13 624 1273 - 72 1224
624 + 23 632 1224 + 92 1305
3
632 -3 605 49. Answer: E)
605 + 43 669 11 11
669 - 53 544 11 ×2 22
45. Answer: A) 22 ×3 66
Page 37 of 199

Subscribe The Xpress Video Course & Mock Test Package for Bank & Insurance Exams
If there are any suggestions/ errors in our PDFs Feel Free to contact us via this email: admin@exampundit.in
Ultra Practice Bundle PDF
IBPS PO Prelims - Quantitative Aptitude
66 ×4 264 32 × 0.5 - 12 15
2
264 ×5 1320 15 × 0.5 - 2 3.5
1320 ×6 7920 3.5 × 0.5 - 32 - 7.25
50. Answer: C) -7.25 × 0.5 - 42 - 19.625
32 32

Number Series Practice Questions PDF Download


Download Quantitative Aptitude Practice Questions PDF

Wrong Number
Direction (1 to 50): Find out the wrong number in the D.353
following number series E.339
1) 284, 140, 68, 34, 14, 5 3) 70, 68, 135, 395, 1575, 7869
A.284 A.1575
B.140 B.68
C.34 C.135
D.14 D.70
E.5 E.5395
2) 339, 341, 345, 353, 369, 400 4) 96, 420, 131, 385, 162, 358
A.400 A.385
B.341 B.162
C.345 C.420

Page 38 of 199

Subscribe The Xpress Video Course & Mock Test Package for Bank & Insurance Exams
If there are any suggestions/ errors in our PDFs Feel Free to contact us via this email: admin@exampundit.in
Ultra Practice Bundle PDF
IBPS PO Prelims - Quantitative Aptitude
D.358 A.8057
E.131 B.7993
5) 852, 880, 945, 1073, 1288, 1632 C.8226
A.852 D.8202
B.880 E.8105
C.945 10) 13, 90, 546, 2730, 10920, 32760
D.1288 A.546
E.1073 B.90
6) 19, 235, 260, 324, 335, 341 C.2730
A.341 D.32760
B.235 E.10920
C.235 11) 13, 27, 56, 102, 167, 254
D.335 A.167
E.335 B.102
7) 4000, 4400, 4820, 5324, 5856.4, 6442.04 C.56
A.4400 D.27
B.4820 E.254
C.5324 12) 11, 18, 48, 180, 910, 5448
D.5856.4 A.18
E.6442.04 B.180
8) 384, 315, 248, 183, 120, 100 C.910
A. 120 D.5448
B.183 E.48
C. 100 13) 155, 153, 148, 138, 121, 97, 58
D.248 A.121
E.315 B.153
9) 7993, 8202, 8057, 8226, 8105 C.97
Page 39 of 199

Subscribe The Xpress Video Course & Mock Test Package for Bank & Insurance Exams
If there are any suggestions/ errors in our PDFs Feel Free to contact us via this email: admin@exampundit.in
Ultra Practice Bundle PDF
IBPS PO Prelims - Quantitative Aptitude
D.58 A.80
E.155 B.98
14) 118, 122, 149, 165, 291, 326 C.51
A.118 D.27
B.291 E.34
C.325 19) 53760 6710 960 160 32 8
D.122 A.32
E.165 B.960
15) 750, 550, 450, 400, 375, 365 C.6710
A.550 D.160
B.450 E.53760
C.400 20) 162, 163, 170, 182, 202, 232
D.375 A.162
E.365 B.170
16) 280 285 292 302 318 329 C.182
A.329 D.163
B.280 E.202
C.302 21) 12166, 9259, 6856, 4909, 3370, 2190
D.285 A.9259
E.318 B.4909
17) 8 8 15 41 105 228 C.3370
A.15 D.2190
B.41 E.6856
C.228 22) 102, 84, 64, 44, 26, 13
D.105 A.13
E.None of these B.84
18) 27 34 51 62 80 98 C.64
Page 40 of 199

Subscribe The Xpress Video Course & Mock Test Package for Bank & Insurance Exams
If there are any suggestions/ errors in our PDFs Feel Free to contact us via this email: admin@exampundit.in
Ultra Practice Bundle PDF
IBPS PO Prelims - Quantitative Aptitude
D.44 A.463
E.26 B.382
23) 20, 58, 120, 212, 342, 510 C.138
A.212 D.794
B.342 E.922
C.120 28) 705, 709, 727, 775, 875, 1044, 1349
D.510 A.709
E.58 B.727
24) 22, 32, 49, 78, 112, 162 C.875
A.49 D.1349
B.162 E.1044
C.32 29) 470, 555, 648, 768, 952, 1261, 1886
D.112 A.1886
E.78 B.1261
25) 54, 135, 300, 675, 1350, 3375 C.952
A.300 D.470
B.675 E.768
C.1350 30) 6, 12, 58, 516, 6254, 93318
D.3375 A.6
E.135 B.58
26) 100, 115, 150, 190, 250, 325 C.6254
A.100 D.516
B.115 E.93318
C.150 31) 14 20 45 151 629 3181 19135
D.190 A.3181
E.250 B.45
27) 138, 190, 382, 463, 794, 922 C.629
Page 41 of 199

Subscribe The Xpress Video Course & Mock Test Package for Bank & Insurance Exams
If there are any suggestions/ errors in our PDFs Feel Free to contact us via this email: admin@exampundit.in
Ultra Practice Bundle PDF
IBPS PO Prelims - Quantitative Aptitude
D.151 A.24
E.20 B.22
32) 8.5, 28, 65.5, 127, 218, 346, 515.5 C.116
A.65.5 D.250
B.128 E.None of these
C.28 37)
D.346 10000 2000 800 640 1024 3276.6
E.218 A.2000
33) 9, 10, 18, 83, 594, 4690 B.640
A.9 C.3276.6
B.10 D.800
C.4690 E.1024
D.594 38) 48 168 504 1260 2520 3760
E.83 A.48
34) 30, 39, 64, 113, 201, 315 B.3760
A.30 C.504
B.39 D.1260
C.64 E.2520
D.113 39) 704 352 528 1310 4620 20790
E.201 A.704
35) 117, 460, 676, 812, 865, 892 B.352
A.812 C.528
B.892 D.1310
C.460 E.4620
D.865 40) 32, 44, 74, 116, 172, 242
E.676 A.32
36) 22 24 30 60 116 250 B.44
Page 42 of 199

Subscribe The Xpress Video Course & Mock Test Package for Bank & Insurance Exams
If there are any suggestions/ errors in our PDFs Feel Free to contact us via this email: admin@exampundit.in
Ultra Practice Bundle PDF
IBPS PO Prelims - Quantitative Aptitude
C.74 45) 5 10 83 244 819 1634
D.172 A.819
E.242 B.83
41) 14, 40, 92, 196, 376, 664 C.1634
A.664 D.10
B.196 E.244
C.376 46) 12.5 26 82 337 1710 10231
D.92 A.10231
E.40 B.82
42) 39, 41, 49, 98, 314, 939 C.337
A.41 D.1710
B.314 E.12.5
C.98 47) 22 27 38 62 101 177 282
D.39 A.101
E.49 B.282
43) 8, 10, 22, 52, 109, 198 C.38
A.10 D.27
B.52 E.62
C.109 48) 31 38 47.1 58.5 72.4 87.8
D.22 A.38
E.198 B.72.4
44) 50 53 63 85.5 126 181 283 C.58.5
A.181 D.31
B.53 E.87.8
C.126 49) 35, 65, 113, 181, 271, 390
D.50 A.65
E.85.5 B.390
Page 43 of 199

Subscribe The Xpress Video Course & Mock Test Package for Bank & Insurance Exams
If there are any suggestions/ errors in our PDFs Feel Free to contact us via this email: admin@exampundit.in
Ultra Practice Bundle PDF
IBPS PO Prelims - Quantitative Aptitude
C.271 B.1238
D.113 C.1150
E.181 D.1182
50) 1400, 1310, 1238, 1182, 1150, 1110 E.1310
A.1110

Wrong Number - Answer and Detailed Explanation


1) Answer: C 3) Answer: C
284/2=142-2=140 70 x 1 - 2 =68
140/2=70-2=68 68 x 2 – 3 = 133
68/2=34-2=32 133 x 3 – 4 = 395
32/2=16-2=14 395 x 4 – 5 = 1575
14/2=7-2=5 1575 x 5 -6 = 7869
2) Answer: A 4) Answer: A
339 + 21 = 341 96 + 182 = 420
341 + 22 = 345 420 – 172 = 131
345 + 23 = 353 131 + 162 = 387
353 + 24 = 369 387 – 152 = 162
369 + 25 = 401 162 + 142 = 358

5) Answer: E
The correct series is,
852, 880, 945, 1071, 1288, 1632
28 65 126 217 344
33 + 1 43 + 1 53 + 1 63 + 1 73 + 1
The difference is, 33 + 1, 43 + 1, 53 + 1, 63 + 1, 73 + 1,....
The wrong term is, 1073

Page 44 of 199

Subscribe The Xpress Video Course & Mock Test Package for Bank & Insurance Exams
If there are any suggestions/ errors in our PDFs Feel Free to contact us via this email: admin@exampundit.in
Ultra Practice Bundle PDF
IBPS PO Prelims - Quantitative Aptitude

6) Answer: D 13 * 7 = 91
19 + 63 = 235 91 * 6 = 546
235 + 52 = 260 546 * 5 = 2730
260 + 43 = 324 2730 * 4 = 10920
324 + 32 = 333 10920 * 3 = 32760
3
333 + 2 = 341 11) Answer: E
The wrong number is 335. 13 + 22 + 10 = 27
7) Answer: B 27 + 32 + 20 = 56
4000 + 10% of 4000 = 4400 56 + 42 + 30 = 102
4400 + 10% of 4400 = 4840 (not 4820) 102 + 52 + 40 = 167
4840 + 10% of 4840 = 5324 167 + 62 + 50 = 253
5324 + 10% of 5324 = 5856.4 The wrong term is 254
5856.4 + 10% of 5856.4 = 6442.04 12) Answer: B
8) Answer: C (11 – 2) * 2 = 18
64 * 6 = 384 (18 – 2) * 3 = 48
63 * 5 = 315 (48 – 2) * 4 = 184
62 * 4 = 248 (184 – 2) * 5 = 910
61 * 3 = 183 (910 – 2) * 6 = 5448
60 * 2 = 120 The wrong term is 180
59 * 1 = 59 13) Answer: C
9) Answer: D 155-(2) = 153
7993 + 172 = 8282 (not 8202) 153-(2+3) = 148
2
8282 – 15 = 8057 148-(2+3+5) = 138
2
8057 + 13 = 8226 138-(2+3+5+7) =121
8226 –112 = 8105 121-(2+3+5+7+9) =95
10) Answer: B 95-(2+3+5+7+9+11) =58
Page 45 of 199

Subscribe The Xpress Video Course & Mock Test Package for Bank & Insurance Exams
If there are any suggestions/ errors in our PDFs Feel Free to contact us via this email: admin@exampundit.in
Ultra Practice Bundle PDF
IBPS PO Prelims - Quantitative Aptitude
14) Answer: B 280 285 292 302 314 329
2
118 + 2 = 118 + 4=122 5 7 10 12 15
122 + 33 = 122 + 27=149 2 3 2 3
149 + 42 = 149 + 16=165 17) Answer: D
165 + 53 = 165 + 125=290 8 + 1³ - 1 = 8
290 + 62 = 290 + 36=326 8 + 2³ - 1 = 15
15) Answer: E 15 + 3³ - 1 = 41
750-200=550 41 + 4³ - 1 = 104
550-100=450 104+ 5³ - 1 = 228
450-50=400 18) Answer: A
400-25=375 27 34 51 62 83 98
375-12.5=362.5 52+2 62-2 72+2 82-2 92+2 102-2
16) Answer: E
19) Answer: C
Given series is,
53760 6720 960 160 32 8
÷8 ÷7 ÷6 ÷5 ÷4
20) Answer: D 173 – 4 = 4909
162 + (2*1) =164 153- 5 = 3370
164 + (3*2) = 170 133 – 6 =2191
170 + (4*3) = 182 22) Answer: A
182 + (5*4) = 202 102 – (6 * 3) = 84
202 + (6*5) = 232 84 – (5 * 4) = 64
21) Answer: D 64 – (4 * 5) = 44
3
23 – 1 = 12166 44 – (3 * 6) = 26
213 – 2 = 9259 26 – (2 * 7) = 12
193 – 3 = 6856 23) Answer: B
Page 46 of 199

Subscribe The Xpress Video Course & Mock Test Package for Bank & Insurance Exams
If there are any suggestions/ errors in our PDFs Feel Free to contact us via this email: admin@exampundit.in
Ultra Practice Bundle PDF
IBPS PO Prelims - Quantitative Aptitude
33 – 7 = 20 115 + 15 * 2 = 145 (not 150)
3
4 – 6 = 58 145 + 15 * 3 = 190
53 – 5 = 120 190 + 15 * 4 = 250
63 – 4 = 212 250 + 15 * 5 = 325
73 – 3 = 340 27) Answer: A
83 – 2 = 510
24) Answer: E
22 + 32 + 1 = 32
32 + 42 + 1 = 49
49 + 52 + 1 = 75
75 + 62 + 1 = 112
112 + 72 + 1 = 162
25) Answer: A
54 * 2.5 = 135
28) Answer: E
135 * 2 = 270 (not 300)
705 + (1 x 22) = 709
270 * 2.5 = 675
709 + (2 x 32) = 727
675 * 2 = 1350
727 + (3 x 42) = 775
1350 * 2.5 = 3375
775 + (4 x 52) = 875
26) Answer: C
875 + (5 x 62) = 1055 (not 1044)
100 + 15 * 1 = 115
1055 + (6 x 72) = 1349
29) Answer: A

Page 47 of 199

Subscribe The Xpress Video Course & Mock Test Package for Bank & Insurance Exams
If there are any suggestions/ errors in our PDFs Feel Free to contact us via this email: admin@exampundit.in
Ultra Practice Bundle PDF
IBPS PO Prelims - Quantitative Aptitude
30) Answer: E 594 + 84 = 4690
11 x 2 + 22= 6 34) Answer: E
22 x 2 + 22 = 12 30 + 32 = 39
33 x 2 + 22 = 58 39 + 52 = 64
44 x 2 + 22 = 516 64 + 72 = 113
55 x 2 + 22 = 6254 113 + 92 = 194 (not 201)
66 x 2 + 22 = 93316 (Not 93318) 194 + 112 = 315
31) Answer: E 35) Answer: A
14 × 1 + (1 + 1)2 = 18 (Not 20) 117 + 73 = 460
18 × 2+ (1 + 2)2 = 45 460 + 63 = 676
45 × 3+ (1 + 3)2 = 151 676 + 53 = 801
151 × 4+ (1 + 4)2 = 629 801 + 43 = 865
629 × 5 + (1 + 5)2 = 3181 865 + 33 = 892
3181 × 6 + (1 + 6)2 = 19135 36) Answer: C
32) Answer: E 22 + (1³ + 1) = 24
2 x 22 + 0.5 = 8.5 24 + (2³ - 2) = 30
3 x 32 + 1 = 28 30 + (3³ + 3) = 60
4 x 42 + 1.5 = 65.5 60 + (4³ - 4) = 120
2
5 x 5 + 2 = 127 120 + (5³ + 5) = 250
6 x 62 + 2.5 = 218.5 (not 218) 37) Answer: C
7 x 72 + 3 = 346 10000 x 20% = 2000
8 x 82 + 3.5 = 515.5 2000 x 40% = 800
33) Answer: E 800 x 80% = 640
0
9 + 8 = 10 640 x 160% = 1024
1
10 + 8 = 18 1024 x 320% = 3276.8
18 + 82 = 82(not 83) 38) Answer: B
82 + 83 = 594 48 x 3.5 = 168
Page 48 of 199

Subscribe The Xpress Video Course & Mock Test Package for Bank & Insurance Exams
If there are any suggestions/ errors in our PDFs Feel Free to contact us via this email: admin@exampundit.in
Ultra Practice Bundle PDF
IBPS PO Prelims - Quantitative Aptitude
168 x 3 = 504 8 10 22 52 109 203
504 x 2.5 = 1260 2 12 30 57 94
1260 x 2 =2520 10 18 27 37
2520 x 1.5 = 3780 8 9 10
39) Answer: D 44) Answer: A
704 x 1/2 = 352 50 53 63 85.5 126 190 283
352 x 3/2 = 528 3 10 22.5 40.5 64 93
528 x 5/2 = 1320 7 12.5 18 23.5 29
1320 x 7/2 = 4620 5.5 5.5 5.5 5.5
4620 x 9/2 = 20790 Hence, the required answer is = 181.
40) Answer: B 45) Answer: A
32+ (14*1)= 32 + 14 =46 5 + 13 + 22 = 10
46+ (14*2)= 46 + 28=74 10 + 32+ 43= 83
74+ (14*3)= 74 + 42=116 83 + 53 + 62 = 244
116+ (14*4)= 116 + 56=172 244 + 72+ 83 = 805
172+ (14*5)= 172 + 70=242 805 + 93 + 102 = 1634
41) Answer: C Hence, the required answer is = 819.
14 + (13*2) = 40 46) Answer: D
40+ (13*4) = 92 12.5 × 2 + 12 = 26
92+ (13*8) = 196 26 × 3 + 22 = 82
196 + (13*14) = 378 82 × 4 + 32 = 337
378 + (13*22) = 664 337 × 5 + 42 = 1701
42) Answer: D 1701 × 6 + 52 = 10231
Given series is Hence, the required answer is = 1710.
41 41 49 98 314 939 47) Answer: A
90 81 72 63 54 22 27 38 62 106 177 282
43) Answer: E 5 11 24 44 71 105
Page 49 of 199

Subscribe The Xpress Video Course & Mock Test Package for Bank & Insurance Exams
If there are any suggestions/ errors in our PDFs Feel Free to contact us via this email: admin@exampundit.in
Ultra Practice Bundle PDF
IBPS PO Prelims - Quantitative Aptitude
6 13 20 27 34 65 + 32 * 1.5 = 113
7 7 7 7 113 + 34 * 2 = 181
Hence, the required answer is = 101. 181 + 36 * 2.5 = 271
48) Answer: E 271 + 38 * 3 = 385
31 38 47.1 58.5 72.4 89 50) Answer: C
7 9.1 11.4 13.9 16.6 1400 - (92 + 9) = 1310
2.1 2.3 2.5 2.7 1310 – (82 + 8) = 1238
Hence, the required answer is = 87.8. 1238 – (72 + 7) = 1182
49) Answer: B 1182 – (62 + 6) = 1140
35 + 30 * 1 = 65 1140 – (52 + 5) = 1110

Number Series Practice Questions PDF Download


Download Quantitative Aptitude Practice Questions PDF

Quadratic Equation
1. I. x2 -22x + 120 = 0 II) 2y4 – 5y2 + 3 = 0
II. 2y2 -3y - 65 = 0 a) x> y
a) x< y b) x ≥ y
b) x> y c) x< y
c) x ≤ y d) x = y or Relationship between x and y cannot be
d) x ≥ y determined
e) x = y or relationship cannot be established e) x ≤ y

2. I) 2x2 – 5x + 3 = 0 3. I. 2x2 – 9x + 4 = 0

Page 50 of 199

Subscribe The Xpress Video Course & Mock Test Package for Bank & Insurance Exams
If there are any suggestions/ errors in our PDFs Feel Free to contact us via this email: admin@exampundit.in
Ultra Practice Bundle PDF
IBPS PO Prelims - Quantitative Aptitude
II. 2y2 + 29y – 31 = 0 c) x = y or the relationship cannot be established
a) x< y d) x ≥ y
b) x> y e) x ≤ y
c) x = y OR the relationship cannot be determined
d) x ≥ y 7. I. 2x2 – 20x + 48 = 0
e) x ≤ y II. 2y2 + y – 21 = 0
a) x> y
4. I. x2 – 17x + 60 = 0 b) x< y
II. 5y2 – 23y + 24 = 0 c) x = y or the relationship cannot be established
a) x> y d) x ≥ y
b) x< y e) x ≤ y
c) x = y or the relationship cannot be established
d) x ≥ y 8. I. x2 + 13x – 198 = 0
e) x ≤ y II. y2 – 11y – 102 = 0
a) x< y
5. I. x2 +x - 56 = 0 b) x> y
II. y2 – 15y - 56 = 0 c) x = y OR the relationship cannot be determined
a) x> y d) x ≥ y
b) x< y e) x ≤ y
c) x = y or the relationship cannot be established 9. I. x2 + 7x – 8 = 0
d) x ≥ y II. 3y2 – 21y + 36 = 0
e) x ≤ y a) x< y
b) x> y
2
6. I. 5x - 16x + 11 = 0 c) x = y OR the relationship cannot be determined
II. y2- 15y + 56 = 0 d) x ≥ y
a) x> y e) x ≤ y
b) x< y
Page 51 of 199

Subscribe The Xpress Video Course & Mock Test Package for Bank & Insurance Exams
If there are any suggestions/ errors in our PDFs Feel Free to contact us via this email: admin@exampundit.in
Ultra Practice Bundle PDF
IBPS PO Prelims - Quantitative Aptitude
10. I. 2x2 – 19x + 42 = 0 b) x> y
II. y2 - 14y + 45 = 0 c) x ≤ y
a) x< y d) x ≥ y
b) x> y e) x = y or relationship cannot be established
c) x = y OR the relationship cannot be determined
d) x ≥ y 14. I. 2x2 - 17x + 36 =0
e) x ≤ y II. 6y2 + 19y + 15 = 0
a) x< y
11. I. 2x2 + 19x + 45 = 0 b) x> y
II. 3y2 - 13y + 12 = 0 c) x ≤ y
a) x< y d) x ≥ y
b) x> y e) x = y or relationship cannot be established
c) x ≤ y
d) x ≥ y 15. I.12x2– 40x + 32 = 0
e) x = y or relationship cannot be established] II. 15y2 + 68y + 77 = 0
a) x< y
12. I. 12x2 – 7x = -1 b) x> y
II.2y2+ 22y + 56 = 0 c) x ≤ y
a) x< y d) x ≥ y
b) x> y e) x = y or relationship cannot be established
c) x ≤ y
d) x ≥ y 16.I. y31/7 × y4/7 × 5 =125 ×
e) x = y or relationship cannot be established II. 17x2 + 22x+9=13x2 +10x
a) x< y
13. I. 20x2 + 9x + 1 =0 b) x> y
II. 6y2 - 7y+ 2 = 0 c) x ≤ y
a) x< y d) x ≥ y
Page 52 of 199

Subscribe The Xpress Video Course & Mock Test Package for Bank & Insurance Exams
If there are any suggestions/ errors in our PDFs Feel Free to contact us via this email: admin@exampundit.in
Ultra Practice Bundle PDF
IBPS PO Prelims - Quantitative Aptitude
e) x = y or relationship cannot be established II. y2 - 27y + 162 = 0
a) x< y
17.I.x² = 4624 b) x> y
II.y = √4096 c) x ≤ y
a) x< y d) x ≥ y
b) x> y e) x = y or relationship cannot be established
c) x ≤ y
d) x ≥ y 21. I. 2x2 - x - 1 = 0
e) x = y or relationship cannot be established II. y2 - 7y + 10 = 0
a) x< y
18. I.x² - 28x + 192 = 0 b) x> y
II.y² – 29y + 210 = 0 c) x ≤ y
a) x< y d) x ≥ y
b) x> y e) x = y or relationship cannot be established
c) x ≤ y
d) x ≥ y 22. I. 8x2 – 40x + 48 = 0
e) x = y or relationship cannot be established II. 4y2 + 3y - 22 = 0
a) x< y
19. I. 5 x + 7 y = -43 b) x> y
II. 9 x - 17 y = 41 c) x ≤ y
a) x< y d) x ≥ y
b) x> y e) x = y or relationship cannot be established
c) x ≤ y 23. I. p = √7225
d) x ≥ y II. q2 = 7225
e) x = y or relationship cannot be established a) p< q
b) p> q
20. I. x2 - 29x + 208 = 0 c) p ≤ q
Page 53 of 199

Subscribe The Xpress Video Course & Mock Test Package for Bank & Insurance Exams
If there are any suggestions/ errors in our PDFs Feel Free to contact us via this email: admin@exampundit.in
Ultra Practice Bundle PDF
IBPS PO Prelims - Quantitative Aptitude
d) p ≥ q 27. I. x = 4√2401
e) p = q or relationship cannot be established II. √3025 y + √121 = 0
a) x< y
24. I. x2 + 16x + 63 =0 b) x> y
II. y2+ y - 30 = 0 c) x ≤ y
a) x< y d) x ≥ y
b) x> y e) x = y or relationship cannot be established
c) x ≤ y
d) x ≥ y 28. I.x² = (43 + 62) ÷ 22
e) x = y or relationship cannot be established II.y = √[(25)1/2 × (625)1/4]
a) x< y
25. I. x2 – 2025 = 0 b) x> y
II. y - √225 = 30 c) x ≤ y
a) x< y d) x ≥ y
b) x> y e) x = y or relationship cannot be established
c) x ≤ y
d) x ≥ y 29. I. 5x - 2y = 28
e) x = y or relationship cannot be established II. x + 2y = -16
a) x< y
26.I. x2 – 2x = 0 b) x> y
II. 10y2 + y – 24 = 0 c) x ≤ y
a) x< y d) x ≥ y
b) x> y e) x = y or relationship cannot be established
c) x ≤ y
d) x ≥ y 30. I. 2x2- 15x+ 7 = 0
e) x = y or relationship cannot be established II. 6y2- 47y + 80 = 0
a) x< y
Page 54 of 199

Subscribe The Xpress Video Course & Mock Test Package for Bank & Insurance Exams
If there are any suggestions/ errors in our PDFs Feel Free to contact us via this email: admin@exampundit.in
Ultra Practice Bundle PDF
IBPS PO Prelims - Quantitative Aptitude
b) x> y e) x = y or relationship cannot be established
c) x ≤ y
d) x ≥ y 34. I. 2x2 + x - 3 =0
e) x = y or relationship cannot be established II. 6y2 + 13y - 5 = 0
a) x< y
31. I. 2x2 + 11x + 15 = 0 b) x> y
2
II. 5y + 16y + 11 = 0 c) x ≤ y
a) x< y d) x ≥ y
b) x> y e) x = y or relationship cannot be established
c) x ≤ y
d) x ≥ y 35. I. 625x2 – 1 = 0
e) x = y or relationship cannot be established II. y = 1 ‚ √625
a) x< y
32. I. 8x2 – 31x + 21 = 0 b) x> y
II. 2y2 + 9y + 10 = 0 c) x ≤ y
a) x< y d) x ≥ y
b) x> y e) x = y or relationship cannot be established
c) x ≤ y
d) x ≥ y 36.I. 21x2 + 122x + 165 = 0
e) x = y or relationship cannot be established II. 5y2 + 87y + 378 = 0
a) x< y
33. I. x = 3√5832 b) x> y
II. y2 = 324 c) x ≤ y
a) x< y d) x ≥ y
b) x> y e) x = y or relationship cannot be established
c) x ≤ y 37. I. p = 3√5832
d) x ≥ y II. √576q - 432 = 0
Page 55 of 199

Subscribe The Xpress Video Course & Mock Test Package for Bank & Insurance Exams
If there are any suggestions/ errors in our PDFs Feel Free to contact us via this email: admin@exampundit.in
Ultra Practice Bundle PDF
IBPS PO Prelims - Quantitative Aptitude
a) p< q d) x ≥ y
b) p> q e) x = y or relationship cannot be established
c) p ≤ q
d) p ≥ q 41. I. 7x2 – 9x = - 2
e) p = q or relationship cannot be established II. - 3 = y2 – 4y
a) x< y
38. I.x = √196 b) x> y
II.y2 – 19y + 70 = 0 c) x ≤ y
a) x< y d) x ≥ y
b) x> y e) x = y or relationship cannot be established
c) x ≤ y
d) x ≥ y 42.I. x2= - x + 20
e) x = y or relationship cannot be established II. 2y2 – 19y + 45 = 0
a) x< y
39. I. 2x - 5y = 6 b) x> y
II. 5x - 11y = 3 c) x ≤ y
a) x< y d) x ≥ y
b) x> y e) x = y or relationship cannot be established
c) x ≤ y
d) x ≥ y 43.I. 7x + 3y - 26 = 0
e) x = y or relationship cannot be established II. 2x + 17y + 41 = 0
a) x< y
40. I. 28x2 + 53x + 24 = 0 b) x> y
2
II. 3y - 49y + 200 = 0 c) x ≤ y
a) x< y d) x ≥ y
b) x> y e) x = y or relationship cannot be established
c) x ≤ y
Page 56 of 199

Subscribe The Xpress Video Course & Mock Test Package for Bank & Insurance Exams
If there are any suggestions/ errors in our PDFs Feel Free to contact us via this email: admin@exampundit.in
Ultra Practice Bundle PDF
IBPS PO Prelims - Quantitative Aptitude
44.I. 3x2 – 20x = - 33 b) x> y
II. 2y2 – 11y = - 15 c) x ≤ y
a) x< y d) x ≥ y
b) x> y e) x = y or relationship cannot be established
c) x ≤ y
d) x ≥ y 48.I. x2= 5x + 14
e) x = y or relationship cannot be established II. y2+ 7y + 10 = 0
a) x< y
45.I. 5x2 = - 2x + 3 b) x> y
II. 2y2+ 7y + 6 = 0 c) x ≤ y
a) x< y d) x ≥ y
b) x> y e) x = y or relationship cannot be established
c) x ≤ y
d) x ≥ y 49.I. x2– 64 = 0
e) x = y or relationship cannot be established II. 2y2 + 25y + 72 = 0
a) x< y
46.I. – 53x + 20 = - 35x2 b) x> y
II. 56y2-97y + 42 = 0 c) x ≤ y
a) x< y d) x ≥ y
b) x> y e) x = y or relationship cannot be established
c) x ≤ y
d) x ≥ y 50.I. x2 = 3481
e) x = y or relationship cannot be established II. 3y2 = 3√216000 y
a) x< y
47.I. x = 3√4913 b) x> y
II. 13y + 3x = 246 c) x ≤ y
a) x< y d) x ≥ y
Page 57 of 199

Subscribe The Xpress Video Course & Mock Test Package for Bank & Insurance Exams
If there are any suggestions/ errors in our PDFs Feel Free to contact us via this email: admin@exampundit.in
Ultra Practice Bundle PDF
IBPS PO Prelims - Quantitative Aptitude
e) x = y or relationship cannot be established

Quadratic Equation - Answer and Detailed Explanation

1. Answer: B) II) 2y4 – 5y2 + 3 = 0


I.x2 -22x + 120 = 0 Let y2 = t
x2 - 10x –12x + 120 = 0 y = √t
x(x – 10) –12(x - 10) = 0 2t2 – 5t + 3 = 0
(x - 10)(x -12)= 0 2t2 – 3t -2t + 3 = 0
x = 10, 12 t(2t – 3) -(2t – 3) = 0
(t – 1) (2t – 3) = 0
2
II.2y -3y - 65 = 0 t = 1, 2/3
2y2 -13y +10y + 65 = 0 y = √1,√2/3
y(2y -13) +5(2y - 13) = 0 Hence, x > y.
(2y -13)(y + 5) = 0 Hence, option A is correct.
y = 13/2, -5
Hence, x > y. 3. Answer: C)
Hence, option B is correct. I. 2x2 – 9x + 4 = 0
2x2 – 8x – x + 4 = 0
2. Answer: A) 2x(x – 4) – 1(x – 4) = 0
I) 2x2 – 5x + 3 = 0 (x – 4)(2x – 1) = 0
2x2 – 3x -2x + 3 = 0 x = 1/2,4
x(2x – 3) -(2x-3) = 0
(x – 1) (2x – 3) = 0 II. 2y2 + 29y – 31 = 0
x = 1, 2/3 2y2 + 31y – 2y – 31 = 0
y(2y + 31) – (2y + 31) = 0
Page 58 of 199

Subscribe The Xpress Video Course & Mock Test Package for Bank & Insurance Exams
If there are any suggestions/ errors in our PDFs Feel Free to contact us via this email: admin@exampundit.in
Ultra Practice Bundle PDF
IBPS PO Prelims - Quantitative Aptitude
(2y + 31)(y – 1) = 0
y = - 31/2 or y = + 1 II.y2 – 15y + 56 = 0
While comparing the values of x and y, one root value of y2 – 7y – 8y + 56 = 0
y lies between the root values of x y(y – 7) – 8(y – 7) = 0
Hence, option C is correct. (y – 8)(y – 7) = 0
y = 8, 7
4. Answer: A) Hence, x ≤ y.
I. x2 – 17x + 60 = 0 Hence, option E is correct.
x2 – 12x - 5x + 60 = 0
x(x – 12) - 5 (x – 12) = 0 6. Answer: B)
(x – 12) (x - 5) = 0 I. 5x2 - 16x + 11 = 0
x = 5, 12 5x2 - 11x - 5x + 11 = 0
x(5x - 11) - (5x - 11) = 0
II. 5y2 – 23y + 24 = 0 (x - 1)(5x - 11) = 0
5y2 – 15y – 8y + 24 = 0 x = 1, 11/5
5y(y – 3) – 8(y – 3) = 0
(y – 3)(5y – 8) = 0 II.y2 - 15y + 56 = 0
y = 3, 8/5 y2 - 7y - 8y + 56 = 0
Hence, x > y. y(y - 7) - 8(y - 7) = 0
Hence, option A is correct. (y - 7)(y - 8) = 0
y = 7, 8
5. Answer: E) Hence, x < y.
I. x2 + x - 56 = 0 Hence, option B is correct
2
x - 7x + 8x – 56 = 0
x(x - 7) + 8 (x - 7) = 0 7. Answer: A)
(x - 7) (x + 8) = 0 I. 2x2 – 20x + 48 = 0
x = 7, - 8 2x2 – 8x – 12x + 48 = 0

Page 59 of 199

Subscribe The Xpress Video Course & Mock Test Package for Bank & Insurance Exams
If there are any suggestions/ errors in our PDFs Feel Free to contact us via this email: admin@exampundit.in
Ultra Practice Bundle PDF
IBPS PO Prelims - Quantitative Aptitude
2x(x – 4) – 12(x – 4) = 0 9. Answer: A)
(x – 4)(2x – 12) = 0 I. x2 + 7x – 8 = 0
x = 4, 6 x2 + 8x – x – 8 = 0
x(x + 8) – 1(x + 8) = 0
II.2y2 + y – 21 = 0 (x + 8)(x – 1) = 0
2y2 + 7y – 6y – 21 = 0 x = - 8 or x = + 1
y(2y + 7) – 3(2y + 7) = 0
(2y + 7)(y – 3) = 0 II. 3y2 – 21y + 36 = 0
y = -7/2, 3 3y2 – 12y – 9y + 36 = 0
Hence, x > y. 3y(y – 4) – 9(y – 4) = 0
Hence, option A is correct (y – 4)(3y – 9) = 0
y = + 4 or y = + 9/3
8. Answer: C) So, we can observe that x < y.
I. x2 + 13x – 198 = 0
x2 + 22x – 9x – 198 = 0 10. Answer: C)
x(x + 22) – 9(x + 22) = 0 I. 2x2 – 19x + 42 = 0
(x + 22)(x – 9) = 0 2x2 – 12x – 7x + 42 = 0
x = -22 or x = +9 2x(x – 6) – 7(x – 6) = 0
(x – 6)(2x – 7) = 0
II. y2 – 11y – 102 = 0 x = + 6 or x = + 7/2
y2 – 17y + 6y – 102 = 0
y(y – 17) + 6(y – 17) = 0 II. y2 - 14y + 45 = 0
(y – 17)(y + 6) = 0 y2 - 9y - 5y + 45 = 0
y = 17 or y = -6 y(y - 9) - 5(y - 9) = 0
So, we can observe that no clear relationship cannot be (y - 9)(y - 5) = 0
determined between x and y. y = 9 or y = 5

Page 60 of 199

Subscribe The Xpress Video Course & Mock Test Package for Bank & Insurance Exams
If there are any suggestions/ errors in our PDFs Feel Free to contact us via this email: admin@exampundit.in
Ultra Practice Bundle PDF
IBPS PO Prelims - Quantitative Aptitude
While comparing the values of x and y, one root value of II.2y2 + 22y + 56 = 0
y lies between the root values of x Y2+11y+28=0
Hence, option C is correct. Y2 + 4y +7y + 28 = 0
y (y +4) +7 (y + 4) = 0
11. Answer: A (y +4) (y + 7) = 0
I. 2x2 + 19x + 45 = 0 y = - 4, y= - 7
2
2x + 9x + 10x + 45 = 0 x>y
x (2x + 9) + 5 (2x + 9) = 0 Hence, option B is correct
(2x + 9) (x + 5) = 0
x = -9/2 = -4.5, x = -5 13. Answer: A
I.20x2 + 9x + 1 = 0
II. 3y2 - 13y + 12 = 0 20x2 + 5x + 4x + 1 =0
3y2– 9y - 4y + 12 = 0 5x(4x + 1) + 1(4x + 1) = 0
3y (y - 3) -4 (y - 3) = 0 (4x + 1) (5x + 1) = 0
(3y - 4) (y - 3) = 0 x = -1/4, -1/5
y = 4/3 = 1.3, y= 3
x<y II. 6y2 - 7y + 2 =0
Hence, option A is correct 6y2 - 3y - 4y + 2 = 0
3y(2y - 1) - 2(2y - 1) = 0
12. Answer: B (2y - 1) (3y - 2) = 0
I. 12x2 – 7x = -1 y = 1/2, 2/3
12x2-7x+1=0 x<y
12x2 - 4x - 3x + 1 = 0 Hence, option A is correct.
4x (3x -1) -1 (3x - 1) = 0
(4x - 1) (3x - 1) = 0 14. Answer: B
x = 1/3 = 0.33, x = 1/4 = 0.25 I. 2x2 - 17x + 36 =0
2x2 - 8x - 9x + 36 =0

Page 61 of 199

Subscribe The Xpress Video Course & Mock Test Package for Bank & Insurance Exams
If there are any suggestions/ errors in our PDFs Feel Free to contact us via this email: admin@exampundit.in
Ultra Practice Bundle PDF
IBPS PO Prelims - Quantitative Aptitude
2x(x-4)-9(x-4) = 0 16.Answer: E
(2x-9) (x-4) = 0 I. y31/7 × y4/7 × 5 =125 ×
x = 9/2, 4 Y31/7 + 4/7 -3 = 125/5
Y2 =25
II. 6y2 + 19y + 15 = 0 y = 5, -5
6y2 + 9y +10y + 15 = 0
3y(2y+3)+5(2y+3) = 0 II.17x2 + 22x + 9 =13x2 + 10x
(3y+5)(2y+3) = 0 4x2 + 12x + 9 = 0
y = -5/3, -3/2 4x2 + 6x + 6x + 9=0
x>y 2x(2x + 3) + 3(2x + 3) =0
Hence, option B is correct. (2x+3) + (2x+3) = 0
x = -3/2,-3/2 = -1.5,-1.5
15. Answer: B While comparing the values of x and y, one root value of
I.12x2– 40x + 32 = 0 x lies between the root values of y
12x2– 24x -16x +32 = 0 Hence, option E is correct
12x(x - 2) – 16 (x - 2) = 0
(12x - 16) (x - 2) = 0 17. Answer: E
x = 16/12,2 =1.33,2 I. x2 = 4624
X = +68, -68
II.15y2 + 68y + 77 = 0 II.y = √4096
15y2+ 35y + 33y + 77 = 0 Y= 64
5y(3y + 7) + 11(3y + 7) = 0 While comparing the values of x and y, one root value of
(5y + 11) (3y + 7) = 0 y lies between the root values of x
Y = -11/5,-7/3 =-2.2,-2.33 Hence, option E is correct
x>y
Hence, option B is correct. 18. Answer: E
I.x² - 28x + 192 = 0

Page 62 of 199

Subscribe The Xpress Video Course & Mock Test Package for Bank & Insurance Exams
If there are any suggestions/ errors in our PDFs Feel Free to contact us via this email: admin@exampundit.in
Ultra Practice Bundle PDF
IBPS PO Prelims - Quantitative Aptitude
x² - 16x -12x + 192 = 0
x(x-16)-12(x-16) = 0 20. Answer: E
(x-12)(x-16) = 0 I. x2 - 29x + 208 = 0
x = 12, 16 X2 – 16x – 13x + 208 = 0
x(x -16)-13(x - 16) = 0
II.y² – 29y + 210 = 0 (x-13)(x-16) = 0
y² – 14y-15y + 210 = 0 x = 13, 16
y(y-14)-15(y-14)=0
(y-14)(y-15) = 0 II. y2 - 27y + 162 = 0
y = 14, 15 Y2 - 18y – 9y + 162 = 0
While comparing the values of x and y, root value of y y(y-18)-9(y-18) = 0
lies between the root values of x (y – 18) (y - 9) = 0
Hence, option E is correct y = 18, 9
While comparing the values of x and y, root value of x
19. Answer: B lies between the root values of y.
5 x + 7 y = -43….(I) Hence, option E is correct
9 x - 17 y = 41….(II)
21. Answer: A
Eqn (I) × 9 = 45 x+ 63y= -387 I. 2x2 - x - 1 = 0
Eqn (II) × 5 = 45 x -85 y =205 2x2 + x - 2x - 1 = 0
x (2x + 1) -1 (2x + 1) = 0
Eqn (I) – Eqn (II), (2x + 1 ) (x - 1 ) = 0
148y=-592 x = -1/2 , 1 = -0.5 , 1
Y= -4
y = –4 and x = -3 II. y2 - 7y + 10 = 0
x>y y2– 5y - 2y + 10 = 0
Hence, option B is correct. y (y - 5) – 2 (y - 5) = 0

Page 63 of 199

Subscribe The Xpress Video Course & Mock Test Package for Bank & Insurance Exams
If there are any suggestions/ errors in our PDFs Feel Free to contact us via this email: admin@exampundit.in
Ultra Practice Bundle PDF
IBPS PO Prelims - Quantitative Aptitude
(y - 2 ) (y - 5) = 0 Hence, option D is correct.
y = 2 , y= 5
x< y 24.Answer: A
Hence, option A is correct I. x2 + 16x + 63 =0
x2 + 7x + 9x + 63 =0
22. Answer: D x (x + 7) + 9 ( x + 7) = 0
2
I. 8x – 40x +48 = 0 (x + 9) ( x + 7) = 0
8x2 - 16x -24x + 48=0 x = -9 , -7
8x ( x – 2) – 24 ( x – 2) = 0
(8x – 24) ( x - 2 ) = 0 II. y2 + y - 30 = 0
x = 24/8, 2 = 3 , 2 y2 + 6y - 5y - 30 = 0
y (y + 6) -5 (y + 6) = 0
2
II. 4y + 3y - 22 = 0 (y - 5) (y + 6) = 0
4Y2 - 8y + 11y - 22 = 0 y = 5 , -6
4y (y - 2) + 11 (y - 2) = 0 x<y
(4y + 11) (y - 2) = 0 Hence, option A is correct.
y = -11/4 , 2 = -2.75 , 2
x≥y 25. Answer: C
Hence, option D is correct I. x2 – 2025 = 0
x2 = 2025
23. Answer: D x = + 45 , -45
I. p = √7225
p = 85 II. y - √225 = 30
y = √ 225 + 30
2
II. q = 7225 y = 15 + 30
q = +85 , -85 y = 45
p≥ q x≤y

Page 64 of 199

Subscribe The Xpress Video Course & Mock Test Package for Bank & Insurance Exams
If there are any suggestions/ errors in our PDFs Feel Free to contact us via this email: admin@exampundit.in
Ultra Practice Bundle PDF
IBPS PO Prelims - Quantitative Aptitude
Hence, option C is correct. 28. Answer: C
I.x² = (43 + 62) ÷ 22
26. Answer: E x²= (64 + 36 ) ÷ 4
I. x2 – 2x = 0 x²= 100 ÷ 4
x ( x – 2) = 0 x² = 25
x = 0, 2 x = +5 , -5

II. 10y2 + y – 24 = 0 II. y = √[(25)1/2 × (625)1/4]


10y2 + 16y -15y -24 = 0 y = √[(52)1/2 × (54)1/4 ]
2y (5y + 8) – 3 (5y + 8) = 0 y=√5×5
(2y – 3) (5y + 8) = 0 y=5
y = 3/2 , -8/5 = 1.5 , -1.6 x≤y
While comparing the values of x and y, one root value of Hence, option C is correct
x lies between the root values of y
Hence, option E is correct 29. Answer: B
5x - 2y = 28….(I)
27. Answer: B x + 2y = -16….(II)
I. x = 4√2401
x=7 Eqn (I) + Eqn (II),
6x = 12
II. √3025 y + √121 = 0 x=2
55y + 11 = 0
55y = -11 Substitute x = 2 in Eqn (I),
y = -11/55 = -1/5 = -0.2 5 (2) - 2y = 28
x>y 10 - 2y = 28
Hence, option B is correct -2y = 18
y = -9

Page 65 of 199

Subscribe The Xpress Video Course & Mock Test Package for Bank & Insurance Exams
If there are any suggestions/ errors in our PDFs Feel Free to contact us via this email: admin@exampundit.in
Ultra Practice Bundle PDF
IBPS PO Prelims - Quantitative Aptitude
x>y 5y2+ 5y + 11y + 11 = 0
Hence, option B is correct. 5y (y + 1) + 11 (y + 1) = 0
(5y + 11 ) (y + 1) = 0
30. Answer: E y = -11/5, -1 = -2.2 , -1
I. 2x2 - 15x + 7 = 0 x< y
2x2 - 14x – 1x + 7 = 0 Hence, option A is correct
2x (x - 7) – 1 (x - 7) = 0
(2x - 1) (x - 7) = 0 32. Answer: B
x = 1/2 , 7 = 0.5 , 7 I. 8x2 – 31x + 21 = 0
8x2 - 24x - 7x + 21=0
II. 6y2 - 47y + 80 = 0 8x ( x – 3) – 7 ( x – 3) = 0
6y2 - 15y - 32y + 80 = 0 (8x – 7) ( x - 3 ) = 0
3y (2y - 5) - 16 (2y - 5) = 0 x = 7/8, 3 = 0.8 , 3
(3y – 16) (2y - 5) = 0
y = 16/3 , 5/2 = 5.3 , 2.5 II. 2y2 + 9y + 10 = 0
While comparing the values of x and y, root value of y 2y2 + 4y + 5y + 10 = 0
lies between the root values of x. 2y (y + 2) + 5 (y+ 2) = 0
Hence, option E is correct (2y + 5) (y + 2) = 0
y = -5/2 , -2 = -2.5 , -2
31. Answer: A x>y
I. 2x2 + 11x + 15 = 0 Hence, option B is correct
2x2 + 6x + 5x + 15 = 0
2x (x + 3) + 5 (x + 3) = 0 33. Answer: D
(2x + 5 ) (x + 3 ) = 0 I. x = 3√5832
x = -5/2 ,-3 = -2.5 , -3 x = 18

II. 5y2 + 16y + 11 = 0 II. y2 = 324

Page 66 of 199

Subscribe The Xpress Video Course & Mock Test Package for Bank & Insurance Exams
If there are any suggestions/ errors in our PDFs Feel Free to contact us via this email: admin@exampundit.in
Ultra Practice Bundle PDF
IBPS PO Prelims - Quantitative Aptitude
y = +18 , -18 II. y = 1 ÷ √625
x≥y y = 1 ÷ 25
Hence, option D is correct. x≤y
Hence, option C is correct.
34. Answer: E
I. 2x2 + x - 3 =0 36. Answer: B
2
2x - 2x + 3x - 3 =0 I. 21x2 + 122x + 165 = 0
2x (x - 1) + 3 ( x - 1) = 0 21x2 + 45x + 77x + 165 = 0
(2x + 3) ( x - 1) = 0 3x (7x + 15) + 11 (7x + 15) = 0
x = -3/2 , 1 = -1.5 , 1 (3x + 11) (7x + 15) = 0
x = -11/3 , -15/7 = -3.6 , - 2.1
II. 6y2 + 13y - 5 = 0
6y2 - 2y + 15y - 5 = 0 II. 5y2 + 87y + 378 = 0
2y (3y - 1) + 5 (3y - 1) = 0 5y2 + 45y + 42y + 378 = 0
(2y + 5) (3y - 1) = 0 5y (y + 9) + 42 (y + 9) = 0
y = -5/2 , 1/3 = -2.5 , 0.33 (5y + 42) (y + 9) = 0
While comparing the values of x and y, one root value of y = -42/5 , -9 = -8.4 , -9
x lies between the root values of y x>y
Hence, option E is correct Hence, option B is correct

35. Answer: C 37. Answer: E


I. 625x2 – 1 = 0 I. p = 3√5832
625x2 = 1 p = 18
2
x = 1 ÷ 625 II. √576q - 432 = 0
x = √1÷625 24q - 432 = 0
x = +1/25 , -1/25 24q = 432
q = 18

Page 67 of 199

Subscribe The Xpress Video Course & Mock Test Package for Bank & Insurance Exams
If there are any suggestions/ errors in our PDFs Feel Free to contact us via this email: admin@exampundit.in
Ultra Practice Bundle PDF
IBPS PO Prelims - Quantitative Aptitude
p=q 2x – 5(-8) = 6
Hence, option E is correct 2x + 40 = 6
2x = - 34
38. Answer: D x = -17
I.x = √196 x<y
x = 14 Hence, option A is correct.

II.y2 – 19y + 70 = 0 40. Answer: A


y2 – 5y – 14y + 70 = 0 I. 28x2 + 53x + 24 = 0
y (y – 5 ) – 14 (y – 5) = 0 28x2 + 21x + 32x + 24 = 0
(y – 14) (y – 5) = 0 7x (4x + 3) + 8 (4x + 3) = 0
y = 5, 14 (7x + 8) (4x + 3) = 0
x≥y x = -8/7 , -3/4 = -1.1 , -0.7
Hence, option D is correct
II. 3y2 - 49y + 200 = 0
39. Answer: A 3y2 - 24y - 25y + 200 = 0
2x - 5y = 6….(I) 3y (y - 8) - 25 (y - 8) = 0
5x - 11y = 3….(II) (3y - 25) (y - 8) = 0
y = 25/3 , 8 = 8.3 , 8
Eqn (I) × 5 , 10x – 25y = 30 x<y
Eqn (II) × 2 , 10x – 22y = 6 Hence, option A is correct

Eqn (I) - Eqn (II), 41. Answer: C


-3y = 24 I. 7x2 – 9x = - 2
y = -8 7x2– 7x – 2x + 2 = 0
7x(x – 1) – 2(x – 1) = 0
Substitute y = -8 in Eqn (I), (7x – 2) (x – 1) = 0

Page 68 of 199

Subscribe The Xpress Video Course & Mock Test Package for Bank & Insurance Exams
If there are any suggestions/ errors in our PDFs Feel Free to contact us via this email: admin@exampundit.in
Ultra Practice Bundle PDF
IBPS PO Prelims - Quantitative Aptitude
x =2/7, 1 7x + 3y = 26 …. Eqn (I)

II. - 3 = y2 – 4y II. 2x + 17y + 41 = 0


Y2 – 4y + 3 = 0 2x + 17y = - 41….. Eqn (II)
Y2– y – 3y + 3 = 0
y(y – 1) – 3(y – 1) = 0 Eqn (I) × 2 => 14x + 6y = 52…Eqn (III)
(y – 3) (y – 1) = 0
y = 1, 3 Eqn (II) × 7 =>14x + 119y = – 287 …Eqn (IV)
so, x ≤ y
Hence, option C is correct. Eqn (III) + Eqn (IV)
– 113y = 339
42. Answer: A y = – 3 and x = 5
2
I. x = - x + 20 x>y
X2+ 5x – 4x – 20 = 0 Hence, option B is correct.
x(x + 5) – 4(x + 5) = 0
(x – 4) (x + 5) = 0 44. Answer: D
x = 4, – 5 I. 3x2 – 20x = - 33
3x2– 9x – 11x + 33 = 0
II. 2y2– 10y – 9y + 45 = 0 3x(x – 3) – 11(x – 3) = 0
2y(y – 5) – 9(y – 5) = 0 (3x – 11) (x – 3) = 0
(y – 5) (2y – 9) = 0 x = 3, 11/3
y = 5 , 9/2
so, x < y II. 2y2 – 11y = - 15
Hence, option A is correct. 2y2– 6y – 5y + 15 = 0
2y(y – 3) – 5(y – 3) = 0
43. Answer: B (y – 3) (2y – 5) = 0
I. 7x + 3y - 26 = 0 y = 3, 5/2

Page 69 of 199

Subscribe The Xpress Video Course & Mock Test Package for Bank & Insurance Exams
If there are any suggestions/ errors in our PDFs Feel Free to contact us via this email: admin@exampundit.in
Ultra Practice Bundle PDF
IBPS PO Prelims - Quantitative Aptitude
So, x ≥ y 8y(7y – 6) – 7(7y – 6) = 0
Hence, option D is correct. (8y – 7) (7y – 6) = 0
y = 7/8 , 6/7
45. Answer: B So, x<y
I.5x2= - 2x + 3 Hence, option A is correct.
5x2+ 5x – 3x – 3 = 0
5x (x + 1) – 3(x + 1) = 0 47. Answer: B
(5x – 3) (x + 1) = 0 I. x = 3√4913
x= 3/5, -1 x = 17

II. 2y2+ 7y + 6 = 0 II. 13y = 246 – 3x


2y2+ 4x + 3y + 6 = 0 13y = 246 – 51 = 195
2y(y + 2) + 3(y + 2) =0 y = 15
(2y + 3) (y + 2) = 0 So, x > y
y = -3/2 , -2 Hence, option B is correct.
So, x>y
Hence, option B is correct. 48. Answer: D
I. x2 = 5x + 14
46. Answer: A X2– 7x + 2x – 14 = 0
I. – 53x + 20 = - 35x2 x(x – 7) + 2(x – 7) = 0
35x2 – 28x – 25x + 20 = 0 (x + 2) (x – 7) = 0
7x(5x – 4) – 5(5x – 4) = 0 x = -2, 7
(7x – 5) (5x – 4) = 0
x = 5/7 , 4/5 II. y2+ 7y + 10 = 0
Y2+ 5y + 2y + 10 = 0
II. 56y2 -97y + 42 = 0 y(y + 5) + 2(y + 5) = 0
56y2 – 48y – 49y + 42 = 0 (y + 2) (y + 5) = 0

Page 70 of 199

Subscribe The Xpress Video Course & Mock Test Package for Bank & Insurance Exams
If there are any suggestions/ errors in our PDFs Feel Free to contact us via this email: admin@exampundit.in
Ultra Practice Bundle PDF
IBPS PO Prelims - Quantitative Aptitude
y = -2, -5 So, x = y or relationship cannot be established
So, x ≥ y Hence, option E is correct.
Hence, option D is correct.
50. Answer: E
49. Answer: E I. x2= 3481
I. x2 – 64 = 0 X2= 3481
X2= 64 x = ± 59
x = ±8
II. 3y2 = 3√216000 y
II. 2y2 + 25y + 72 = 0 3y2 = 3 √216000 y
2y2+ 9y + 16y + 72 = 0 3y2= 60y
y(2y + 9) + 8(2y + 9) = 0 y = 20
(y + 8) (2y + 9) = 0 So, x = y or relationship cannot be established
y = -8 , -9/2 Hence, option E is correct.

Quadratic Equation Practice Questions PDF Download


Download Quantitative Aptitude Practice Questions PDF

Table DI
Direction (1-5) Study the table and answer the question.
Number of students recruited by six different companies from college ABC over the years.

TCS CTS IBM L&T CGI


2013 260 184 123 124 212
2014 320 246 126 320 124

Page 71 of 199

Subscribe The Xpress Video Course & Mock Test Package for Bank & Insurance Exams
If there are any suggestions/ errors in our PDFs Feel Free to contact us via this email: admin@exampundit.in
Ultra Practice Bundle PDF
IBPS PO Prelims - Quantitative Aptitude
2015 328 342 178 210 230
2016 412 234 214 124 242
2017 252 120 246 68 124

1. What is the total number of student recruited by d.129


all the companies together in the 2017? e.123
a.452 4. What is ratio of total number of student
b.569 recruited by TCS and CTS together in the year
c.852 2015 to the total number of student recruited by
d.812 CGI in the year 2015 and 2016 together?
e.810 a.452:369
2. What is the percent increase in the number of b.335:236
student recruited by L&T in 2014 from the c.236:598
previous year? d.452:369
a.148% e.321:569
b.152% 5. Number of student recruited by TCS in 2014 is
c.151% approximately what percent of the total number of
d.158% student recruited by all the companies together in
e.196% that year?
3. What is the average (approx) number of student a.28.16%
recruited by IBM over all the year together? b.21.35%
a.173 c.29.36%
b.177 d.27.36%
c.125 e.22.36%
Direction (6-10) Study the table and answer the following question.
Percentage of marks obtained by different students in different subject in the exam.
Note: Number written is bracket is the full marks of that subject.

Page 72 of 199

Subscribe The Xpress Video Course & Mock Test Package for Bank & Insurance Exams
If there are any suggestions/ errors in our PDFs Feel Free to contact us via this email: admin@exampundit.in
Ultra Practice Bundle PDF
IBPS PO Prelims - Quantitative Aptitude

Students EVS(50) Math (100) Science (75) English(120)


P 64 84 56 90
Q 84 68 84 55
R 92 78 68 75
S 72 68 64 70
T 76 84 72 80

6. What are the total marks obtained by R in all the c.23:20


subjects together? d.21:19
a.263 e. none of these
b.262 9. What is the difference between the total marks
c.234 obtained by Q in all the subjects and total marks
d.269 obtained by T in all the subjects?
e.265 a.23
7. What is overall percentage of P in the exam? b.33
a.70% c .31
b.67% d.34
c.71% e.43
d.77% 10. Who score highest marks in math and science
e.69% together?
8. What is the respective ratio of total marks a.T
obtained by T in math and science together to the b.T&P
total marks obtained by S in EVS and English c.R
together? d.S
a.21:29 e.Q
b.23:29

Page 73 of 199

Subscribe The Xpress Video Course & Mock Test Package for Bank & Insurance Exams
If there are any suggestions/ errors in our PDFs Feel Free to contact us via this email: admin@exampundit.in
Ultra Practice Bundle PDF
IBPS PO Prelims - Quantitative Aptitude
Direction (11-15) Study the table and answer the question.
Number of total car (petrol + diesel) of five different showrooms and percentage of diesel car of respective
showrooms are given.

Showrooms Total car Percentage of diesel car


A 150 33.33
B 190 40
C 210 28.57
D 160 56.25
E 240 37.5

11. What is the average number (approx) of the 13. Total number of petrol car in C is what percent
petrol car in all the showrooms together? more of diesel car in D?
a.123 a.65.36%
b.165 b.56.98%
c.152 c.66.66%
d.142 d.25.63%
e.117 e.96.36%
12. What is the difference in the total number 14. What is the respective ratio of petrol car of B
petrol car in A and C together and the diesel car in and diesel car of E?
D and E together? a.29:19
a.96 b.19:15
b.45 c.12:58
c.25 d.17:19
d.70 e.23:21
e.85

Page 74 of 199

Subscribe The Xpress Video Course & Mock Test Package for Bank & Insurance Exams
If there are any suggestions/ errors in our PDFs Feel Free to contact us via this email: admin@exampundit.in
Ultra Practice Bundle PDF
IBPS PO Prelims - Quantitative Aptitude
15. Total petrol car in showrooms A, C, F is 370 b.154
and total diesel car in showrooms B, D, F is 220. c.126
What is total number of car in showrooms F? d.159
a.174 e.123

Direction (16-20) Study the table and answer the question.


Number of people going to mall in five different city, percentage of children and ratio of male and female going to
mall is given.
City Percentage of Ratio of male and Total Number of
children female people going to mall
P 22 17:22 4850
Q 25 13:2 6020
R 40 11:9 4800
S 13 13:16 2600

16. Number of children going to mall in P is b.2569:2369


approximately what percent of the number of c.4569:3260
female going to mall in R? d.3913:1248
a.78.96% e. 3695:1693
b.36.96% 18. Find the difference of total male and children
c.89.45% going to mall in P and the total female and male
d.56.69% going to mall in R?
e.82.33% a.136
17. What is the respective ratio of number of male b.149
going to mall in Q to the number of female going to c.164
mall in S? d.152
a.1258:1369 e.126

Page 75 of 199

Subscribe The Xpress Video Course & Mock Test Package for Bank & Insurance Exams
If there are any suggestions/ errors in our PDFs Feel Free to contact us via this email: admin@exampundit.in
Ultra Practice Bundle PDF
IBPS PO Prelims - Quantitative Aptitude
19. What is the average (approx) of total children 20. Total number of female going to mall in Q is
going to mall in all cities together? what percent more or less than total number male
a.1254 going to mall in P?
b.1207 a.63.49%
c.1321 b.62.23%
d.1254 c.62.02%
e.1452 d.67.23%
e.75.23%
Direction (21-25) Study the table and answer the question.
Mobile sold by five different stores in different month.

Store June July May October


I 240 342 234 210
J 230 234 432 242
K 120 338 235 234
L 280 348 342 178
M 326 456 362 148

21. Find the average (approx) number of mobile a.142


sold by K, L, and M in May? b.125
a.231 c.145
b.256 d.175
c.263 e.123
d.231 23. Number of Mobile sold by store J in July and
e.313 October together is what percent of the mobile sold
22. If 37.5% of mobile sold by store L in June is by store M in May and June?
defective, then find the non defective mobile sold by a.98.33%
store L in June? b.78.32%

Page 76 of 199

Subscribe The Xpress Video Course & Mock Test Package for Bank & Insurance Exams
If there are any suggestions/ errors in our PDFs Feel Free to contact us via this email: admin@exampundit.in
Ultra Practice Bundle PDF
IBPS PO Prelims - Quantitative Aptitude
c.69.18% 25. In October total mobile sold by store I is
d.66.81% 71.42% of total mobile in the shop. Find the
e.56.33% number of unsold mobile in store I in the month
24. What is ratio of the mobile sold by store K and October?
L in June and the mobile sold by I and M in May? a.84
a.29:37 b.86
b.25:37 c.54
c.21:37 d.21
d.100:149 e.26
e.29:31

Direction (26-30) Study the table and answer the question.


Cost price marked up percentage and discount percentage of five different items in a shop is given.
Items Cost price Markup percentage Discount percentage
W 320 37.5% 10%
X 343 28.57% 15%
Y 480 25% 20%
Z 550 20% 15%
V 280 10% 25%

26. What is ratio of selling price of item W and the 27. What is total profit percentage earned by
mark price of item V? shopkeeper by selling items X and Z?
a.5:3 a.9.01
b.3:1 b.8.36
c.7:9 c.6.36
d.3:2 d.4.72
e.9:7 e.2.36

Page 77 of 199

Subscribe The Xpress Video Course & Mock Test Package for Bank & Insurance Exams
If there are any suggestions/ errors in our PDFs Feel Free to contact us via this email: admin@exampundit.in
Ultra Practice Bundle PDF
IBPS PO Prelims - Quantitative Aptitude
28. What is average selling price of all the items b.116.87
together? c.148.36
a.456.2 d.185.96
b.465.3 e.178.23
c.408.4 30. By selling which items shopkeeper make a loss?
d.402.2 a.V
e.432.3 b.W
29. Selling price of item Z is what percent of cost c.X
price of Y? d.Y
a.125.21 e.Z

Direction (31-35) Study the table and answer the question.


Number of students participates in Khelo India from four different universities in two different years.
University 2019 2020
Total number of Percentage of Total number of Percentage of boys
boys participate girls participate girls participate participate
P 360 40% 320 75%
Q 240 60% 300 60%
R 175 75% 270 70%
S 330 45% 240 25%
31. Find the ratio of number of girls participates e.4:5
from university P in 2019 to the number of girls 32. Find the average (approx) number of girls
participates in 2020 from university Q? participate from R and S together in 2020?
a.5:3 a.245
b.9:7 b.285
c.7:8 c.256
d.6:5 d.255

Page 78 of 199

Subscribe The Xpress Video Course & Mock Test Package for Bank & Insurance Exams
If there are any suggestions/ errors in our PDFs Feel Free to contact us via this email: admin@exampundit.in
Ultra Practice Bundle PDF
IBPS PO Prelims - Quantitative Aptitude
e.215 a.20%
33. Find the difference between the numbers of b.25%
boys participates from Q and girls participate from c.32%
R in 2020? d.62%
a.142 e.21%
b.158 35. Find the sum of total girls participate from all
c.180 the college together in 2019?
d.185 a.1395
e.145 b.1256
34. Find the percentage increase in the number of c.1245
student (boys + girls) participate from university Q d.1368
from 2019 to 2020? e.1452

Direction (36-40) Study the table and answer the question.


Total page (Bengali + English) written by A and B in five different months given.
Month A B
May 240 340
June 320 234
July 420 280
August 220 360

36. Total page written by A in May and June d.78


together is approximately what percent of the total e.80
page written by B in May and August together? 37. 28.57% of the total pages written by B in July
a.56 is in Bengali. How many English pages written by B
b.90 are in English?
c.70 a.284

Page 79 of 199

Subscribe The Xpress Video Course & Mock Test Package for Bank & Insurance Exams
If there are any suggestions/ errors in our PDFs Feel Free to contact us via this email: admin@exampundit.in
Ultra Practice Bundle PDF
IBPS PO Prelims - Quantitative Aptitude
b.232 a.31:23
c.268 b.29:35
d.200 c.37:59
e.212 d.39:21
38. What are the average pages written by A in all e.37:31
the month together? 40. How much more percentage of pages written by
a.243 A in June as compared to B in same month?
b.256 a.36.75%
c.300 b.48.36%
d.360 c.25.96%
e.310 d.33.3%
39. What is respective ratio between the total pages e.36.58
written by both A and B in May and the total pages
written by both A and B in July?

Direction (41-45) Study the table and answer the question.


Company XYZ organised a workshop. Number of people and ratio of male and female register for a
Workshop from different city is given.
City Register For workshop Ratio of male and female

Mumbai 1200 5:7

Delhi 950 11:8

Chennai 1800 3:6

Kolkata 2500 12:13

Patna 700 4:3

Indore 800 5:3

Page 80 of 199

Subscribe The Xpress Video Course & Mock Test Package for Bank & Insurance Exams
If there are any suggestions/ errors in our PDFs Feel Free to contact us via this email: admin@exampundit.in
Ultra Practice Bundle PDF
IBPS PO Prelims - Quantitative Aptitude
41. What is the difference between the total number b.11:3
of female register from Mumbai and Delhi together c.13:4
and tthe total number of male register from Patna d.11:7
and Indore together? e.19:7
a.211 44. Total number of male register from Mumbai is
b.259 approximately what percent of total female register
c.256 from Indore?
d.215 a.196.3
e.200 b.166.66
42. What is average number of people register for c.157.5
workshop from Chennai, Kolkata, and Delhi? d.156.3
a.1432 e.161.2
b.1459 45. For female fees of the workshop are 200 and for
c.1285 male are 250.Then how much amount earned by
d.1750 company from Chennai?
e.1258 a.390000
43. What is the respective ratio between the number b.290000
female register in Kolkata and male register in c.250000
Patna? d.360000
a.12:7 e.210000

Direction (46-50) Study the table and answer the question.


Distance (in kms) travelled by five trucks on five different days of the week.

A B C D E
Sunday 280 342 320 360 300
Monday 242 366 250 232 380

Page 81 of 199

Subscribe The Xpress Video Course & Mock Test Package for Bank & Insurance Exams
If there are any suggestions/ errors in our PDFs Feel Free to contact us via this email: admin@exampundit.in
Ultra Practice Bundle PDF
IBPS PO Prelims - Quantitative Aptitude
Tuesday 288 292 240 320 210
Friday 300 264 320 236 238
Saturday 260 340 294 274 286
46. What is the total distance travelled by all the d.282.3
trucks together on Friday? e.276.8
a.1467 49. Speed of the truck A on Sunday was 40 km/hr.
b.1267 Time taken by truck A on Tuesday is 14.28% more
c.1369 than the time taken by truck A on Sunday. What is
d.1234 speed of truck A on Tuesday?
e.1358 a.43
47. If the given travel distance, the time taken by b.36
truck E on Tuesday was 5 hours. What was the speed c.39
of truck on E that day? d.46
a. 52 km/hr e.42
b. 44 km/hr 50. If on Saturday truck B and truck D travelled at
c. 39 km/hr the same speed, what was the respective ratio of time
d. 42 km/hr taken by truck D and time taken by truck B to cover
e. 34 km/hr their respective distances?
48. What was the average distance travelled by truck a.137:170
C in all the days together? b. Cannot be determined
a.270.1 c.139:167
b.274.8 d.123:159
c.284.8 e.129:17

Page 82 of 199

Subscribe The Xpress Video Course & Mock Test Package for Bank & Insurance Exams
If there are any suggestions/ errors in our PDFs Feel Free to contact us via this email: admin@exampundit.in
Ultra Practice Bundle PDF
IBPS PO Prelims - Quantitative Aptitude
Table DI - Answer and Detailed Explanation
Solution (1-5) So the average is 887/5 =177 (approx)
1. Answer: e 4. Answer: b
Total number of student recruited by all the companies in Total number of student recruited by TCS and CTS in
2017 is 2015 is =328+342=670
= (252+120+246+68+124) Total number of student recruited by CGI in 2015 and
= 810 2016 is
2. Answer: d = 230+242=472
Number of student recruited by L&T in 2013 is 124 So the respective ratio is 670:472 =335:236
Number of student recruited by L&T in 2014 is 320 5. Answer: a
So the number of student increase is 320-124 =196 Total number of student recruited by all the company in
So the increase percentage is = (196/124)*100 =158.06% 2014 is
=158% =320+246+126+320+124
3. Answer: b =1136
Total number of student recruited by IBM all over the Recruited by TCS in 2014 are 320
year is So required percentage is = (320/1136)*100 = 28.16%
=123+126+178+214+246=887
Solution (6-10)
Common Explanation:
Total marks obtain by each student given in this chart

EVS MATH SCIENCE ENGLISH Total


Marks
P 50*(64/100)=32 84 75*(56/100)=42 120*90/100 266
=108
Q 50*(84/100)=42 68 75*(84/100)=63 120*55/100 239

Page 83 of 199

Subscribe The Xpress Video Course & Mock Test Package for Bank & Insurance Exams
If there are any suggestions/ errors in our PDFs Feel Free to contact us via this email: admin@exampundit.in
Ultra Practice Bundle PDF
IBPS PO Prelims - Quantitative Aptitude
=66
R 50*(92/100)=46 78 75*(68/100)=51 120*75/100 265
=90
S 50*(72/100)=36 68 75*(64/100)=48 120*70/100 236
=84
T 50*(76/100)=38 84 75*(72/100)=54 120*80/100 272
=96

6. Answer: e So the respective ratio is = 138:120 = 23:20


Total marks obtain by R is 46+78+51+90=265 9. Answer: b
7. Answer: d Difference of total marks of T and total marks of Q is
Total full marks in all subject is 50+100+75+120 =345 = 272-239 =33
Total marks of P in all subject is = 266 10. Answer: a
So percentage of is (266/345)*100 =77.1 Total marks in math and science of P =84+42 =126
8. Answer: c Total marks in math and science of Q=68+63 =131
Total marks of T in Math and Science together is 84+54 Total marks in math and science of R=78+51 =129
=138 Total marks in math and science of S=68+48 =116
Total marks of S in EVS and English together is 36+84 Total marks in math and science of T=84+54 = 138
=120 So T obtains maximum marks in Math and Science.
Solution (11-15)
Common Explanation:

Showrooms Total car Diesel car Petrol car


A 150 150*33.33/100 150-50=100
=50
B 190 190*40/100 190-76=114
=76

Page 84 of 199

Subscribe The Xpress Video Course & Mock Test Package for Bank & Insurance Exams
If there are any suggestions/ errors in our PDFs Feel Free to contact us via this email: admin@exampundit.in
Ultra Practice Bundle PDF
IBPS PO Prelims - Quantitative Aptitude
C 210 210*28.57/100 210-60=150
=60
D 160 160*56.27/100 160-90=70
=90

E 240 240*37.5/100 240-90 =150


=90

11. Answer: e So required percentage is = (60/90)*100 =66.66%


Total number of petrol car in all the showrooms is 14. Answer: b
=(100+114+150+70+150)=584 Petrol car in B is 114.
So the average petrol car is =584/5 =116.8 Diesel car in E is 90.
12. Answer: d So the respective ratio is = 114:90=19:15
Total number of petrol car in A and C are 100 +150 =250 15. Answer: a
Total number of diesel car in D and E are 90+90 =180 Total number of Petrol car in A, C is 100+150=250
So the respective difference is 250-180 =70 Total number of petrol car in A,C, F is 370
13. Answer: c So petrol car in F is 370-250 =120
Petrol car in C is 150. Total number of diesel car in B, D is 76+90 =166
Diesel car in D is 90. Total number of diesel car in B, D, F is 220
So in C, there is 150-90=60 more petrol car than diesel Total number of diesel car in F is 220-166=54
car in D. Total car in F is 120+54=174
Solution (16-20)
City Total person going to Number of Children Total male and Male Female
mall female
P 4850-1067 3783*17/39 3783*22/39
4850 3783 1649 2134

Q 6020 6020*25/100 6020-1505 4515*13/15 4515*2/15


1505 4515 3913 602
Page 85 of 199

Subscribe The Xpress Video Course & Mock Test Package for Bank & Insurance Exams
If there are any suggestions/ errors in our PDFs Feel Free to contact us via this email: admin@exampundit.in
Ultra Practice Bundle PDF
IBPS PO Prelims - Quantitative Aptitude

R 4800 4800*40/100 4800-1920 2880*11/20 2880*9/20


1920 2880 1584 1296
S 2600 2600*13/100 2600-338 2262*13/29 2262*16/29

338 2262 1014 1248

16. Answer: e Total number of mobile sold by store K, L, M in May is


Required percentage = (1067/1296)*100 = 82.33 =235+342+362=939
17. Answer: d So the average mobile sold by these three store in May is
Respective ratio is = 3913:1248 939/3=313
18. Answer: c 22. Answer: d
Total male and children from P going to mall is Total number of mobile sold by store L in June is 280.
1067+1649 =2716 Out of that 37.5% is defective.
Total male and female from R going to mall is So number of non-defective mobile is = 280*62.5/100
1584+1296=2880 =175
So required difference is 2880-2716=164 23. Answer: c
19. Answer: b In store J total mobile sold in July and October is
Total children from all the city together is =234+242 =476
=1067+1505+1920+338=4830 In store M total mobile sold in May and June is
So the average is = 4830/4 =1207.5 1207 =326+362 =688
20. Answer: a So Required percentage = (476/688)*100 =69.18%
So the required percentage is = {(male in P –female in 24. Answer: d
Q)/male in P}*100 Mobile sold by K&L in June is 120+280 =400
= {(1649-602)/1649}*100 Mobile sold by I&M in May is 234+362=596
= 63.49% So respective ratio is = 400:596 = 100:149
Solution (21-25) 25. Answer: a
21. Answer: e Let, total mobile in the store I in October is x.

Page 86 of 199

Subscribe The Xpress Video Course & Mock Test Package for Bank & Insurance Exams
If there are any suggestions/ errors in our PDFs Feel Free to contact us via this email: admin@exampundit.in
Ultra Practice Bundle PDF
IBPS PO Prelims - Quantitative Aptitude
Total mobile sold in October is 71.42% of total mobile in x* , x=294
the store in October.
Number of unsold mobile in October in store I is 294-
So,
210= 84
Solution (26-30)
Common Explanation:
Mark price = {acost price *(100+marked up percentage/100)}
Selling price ={marked price *(100- discount percentage/100)}
Item Cost price Mark price Selling price
W 320 320*137.5/100 440*90/100
=440 =396
X 343 343*128.57/100 441*85/100
=441 =374
Y 480 480*125/100 600*80/100
=600 =480
Z 550 550*120/100 660*85/100
=660 =561
V 280 280*110/100 308*75/100
=308 =231

26. Answer: e Total selling price of all the items together is


Required ratio =396:308=9:7 =396+374+480+561+231=2042
27. Answer: d So the average selling price is 2042/5 =408.4
Total cost price of X and Z is =343+550=893 29. Answer: b
Total selling price of X and Z is=374+561=935 Required percentage is = (561/480)*100=116.87%
So profit percentage is = {(935-893)/893}*100 =4.72% 30. Answer: a
28. Answer: c By selling item V shopkeeper make a loss.
Solution (31 -35)
Page 87 of 199

Subscribe The Xpress Video Course & Mock Test Package for Bank & Insurance Exams
If there are any suggestions/ errors in our PDFs Feel Free to contact us via this email: admin@exampundit.in
Ultra Practice Bundle PDF
IBPS PO Prelims - Quantitative Aptitude
Common Explanation:

University 2019 2020


Number of boys Number of Number of girls Number of boys
participated girls participated Participated
participated
P 360 (360/60)*40 320 (320/25)*75
=240 =960
Q 240 (240/40)*60 300 (300/40)*60
=360 =450
R 175 (175/25)*75 270 (270/30)*70
=525 =630
S 330 (330/55)*45 240 (240/75)*25
=270 =80

31. Answer: e So required difference is = (boys from Q in 2020 – girls


Required ratio = girls from P in 2019: girls from Q in from R in 2020)
2020 =450-270 =180
=240:300 =4:5 34. Answer: b
32. Answer: d Total participated from Q in 2019 is 240+360 =600
Total girls from R and S in 2020 is 270+240 =510 Total participated from Q in 2019 is 300+450 =750
So the average number of girls from R and S in 2020 is So required percentage increase = (150/600)*100 =25%
510/2 =255 35. Answer: a
33. Answer: c Total number of girls participated in 2019 from all the
college together is
=240+360+525+270=1395
Solution (36-40)

Page 88 of 199

Subscribe The Xpress Video Course & Mock Test Package for Bank & Insurance Exams
If there are any suggestions/ errors in our PDFs Feel Free to contact us via this email: admin@exampundit.in
Ultra Practice Bundle PDF
IBPS PO Prelims - Quantitative Aptitude
36. Answer: e So average pages written by A is = 1200/4 =300
Total page written by A in May and June is 39. Answer: b
240+320=560 Total pages written by A and B in May is = 240+340
Total page written by B in May and August =580
340+360=700 Total pages written by A and B in July is = 420+280
So the required percentage is =(560/700)*100 =80 =700
37. Answer: d So the respective ratio is = 580:700=29:35
Total page written by B in July is 280. Out of that 40. Answer: a
28.57% is written in Bengali. Total pages written by A in June are 320. Total pages
Bengali page written by B is 280*28.57/100 =80 written by B in June are 234.
So English page written by B in July is 280-80=200 So A written (320-234) =86 more pages.
38. Answer: c So the required percentage is (86/234)*100 =36.75%
Total pages written by A in all the month together is =
240+320+420+220 =1200
Soluion (41-45)
Common Explanation:

City Total register Male female


for workshop
Mumbai 1200 1200*5/12 =500 1200-500=700
Delhi 950 950*11/19=550 950-550 =400
Chennai 1800 1800*3/9=600 1800-600=1200
Kolkata 2500 2500*12/25=1200 2500-1200=1300
Patna 700 700*4/7=400 700-400=300
Indore 800 800*5/8=500 800*3/8=300
41. Answer: e

Page 89 of 199

Subscribe The Xpress Video Course & Mock Test Package for Bank & Insurance Exams
If there are any suggestions/ errors in our PDFs Feel Free to contact us via this email: admin@exampundit.in
Ultra Practice Bundle PDF
IBPS PO Prelims - Quantitative Aptitude
Total female register from Mumbai and Delhi is Total distance travelled by E on Tuesday is 210.Time
700+400=1100 taken by E to cover that distance is 5 hour.
Total male register from Patna and Indore is So the speed of the truck E on Tuesday is =210/5=42
400+500=900 km/hr
So the respective difference is 1100-900 = 200 48. Answer: c
42. Answer: d Total distance travelled by C in all the days together is =
Total number of register from Chennai, Kolkata, Delhi is (320+250+240+320+294)=1424
= 1800+2500+950 = 5250 So the average travelled by C is =1424/5=284.8
So the average is 5250/3 = 1750 49. Answer: b
43. Answer: c Time taken by A to cover the distance on Sunday is
Respective ratio is 1300:400 =13:4 280/40 =7 hr.
44. Answer: b Time taken by A to cover the distance on Tuesday is
Required percentage is (500/300)*100 =166.66 7*(114.28/100) = 8hr
45. Answer: a So the speed of truck A on Tuesday is = 288/8 = 36
Total earn by the company from Chennai is km/hr
= {(200*1200) + (250*600)} = 240000+150000=390000 50. Answer: a
Soluion (46-50)Common Explanation: Let, the speed of truck B and D on Saturday is S.
46. Answer: e So,
Total distance travelled by all the truck on Friday is Time taken by B is (t1) = 340/S
= (300+264+320+236++238) Time taken by D is (t2) =274/S
=1358 So respective ratio is = t2:t1 =(274/S) : (340/S) =
47. Answer: d 137:170

Data Interpretation Practice Questions PDF Download


Download Quantitative Aptitude Practice Questions PDF

Page 90 of 199

Subscribe The Xpress Video Course & Mock Test Package for Bank & Insurance Exams
If there are any suggestions/ errors in our PDFs Feel Free to contact us via this email: admin@exampundit.in
Ultra Practice Bundle PDF
IBPS PO Prelims - Quantitative Aptitude
Bar DI
Directions (Q. 1 -5) Study the graph and answer the question.
Number of 6’s and 4’s hit by five team in IPL 2020 is given below.

140 130
112
120 108 110
102 104
98
100
84 82
78
80
6's
60 4's

40

20

0
CSK MI RCB DC SRH

1. Total 6’s hit by CSK and SRH is how much a.16


percent more or less than total 4’s hit by All b.17
teams together? c.14
a.72.54 d.15
b70.56 e. None of these
c.45.67 3. Find the Ratio of total 4’s hit by SRH, MI and
d.77.88 DC to total 6’s hit by all teams together?
e. None of these a.21:29
2. Average number of 6’s in IPL 2020 hit by MI, b.23:29
DC, and KKR is 89 and Avearge of 4‘s in IPL 2020 c.29:37
hit by SRH, RCB, and KKR is 120. Then find the d.29:12
difference between total boundaries (6’s+4’s) hit by e. None of these
CSK and KKR?
Page 91 of 199

Subscribe The Xpress Video Course & Mock Test Package for Bank & Insurance Exams
If there are any suggestions/ errors in our PDFs Feel Free to contact us via this email: admin@exampundit.in
Ultra Practice Bundle PDF
IBPS PO Prelims - Quantitative Aptitude
4. MI hit 25% more 4’s and 33.33% less 6’s in 2019 5. Which team’s difference between 4’s and 6’s is
then in 2020. What is total boundary (4’s+6’s) hit Maximum?
by MI in 2019? a.DC
a.201 b.RCB
b.202 c.MI
c.204 d. RCB and MI
d.203 e. None of these
e. None of these
Directions (Q. 6-10) Study the graph and answer the question.
Salary of four friends is same i.e 25000. Their expenditiure in internet, food, rent and savings out of salary is given in
%.
120

100

80
SAVINGS
60 ROOM RENT
FOOD
40
INTERNET
20

0
A B C D

6. Find the ratio between expenditure of food and c.13:10


room rent of A and the expenditure of internet and d.12:13
food of D. e. None of these
a.10:11 7. C gets promotion so his salary is increased by
b.12:11 25%. Now he spends 20% on internet, 15% on

Page 92 of 199

Subscribe The Xpress Video Course & Mock Test Package for Bank & Insurance Exams
If there are any suggestions/ errors in our PDFs Feel Free to contact us via this email: admin@exampundit.in
Ultra Practice Bundle PDF
IBPS PO Prelims - Quantitative Aptitude
food, 25% on room rent and rest is savings. What is e. None of these
the difference between savings on Salary before 9. Who spends minimum in food and room rent
and after Promotion? together.
a.4500 a.A
b.3250 b.B
c.3750 c.C
d.3560 d.A&B
e. None of these e. None of these
8. If D work from home for a month and he did not 10. A uses airtel for internet and C uses Jio. In a
spend any money for room rent. But internet bill month airtel gives 5% discount and Jio gives 10%
increases by 10 % and food bill increases by 5%. discount. What is the difference between amount
What is the ratio of amount he spends for food to spend by A and C on internet after discount?
internet? a.3625
a.12:5 b.4456
b.11:7 c.3460
c.13:11 d.4500
d.7:13 e. None of these

Page 93 of 199

Subscribe The Xpress Video Course & Mock Test Package for Bank & Insurance Exams
If there are any suggestions/ errors in our PDFs Feel Free to contact us via this email: admin@exampundit.in
Ultra Practice Bundle PDF
IBPS PO Prelims - Quantitative Aptitude
Directions (Q.11-15) Study the graph and answer the question.
Rate of interest of both Simple interest (SI) and Compound interest (CI) for five different companies are given.
35

30

25

20
SI
15 CI

10

0
P Q R S

11. Ram invests 5000 for 2 year at Q on CI. At the b.2450


end of 2 year he invests total amount received from c.2800
Q at S on SI for 3 years. How much he get as intrest d.2850
from S? e.2750
a.3888 13. What amount a person gets as total interest if
b.4351 he invests 10000 at P in CI and 18000 at S in SI for
c.3788 2 year?
d.5648 a.7474
e. None of these b.7674
12. Joy invest 20000 at R on SI for two year. If he c.7644
invest the same amount at Q on CI for 2 year then d.7862
find the difference of interest he gets from Q and R. e.7642
a.2400
Page 94 of 199

Subscribe The Xpress Video Course & Mock Test Package for Bank & Insurance Exams
If there are any suggestions/ errors in our PDFs Feel Free to contact us via this email: admin@exampundit.in
Ultra Practice Bundle PDF
IBPS PO Prelims - Quantitative Aptitude
14. If A invest 5000 at S on CI for 3 year then what some years. If the interest he gets is 1820 then he
is the total amount he gets after 3 years? invests for how many year?
a.12450 a.2.5
b.10985 b.3
c.12345 c.2
d.14325 d.4
e. None of these e.5
15. Raj invests 5000 at P in SI for 5 year and The
interest he get at P later invested at Q on CI for

Directions (Q. 16-20) Study the graph and answer the question.
Percentage of three types of flower trees of three different gardens is given. Total no.of flower trees of different
gardens may not be same.
100%
16.67 20 12.5
25
80%
43.75 62.5
60% 50 40 35 ORCHID

40% LILY
12.5
20% 40 43.75 40 ROSE
33.33 25
0%
W X Y Z V

16. In garden Y there is total 960 flower trees. Due a.150. b.250. c.450. d.300. e. None of these
to heavy rain 25% lily, 20% rose, and 33.33% 18. If P = lily at X and Q = rose at Z then find the
orchid flower trees have been destroyed. Find ratio value of P+Q/P-Q. (Total trees at X is 700 and at
of orchid and rose flower trees left? flower Z is 600)
a.16:63. b.8:33. c.12:61. d.13:67 e. None a.16. b.14 c.13. d.19 e. None of these
17. What is the difference between the orchid at V 19. Total flower at W, X, Y, Z, and V is 660, 700,
and the rose at W, If the total flower trees in V is 960, 600, and 800 respectively. Find the ratio
800, and in W is 50% more than V. between total rose to total lily?
Page 95 of 199

Subscribe The Xpress Video Course & Mock Test Package for Bank & Insurance Exams
If there are any suggestions/ errors in our PDFs Feel Free to contact us via this email: admin@exampundit.in
Ultra Practice Bundle PDF
IBPS PO Prelims - Quantitative Aptitude
a.67:61. b.68:67. c.59:61. d.67:68. e. None of these a.8.33%. b16.67%. c.20%. d.9.11%. e. None
20. Orchid in W is how much percent more or less
than Orchid in Y? (Number Trees is 660 and 960
for W and Y respectively)

Directions (Q. 21-25) Study the graph and answer the question.
There is five store which sells two company’s footballs. 1st graph show’s total football in different sports stores, and
2nd graph shows percent of Nike football out of total football in different sports stores.

Total Football
800
650 720
570
600
450
400
270
200

0
P Q R S T

80

70

60

50

40

30

20

10

0
P Q R S T

Percentage of NIKE

Page 96 of 199

Subscribe The Xpress Video Course & Mock Test Package for Bank & Insurance Exams
If there are any suggestions/ errors in our PDFs Feel Free to contact us via this email: admin@exampundit.in
Ultra Practice Bundle PDF
IBPS PO Prelims - Quantitative Aptitude
21. Adidas football sold in P is what percent more e. None of these
and less then Adidas football sold in S? 24. What the difference between all Adidas football
a.287.78%. b288.88%. c.268% and all NIKE football in all store?
d.258.88%. e. None of these a.235. b.335. c.336. d.435. e. None of these
22. If R’s Marke price of NIKE football is 450 but it 25. If 11.11 % of Nike ball at store T is defected but
gives 30% discount per ball. What is the total price Out of the rest 75% is sold. In store T none of the
of all NIKE balls at store R? Addidas ball is defected and 50% is sold. What is
a.72340. b.98750. c.90720 ratio of sold Adidas to sold Nike?
d.96750. e. None of these a.2:5
23. Find the difference of average nikes ball at Q b.2:1
and T together to the average adidas ball at R and c.2:3
S together? d.3:4
a.13.5. b.29.5. c.16.6. d.18.5 e. None of these
Directions (Q25- 30) Study the graph and answer the question.
Number of males and females of five different city is given.
7000

6000

5000

4000

3000

2000

1000

0
M N O P Q
FEMALES MALE

Page 97 of 199

Subscribe The Xpress Video Course & Mock Test Package for Bank & Insurance Exams
If there are any suggestions/ errors in our PDFs Feel Free to contact us via this email: admin@exampundit.in
Ultra Practice Bundle PDF
IBPS PO Prelims - Quantitative Aptitude
26. Ratio between total males and total females of d.5000
all the city together is? e. None of these
a.31:43 29. If 30% male is graduated and 40% female is
b.31:47 graduted in N and 40% male is graduated and 60%
c.43:31 female is graduted in O then what percent of non
d.47:31 graduated people of N is to the total graduate
e. None of these dpeople of O?
27. 25% of female in city M is doing Phd and is city a.122.35
Q 30% female doing Phd. Find the ratio between b.121.42
total female doing phd to the total male from those c.119.6
two cities. d.123.6
a.4:5 e. None of these
b.2:3 30. In City O, where every one casts vote, total
c.5:4 elegible voters is 80% but 10% votes are invalied.
d.2:1 Out of valid votes 60% votes get by wining
e. None of these candidate. How much vote he win in the election?
28. If 60% of male in city p is going to see movie a.631
and 40% female is going see the movie. Find the b.641
total no.of male and female who don’t go to movie? c.651
a. 6300 d. 655
b.5200 e. None of these
c.4500

Directions (Q.31-35) Study the graph and answer the question.


Percentage of different type of trains running in four different type station is given.
(Total train running at Howrah = 800, Mumbai = 1200, Delhi = 900 and Chennai =720)

Page 98 of 199

Subscribe The Xpress Video Course & Mock Test Package for Bank & Insurance Exams
If there are any suggestions/ errors in our PDFs Feel Free to contact us via this email: admin@exampundit.in
Ultra Practice Bundle PDF
IBPS PO Prelims - Quantitative Aptitude
100%
12.5
25
80%
50 25
60
60% 35

40% 16.67
62.5 25
20% 40 33.33
15
0%
howrah mumbai chennai delhi

express superfast mail

31. If in a day 20% express trains are not running 33. Find the ratio of superfast trains running at
due technical problem. Find the ratio of total Howrah and Chennai together to express train in
express train running in Mumbai in that day to Delhi and Mumbai together.
total mail train running in delhi. a.92:107
a.16:27 b. 97:93
b.13:29 c.91:103
c.31:19 d.49:47
d.27:16 e. None of these
e. None of these 34. Due to heavy rain 40% of trains is cancelled in
32. What is total mail trains running in all the mumbai. The ratio of superfast, express and mail
station together? trains cancel is 4:5:6. What is no.of express train
a.1620 running in Mumbai?
b.1430 a.320
c.1580 b.222
d.1222 c.220
e. None of these d.240
e. None of these

Page 99 of 199

Subscribe The Xpress Video Course & Mock Test Package for Bank & Insurance Exams
If there are any suggestions/ errors in our PDFs Feel Free to contact us via this email: admin@exampundit.in
Ultra Practice Bundle PDF
IBPS PO Prelims - Quantitative Aptitude
35. What is % of Total mail train in Howrah to c.168.88
express train in Delhi? d.124.44
a.158.89 e. None of these
b.148.18

Directions (Q.36-40) Study the graph and answer the question.


Speed of five different trains is given.

C
Speed per hour

0 10 20 30 40 50 60

36. Train A and B is moving in opposite direction e.27.01


and their lenth is 250m and 300m respectively. 37. Train C starts from P to Q at 8 am and Reaches
What is total time taken by them to cross each Q at 1.30 pm. Find the distance between P and Q?
other? a.234
a.21.06 b.264
b.22 c.254
c.23 d.234
d.25.01 e. None of these
Page 100 of 199

Subscribe The Xpress Video Course & Mock Test Package for Bank & Insurance Exams
If there are any suggestions/ errors in our PDFs Feel Free to contact us via this email: admin@exampundit.in
Ultra Practice Bundle PDF
IBPS PO Prelims - Quantitative Aptitude
38. Ratio of the time taken by D to go 630 km to B b. 55
to go 840 km is? c.45
a.5:4 d.43
b.7:10 e. None of these
c.3:7 40. Train A is crosses a man in 10 sec. What is the
d.3:8 length of train A?
e. None of these a.124.44
39. To cross a 325m long platform D take 50 sec. b.144.44
What is time taken by B to cross the same platform c.134.32
if ratio of length of train D and B is 3:2? d.144.14
a.40 e. None of these

Directions (Q 41-45) Study the graph and answer the question.


Area of four different shapes is given.
700 616
600
600
500
400
300 AREA
216
200 144
100
0
SQUARE TRIANGLE RECTANGLE CRICLE

41. If base of the triangle is equal to the side of d.28


square then hight of the triangle is? e. None of these
a.42 42. What is ratio of diameter of circle and the side
b.34 of the square?
c.36 a.3:5

Page 101 of 199

Subscribe The Xpress Video Course & Mock Test Package for Bank & Insurance Exams
If there are any suggestions/ errors in our PDFs Feel Free to contact us via this email: admin@exampundit.in
Ultra Practice Bundle PDF
IBPS PO Prelims - Quantitative Aptitude
b.3:7 44. Ratio of length and breath of rectangle is 3:2.
c.5:7 Find the ratio of perimeter of rectanagle to the
d.3:2 perimeter of square?
e. None of these a.12:25. b.13:23 c.23:19. d.12:23
43. If the Circle is maked to a square then what is e. None of these
the area of square? 45. Average area of all the four shape is?
a.421. b.576. c.324. d.484 a. 375 b.394 c. 384 d. 424
e. None of these
e. None

Directions (Q.46-50) Study the graph and answer the question.


No.of Students (in hundred) take admission in four different colleges in three different streams in each college.

Commerce
science
N
Arts

0 10 20 30 40 50 60 70
number of student( in hundred)

46. At college P out of total student take addmistion Find the ratio of boys in commerce to the boys in
in arts 60% is girls and in commerce 40% is girls. arts at the same?
a.23:19
Page 102 of 199

Subscribe The Xpress Video Course & Mock Test Package for Bank & Insurance Exams
If there are any suggestions/ errors in our PDFs Feel Free to contact us via this email: admin@exampundit.in
Ultra Practice Bundle PDF
IBPS PO Prelims - Quantitative Aptitude
b.27:19 c.5500
c.24:19 d.3400
d.19:23 e. None of these
e. None of these 49. What is ratio of total commerce students to Arts
47. What is the difference of average of science students in all the colleges together?
student of M and N together to the average of Arts a.57:61
students in N and O together? b.53:41
a.350 c.59:40
b.450 d.31:40
c.560 e. None of these
d.320 50. What is the difference of total students in
e. None of these college M and total students in college P?
48. Out of total students take admission in arts at a. 22600
College O 40% perform in fest and the same for b.23200
science is 60%. What is the total no.of students c.23400
perform in the fest? d.23400
A.5600 e. 24300
b.4600

Bar DI - Answer and Detailed Explanation


Solution (1 -5) = {(564-166)/564}*100
1. Ans: b =70.56%
Total 6’s hit by CSK and SRH is = (84 +82) =166 So, 6’s hit by csk and srh is 70.56% than 4’s hit by all
Total 4’s hit by all the team together is = the team together.
(112+108+104+110+130) = 564 2. Ans: b
So required percentage is Average 6’s hit by MI, DC and KKR is 89.
= {(difference of 4’s hit by all team - 6’s hit by CSK So total 6’s hit by hese three team is 89*3=267
and SRH)/ 4’s hit by all the team}*100 Total 6’s hit by MI and Dc is (102+78) = 180
Page 103 of 199

Subscribe The Xpress Video Course & Mock Test Package for Bank & Insurance Exams
If there are any suggestions/ errors in our PDFs Feel Free to contact us via this email: admin@exampundit.in
Ultra Practice Bundle PDF
IBPS PO Prelims - Quantitative Aptitude
So 6’s hit by KKR is = 267-180 =87 =104-98 = 6
Now average 4’s hit by SRH, RCB, and KKR is 120. DC difference between 4’s and 6’s
Total 4’s hit by SRH, RCB, and KKR is 120* 3 =360 = 110-78 =32
Total 4,s hit by srh and rcb is =104+130 =234 SRH difference between 4’s and 6’s
So the total 4’s hit by KKR is 360 – 234 =126. =130 -82 =48
Total boundaries hit by KKR is =87 + 126 = 213 Solution 6-10:
Total boundaries hit by CSK is =84 +112 =196 6. Ans: c
So the difference of boundaries of KKr and CSK is Total expenditure of A in FOOD and Roon rent is =
=213-196=17 25000*65/100 =16250
3. Ans: c Total expenditure of D in internet and food is =
Total 4’s hit by SRH, MI, DC is = (130+108+110) = 25000*50/100=12500
348 So the respective ratio is = 16250:12500 =13:10
Total 6’s hit by all the team together is = 7. Ans: c
(84+102+98+78+82) = 444 In previous C’s saveing is 25000*35/100 =8750
So required ratio is = (348:444) = 29:37. Now his salary is 25000*125/100 =31250
4. Ans: d Now he spend 20% internet, 15% food 25% room rent
MI hit 4’s in 2019 is = (108*125)*100 Total he spend 60% so he save 40%.
=135 Saveing amount is now = 31250*40/100=12500
MI hit 6,s in 2019 is = (102 *66.67)/100 So he save 12500-8750 = 3750 amount more.
=68 8. Ans: b
Total boundaries hit by =135 +68 =203 D’s internet spend was 25000*30/100 =7500
5. Ans e Now internet bill 7500*110/100=8250
CSk difference between 4’s and 6’s He was spend in food 25000*20/100 =5000
= 112-84 =28 Now spend 5000*105/100 =5250
MI difference between 4’s and 6’s So respective ratio is 8250: 5250 =11:7
=108-102 =6 9. Ans: e
RCB difference between 4’s and 6’s Sallary of every person is same.

Page 104 of 199

Subscribe The Xpress Video Course & Mock Test Package for Bank & Insurance Exams
If there are any suggestions/ errors in our PDFs Feel Free to contact us via this email: admin@exampundit.in
Ultra Practice Bundle PDF
IBPS PO Prelims - Quantitative Aptitude
Now from we see the percentage distribution. 13. Ans: c
IN food and room rent A spend 65 %, B 65%, C 60%, From P the man get interest (A) = [{10000*{1 +
and D 55%. (8/100)} ^ 2] - 10000 =1664
From this we can say D spend minimum. From S he get interest (B) = 18000*2*18/100 =6480.
10. Ans: a Total interest (A+B) he get = 1664+6480 =7644
A spend in internet 25000*20/100=5000 14. Ans: b
After getting 5% discount A have to pay 5000 invest at S in CI for 3 year.
5000*95/100=4750 So total amount he get = 5000{1+ (30/100)} ^ 3
C spend in internet 25000*5/100 =1250 =10985
After getting 10% discount C spend is = 1250*90/100 15. Ans: b
=1125 From P he get interest = 5000*5*10/100 =2500
So the difference after getting discount is = (4750- Now 2500 invest at Q in CI for some year .get interest
1125) =3625 1820.
Solution 11- 15 So,
11. Ans: a 1820= 2500{1 + (20/100)} ^ n -2500
From Q, Ram get =5000{1+ (20/100)} ^ 2 So, 4320 = 2500{1 + (20/100)} ^ n
= (5000*120*120)/10000 {1 + (20/100)}^n = 4320/2500
=7200 So the value of n is 3.
Now ram invest 7200 at in SI for 3 year. Solution 16- 20
So interest he get = (7200*18*3)//100 =3888 16. Ans: a
12. Ans: c In Y total flower is 960.
Joy invest 20000 at R in SI for 2 year. Out of rose is = 960*43.75/100 = 420
So interest he get is (I) = (20000*15*2)/100 =6000 Lily is = 960 *43.75/100 =420
Now if he invest same amount at Q in CI interest he Orchid is =960 *12.5/100 =120
get is (J) Now rose left = 420 *75/100 = 315
= 20000{1 + (20/100)} ^ 2 -20000 = 8800 Orchid left = 120*66.67/100 = 80
So the difference J-I is 8800-6000= 2800. Than ratio of orchid and rose is 80:315 = 16:63.

Page 105 of 199

Subscribe The Xpress Video Course & Mock Test Package for Bank & Insurance Exams
If there are any suggestions/ errors in our PDFs Feel Free to contact us via this email: admin@exampundit.in
Ultra Practice Bundle PDF
IBPS PO Prelims - Quantitative Aptitude
17. Ans: d So, required percentage is = (234/81)*100 =288.88%
Total flower at V is 800. 22. Ans c
Total flower at W is 800*150/100= 1200 At R, mrp of NIKE football is 450.
Orchid at V is 800*62.5/100 =500 After giving 30% discount its price is (450*70)/100 =
Rose at W is 1200 *16.67/100 =200 315
So the difference is 500-200= 300 Total price of all the Nike football at R is = 45 *7*288
18. Ans: c = 90720
Value of P is Lily at X. 23. Ans d
P= 700*40/100 =280 Total Nike at Q and T is =208+342 =550
Value of Q is Rose at Z. So the average is 550/2 = 275.
Q=600 *40/100 = 240 Total adidas at R and S is 432+81 =513
P+Q/P-Q =520/40 =13. So the average is = 513/2 = 256.5
19. Ans: b So difference is of average is 275-256.5 =18.5
So from, from table total rose is = 24. Ans c
(220+280+420+240+200) =1360 Total Adidas football all the store is =
Total lily is = (330+280+420+210+100) =1340 (315+442+432+81+228) = 1498
So the rose: lily =1360:1340 = 68:67. Total Nike football all the store is =
20. Ans a (135+208+288+189+342) =1162
Orchid at W is = 660*16.67 /100 =110 So the difference of total adidas and total Nike is
Orchid at Y is = 960*12.5 /100 = 120 (1498 -1162) = 336
So required percentage is = {(120-110) /120}*100 = 25. Ans b
8.33% At T 11.11% football is leak.
So not defected is 88.89%.
Solution 21 - 25 Not defected football is = (342 *88.89)*100 =304
21. Ans b Out of this 75% sold.
No. of addidas at P is 315. And S is 81. So at P is Total sold is =304*75/100 =228
(315-81) =234 more than adidas at S. Where adidas football sold is = 228 *50/100 =114

Page 106 of 199

Subscribe The Xpress Video Course & Mock Test Package for Bank & Insurance Exams
If there are any suggestions/ errors in our PDFs Feel Free to contact us via this email: admin@exampundit.in
Ultra Practice Bundle PDF
IBPS PO Prelims - Quantitative Aptitude
So the ratio of sold adidas to nike is 228: 114 = 2:1 40% female graduate so 60% female is not graduate.
Solution 26- 30 No of female not graduate is = 2500*60/100 =1500
26. Ans a So total not graduate from N is 1050+1500 = 2550
Total male all the city together is = From O graduate male is 40% and female is 60%.
(5000+1500+3000+6500+5500) =21500 Total graduate =
Total female all the city together is = {(3000*40/100)+(1500*60/100)}=1200+900=2100
(4500+2500+1500+4000+3000) =15500 Percentage of Non graduate people of N with the
Ratio of total male to female is 21500:15500 = 43:31 graduate people from O is
27. Ans c = (2550/2100)*100
In M, 25% female doing phd. And in Q 30% doing = 121.42%
phd. 30. Ans b
Female doing phd in M is = 4500*25/100 =1125 Total people of O is 1500+3000 =4500
Female doing phd in Q is =3000*30/100 =900 Total voter is 4500*80/100 =45 *80
So the ratio between doing phd in M and Q female is 10%vote is rejected. Total valid vote is 90%.
1125:900 = 5:4 So the total valid vote is 45*80*90/100 = 45*8*9
28. Ans d Wining candidate got 60% vote. So loseing candidate
60% of male of P go to see movie. so 40% of male gote 40%.so the difference of their vote is 20%.
don’t go to movie. 20% of valid vote is 45*8*9*20/100 =641
So no of male do not go is = 6500*40/100 =2600 By 641 vote wiining candidate win the election.
40% female go to movie. So 60% don’t go. Solution 31- 35
So no of female don’t go movie is 4000*60/100 = 31. Ans a
2400 20% express train is not running. express train running
So total no people do not go to see movie is is 400*80/100 =320
2600+2400 = 5000 Total mail running in delhi is 540.
29. Ans b So the ratio is 320:540 = 16:27
30% male graduate at N. so 70% male not graduate. 32. Ans b
No male no graduate is =1500*70 /100 = 1050

Page 107 of 199

Subscribe The Xpress Video Course & Mock Test Package for Bank & Insurance Exams
If there are any suggestions/ errors in our PDFs Feel Free to contact us via this email: admin@exampundit.in
Ultra Practice Bundle PDF
IBPS PO Prelims - Quantitative Aptitude
total mail train running in all the station together is =( time taken by D is 630 /45 = 14
200+600+90+540) Time taken by B is 840/42 =20
=1430. So the ratio 14:20 = 7:10
33. Ans a 39. Ans c
total superfast in Howrah and Chennai is 280+180 D cross 325 m length in 50 sec.
=460 So 325 + L= 50*45*5/18
Total express train in Mumbai and Delhi is L=300 (length of D)
(400+135) = 535 Length D and B is 3:2.
So the ratio 460:535 = 92:107 Legth of B is 200.
34. Ans d So B take = 525 / (42*5/18) = 45 sec
40% train cancel.i.e 1200*40/100 = 480 train is cancel. 40. Ans b
Out of 480 express train is 480 *5/15 =160 Speed of train A is 52 km/h
So express train running is 400-160 =240. =52*5/18 m/sec
35. Ans b So the length of the trtain is =52*5*10/18 = 144.44
Total mail train in howrah is 200 Solution 41- 45
And total express train in delhi is 135. 41. Ans c
Mail train in howrah is (200/135)*100 =148.14% of area of square is = side ^2 = 144
express in delhi Side =12 unit.
Solution 36- 40 Base of triangle is 12 unit
36. Ans a Area of triangle ½* base *hight =216
Total length they have to cover is (250+300) = 550 m So hight is 36 unit.
Relative velocity is (52+42)*5/18 =26.11 42. Ans b
Total time taken = 550/26.11=21.06sec side of square 12 unit.
37. Ans b Area of circle is 616 = 22/7 * r^2
Total time taken by C is 5 hr 30 min. So r =14 so diameter is 28.
So the distance is 48 *5(1/2) =264 km So the ratio is 3:7.
38. Ans b 43. Ans d

Page 108 of 199

Subscribe The Xpress Video Course & Mock Test Package for Bank & Insurance Exams
If there are any suggestions/ errors in our PDFs Feel Free to contact us via this email: admin@exampundit.in
Ultra Practice Bundle PDF
IBPS PO Prelims - Quantitative Aptitude
radius of the circle is 14 47. Ans b
So perimeter is 2*22/7*14= 88 unit Total science from M and N is (5000+6000) = 11000
Perimeter of square is =4*side=88 so side is 22. So avg science this two college =11000/2 =5500
So area is = 484 unit^2 Total arts from N and O is =11900
44. Ans a So the avg is =5950
Perimeter of square is 12*4=48 So difference of average is =5950-5500 = 450
Let length of rectangle is 3x and breadth is 2x. 48. Ans a
6x^2 is =600 40% of arts at O is = 6500 * 40 / 100
x=10 = 2600
length is 30 breadth is 20 so perimeter is 100 60% of science At O is
so the ratio of perimeter is 48:100 =12:25. =5000*60/100
45. Ans b =3000
Total area of all the shape is = Total student perform in fest is 5600
(144+600+216+616)=1576 49. Ans c
So the average is 1576/4= 394 Total arts student is
Solution (46-50) =(3000+4400+6500+3800)
Arts Science Commerce =17700
M 3000 5600 2500 Total commerce student is=(2500
N 4400 6000 3000 +3000+3300+3200)
O 6500 5000 3300 =12000
P 3800 4500 3200 So the ratio is 17700:12000
46. Ans c =59:40
Boys in Arts at P is 3800*40/100 =1520(60% is girl) 50. Ans a
Boys in commerce at P is = 3200*60/100 =1920(40% total student at M and total student At
is girl) P is
Ratio of commerce boys to the arts boys is =1920: (3000+5600+2500+3800+4500+3200)
1520 =24:19 =22600

Page 109 of 199

Subscribe The Xpress Video Course & Mock Test Package for Bank & Insurance Exams
If there are any suggestions/ errors in our PDFs Feel Free to contact us via this email: admin@exampundit.in
Ultra Practice Bundle PDF
IBPS PO Prelims - Quantitative Aptitude
Line DI
Directions (1 – 5): Read the following information carefully and answer the following questions based on it.
There are two types of Air conditioners sold by 5 different companies: Voltas, Blue star, LG, Daikin and Haier. The
following Line graph given information about total number air conditioners sold by the company and total number of
Split AC sold.
Total AC = Total Window AC + Total Split AC
9000 8430

8000 7296

7000

6000 5430
4890
5000
3963
4000
2955 2784
3000 1917

2000 1479

1000
870
0
Voltas Blue Star LG Daikin Haier

Total AC sold Total number of Spilt AC Sold

1) If split AC are two types A and B for each b) 27.27%less


company and window AC are two types for each c) 45.45% more
company type X and type Y. The number of type A d) 18.18% less
split AC sold by Voltas are 1713 and number of type e) None of these
Y Window AC sold by Voltas are 1683 then find type 2) Find the difference between average number of
B split AC sold by Voltas is approx. how much % total AC sold by LG, Daikin and Haier together and
more or less than type X window AC sold by Voltas? average number of windows AC together by same
a) 36.36% more companies?
Page 110 of 199

Subscribe The Xpress Video Course & Mock Test Package for Bank & Insurance Exams
If there are any suggestions/ errors in our PDFs Feel Free to contact us via this email: admin@exampundit.in
Ultra Practice Bundle PDF
IBPS PO Prelims - Quantitative Aptitude
a) 2393 AC. Find the total revenue generated by LG from
b) 2483 Split AC and Blue star from window AC?
c) 2593 a) 293.58 lakhs
d) 2793 b) 283.58 lakhs
e) None of these c) 273.58 lakhs
3) Total number of Windows AC sold by Daikin and d) 263.58 lakhs
blue star together is how much % more or less than e) None of these
total number of Window AC sold by Haier? 5) If 75% of total Split AC manufactured by all
a) 0.4% more companies together is Sold and 90% of total window
b) 6% less AC manufactured by all companies together are sold,
c) 0.5% more then find total number of AC manufactured by all
d) 0.6% less companies together?
e) None of these a) 31176
4) If price of each LG window AC is Rs. 15000 which b) 31186
is 25% more than price of each LG Split AC and c) 31196
price of each Blue star Split AC is Rs. 8000 which is d) 31296
33.333% more than price of each Blue star Window e) None of these
Directions (6 – 10): Read the following information carefully and answer the question based on it.
There are content developer A, B, C, D and E, those developed question of Quantitative Aptitude (QA) and Logical
reasoning (LR) for Banking Entrance exams. The following line shows the number of QA and LR questions
developed by five content developers and percentage of questions that are selected on Mock test in month of April.

Page 111 of 199

Subscribe The Xpress Video Course & Mock Test Package for Bank & Insurance Exams
If there are any suggestions/ errors in our PDFs Feel Free to contact us via this email: admin@exampundit.in
Ultra Practice Bundle PDF
IBPS PO Prelims - Quantitative Aptitude
1800 1700
1600
1600 1500
1640
1400 1250
1200 1000 1000 1300
1000 1150

800 900

600
400
200 60% 45% 40% 65% 50%
0
A B C D E

Quantitative Aptitude Questions


Logical Reasoning Questions
Percentage of Questions selected for Mock test out of total questions

6) For Content Developer E, if number of QA question selected, then find the total revenue
question selected in mock test is 20% less than generated by content developer D?
number of LR questions selected in mock test. Find a) 5.359 lakhs
number of QA questions selected in mock test is how b) 5.371 lakhs
much % of number of QA question developed by E? c) 5.369 lakhs
a) 43.90% d) 5.367 lakhs
b) 45% e) None of these
c) 37.5% 8) Total number of questions selected in mock test by
d) 40% all content developer together is approx. how much
e) None of these % less than total number of questions developed by
7) For content developer D, if 40% of number of all content developers together?
selected question are of QA. Company pay Rs. 250 for a) 48.3%
per QA selected question and Rs. 325 per LR b) 50.3%
c) 45.3%

Page 112 of 199

Subscribe The Xpress Video Course & Mock Test Package for Bank & Insurance Exams
If there are any suggestions/ errors in our PDFs Feel Free to contact us via this email: admin@exampundit.in
Ultra Practice Bundle PDF
IBPS PO Prelims - Quantitative Aptitude
d) 53.3% e) 10%
e) None of these 10) For content developer B, 45% of selected
9) The difference between the average number of QA questions are QA and revenue generated for per LR
question developed by all content developer together selected question is Rs. 50.ifAverage revenue
and average number of LR questions developed by all generated by selected question is Rs. 72.50, then find
content developer together is approximately how the revenue generated per QA selected question?
much percentage of 20% of total number of questions a) Rs. 80
selected in Mock test? b) Rs. 95
a) 6% c) Rs. 85
b) 9% d) Rs. 105
c) 7% e) None of these
d) 8%

Directions (11 – 15): Read the following Line graph carefully and answer the following questions based on
given information.
The graph given below shows the number of matches played by four players of Indian Cricket Team in year
2019.
Total matches played by a player = T-20 + One day + Test

Page 113 of 199

Subscribe The Xpress Video Course & Mock Test Package for Bank & Insurance Exams
If there are any suggestions/ errors in our PDFs Feel Free to contact us via this email: admin@exampundit.in
Ultra Practice Bundle PDF
IBPS PO Prelims - Quantitative Aptitude
200
180
180
156
160
140
140 132
120
114
120
96
100 86
72 76
80 68

60 48

40

20

0
Ravindra Jadeja Ajinkya Rahane Vijay Shankar Ambati Rayadu

T-20 One Day Test

11) If total number of matches played by of Matches played by all the players against
RavindraJadeja against South Africa is 41, and Rest Australia?
matches played by him is against West Indies and a) 249
England in ratio of 4:3 respectively. Find the matches b) 259
played by RavindraJadeja against England? c) 239
a) 121 d) 269
b)148 e) None of these
c) 111 13) If the difference between the average of T-20
d) 131 matches played by all players together and average of
e) None of these one day matches together by all players is how much
12) If RavindraJadeja Played 36.36% of T-20 % less or more than the total number of test matches
matches, AjinkyaRahane played 35% of Test played by all players together?
Matches, Vijay Shankar played 10/19 of T-20 a) 10% less
matches and Ambati Rayadu played 55% of one day b) 90% less
matches against Australia, then find the total number c) 70% less
Page 114 of 199

Subscribe The Xpress Video Course & Mock Test Package for Bank & Insurance Exams
If there are any suggestions/ errors in our PDFs Feel Free to contact us via this email: admin@exampundit.in
Ultra Practice Bundle PDF
IBPS PO Prelims - Quantitative Aptitude
d) 30% less 15) Find the respective ratio of total number of
e) None of these Matches played by Vijay Shankar and
14) Total one day and test matches together played by AmbatiRayadu together to total matches played by
AjinkyaRahane is how much % of total T-20 and one all players together?
day matches played by Ambati Rayadu? a) 2:1
a) 56.75% b) 3:8
b) 56.25% c) 4:9
c) 58.25% d) 7:12
d) 58.75% e) None of these
e) None of these

Directions (16 – 20): Study the following information carefully and answer the questions given below.
The line graph given below shows the export of grains (in tonnes) from India and the import of grains (in tonnes) to
USA in six different years.
7000
6400 6400 6500
6000

5200
5000
4500
4200 3800 4200
4000

3200 3600 3200


3000
2400
2000

1000

0
2013 2014 2015 2016 2017 2018

Export from India Import to USA

Page 115 of 199

Subscribe The Xpress Video Course & Mock Test Package for Bank & Insurance Exams
If there are any suggestions/ errors in our PDFs Feel Free to contact us via this email: admin@exampundit.in
Ultra Practice Bundle PDF
IBPS PO Prelims - Quantitative Aptitude
16) If it is mentioned that in the year 2015, the ratio tones) of imports of USA from other countries in
of imports to exports for India is 7:6 and in the year these three years ?
2017, exports of USA are 5 more than imports in a) 4216

the same year, then the imports of India in 2015 is b) 4216


what percent of the export of USA in 2017?
c) 4216
a) 65%
b) 70% d) 4216
c) 75% e) None of these
d) 35% 19) The average quantity of grain imported to USA in
e) None of these the year 2013, 2015, 2017 and 2018 together is what
17) It is mentioned that the quantity of grain exported percent of the average exports of grain from India in
from India in the year 2012 is the same as the the years 2014, 2016 and 2017 together?
quantity of grain imported to USA in 2012. If it is a) 100%
mentioned that the ratio of grain exported from India b) 70%
in the period of 2012 -2015 to the grain imported to c) 33 %
USA in the same period is 38:41, then what is the
d) 120%
value of grain exported from India in 2012?
e) None of these
a) 4800 tons
20) It is mentioned that the cost incurred in importing
b) 6000 tons
1 tonne of grain by India in the year 2013 was Rs 62
c) 5700 tons
and the sum of the total cost incurred in exporting
d) 5300 tons
grain from India and the cost incurred in importing
e) None of these
grain to India in 2013 was Rs 616000. What is the
18) If 14 12 and 25% of the exports of grain ratio of the cost incurred per tons in importing grain
from India in the years 2016, 2017 and 2018 by India to the cost incurred per ton exporting grain
respectively were to USA, then what is the average (in in 2013 from India if it is given that ratio of quantity

Page 116 of 199

Subscribe The Xpress Video Course & Mock Test Package for Bank & Insurance Exams
If there are any suggestions/ errors in our PDFs Feel Free to contact us via this email: admin@exampundit.in
Ultra Practice Bundle PDF
IBPS PO Prelims - Quantitative Aptitude
of grain exported to quantity of grain imported is 8:7 c) 31: 41
in the year? d) 31: 21
a) 31: 11 e) None of these
b) 21: 31

Directions (21 – 25): Read the information carefully and answer the questions based on it.
The following Line graph shows the number of Quant Question and Reasoning question prepared by Five Persons in
one week and price per quant question and Reasoning question of each person also given.
Total question = Quant question + Reasoning Question
200
180
180 165
160 150
136
140 125
140 120 118
115
120
95
100

80

60

40 25 40
30 15 18
20 25
12.5 10 10
0 5
A B C D E

Quant Question Reasoning Question


Price of per Quant Question Price of per reasoning Question

21) The total revenue generated by Person A is c) 22% Less


approximately how much percentage more or less d) 27% More
than total revenue generated by Person E? e) 29% More
a) 32% Less
b) 33% More
Page 117 of 199

Subscribe The Xpress Video Course & Mock Test Package for Bank & Insurance Exams
If there are any suggestions/ errors in our PDFs Feel Free to contact us via this email: admin@exampundit.in
Ultra Practice Bundle PDF
IBPS PO Prelims - Quantitative Aptitude
22) Average price per question of person B is but Quant price and reasoning price per question is
approximately what percentage of average price per interchanged then find the approximate percentage
question by person D? change in revenue generatedby Person D as
a) 154% compared to previous week?
b) 132% a) 9.6%
c) 127% b) 8.6%
d) 137% c) 9.1%
e) 142% d) 8.1%
23) Find the difference between total revenue e) None of these
generated by B and total revenue generated by Quant 25) Find the difference between the average number
question by person C? of quant questions and average number of reasoning
a) Rs. 3340 questions by all five persons together?
b) Rs. 3350 a) 26.6
c) Rs. 3360 b) 27.6
d) Rs. 3240 c) 25.6
e) None of these d) 23.6
24) If in next week, person D prepared same number e) 21.6
of Quant and reasoning question as previous week

Directions (26-30): Read the following information carefully and answer the following questions
The line graph given below shows the Total number of students studying from engineering and medical colleges in
various years.

Page 118 of 199

Subscribe The Xpress Video Course & Mock Test Package for Bank & Insurance Exams
If there are any suggestions/ errors in our PDFs Feel Free to contact us via this email: admin@exampundit.in
Ultra Practice Bundle PDF
IBPS PO Prelims - Quantitative Aptitude
40000
37500
35000
32000
30000 30000
28000 27800
25000 26000
25000
24000
22000
20000

15000
12800
10000

5000

0
2016 2017 2018 2019 2020

Number of engineering students Number of medical students

26) What is the average Number of students studying d) 14200


engineering colleges over all the years together? e) 14300
a) 24800 28) In which year, total number ofmedical and
b) 25800 engineering students together is maximum?
c) 26800 a) 2020
d) 28500 b) 2017
e) 28400 c) 2019
27) What is the difference between the no. ofstudents d) 2016
studying medical college in 2016 & 2018 together to e) 2018
no. of students studying engineering college in 29) Find the ratio of difference between engineering
2017&2020 together? & medical students in 2018 to the difference between
a) 14020 engineering & medical students in 2016?
b) 12040 a) 5:14
c) 12400 b) 5:11
Page 119 of 199

Subscribe The Xpress Video Course & Mock Test Package for Bank & Insurance Exams
If there are any suggestions/ errors in our PDFs Feel Free to contact us via this email: admin@exampundit.in
Ultra Practice Bundle PDF
IBPS PO Prelims - Quantitative Aptitude
c) 5:9 total students studying medical and engineering in
d) 10: 23 2021?
e) 10:27 a) 48000
30) If the number of students studying engineeringin b) 72000
2021 is 20% more than no. of students c) 75000
studyingmedicalin 2019 and also number of students d) 45000
studying medical in 2021 is 25% less than no. of e)None of these
students studying engineering in 2016, then find the

Directions (31-35): Study the following information carefully and answer the questions given below.
The given line graph shows the number of employees in four different department in five different companies – A, B,
C, D and E.
3000
2750
2500
2500 2250 2250
2250 2250 2250
2000
2000 2000 2000 2000 2000
1750 1750 1750
1750
1500 1500 1500
1500 1500

1000

500

0
A B C D E

HR IT Sales Finance

31) What is the average number of HR employees in 32) What is the difference between the total number
all the companies together? of employees in B and D in all the departments
a) 2020. b) 2040. c) 2050. d) 2060. e) 2030 together?
Page 120 of 199

Subscribe The Xpress Video Course & Mock Test Package for Bank & Insurance Exams
If there are any suggestions/ errors in our PDFs Feel Free to contact us via this email: admin@exampundit.in
Ultra Practice Bundle PDF
IBPS PO Prelims - Quantitative Aptitude
a) 1600. b) 1800. c) 1500. d) 2000. e) 1900 the total number of HR employees in all the
33) Ratio of the number of male to female employees companies together?
in Finance department in A is 3:2 and the ratio of the a) 80%. b) 85% c) 88%. d) 83%. e) 78%
number of urban to rural male employees in Finance 35) In F, the number of HR employees is 20% more
department in A is 5:4. The total number of female than the number of HR employees in all the
employees and urban male employees in Finance companies together and number of IT employees in F
department in A is what percent of the total number is 25% more than the number of IT employees in all
of employees in A in all the departments together? the companies together. What is the total number of
a) 18%. b) 20%. c) 22%. d) 24%. e) 26% employees in F in HR and IT departments together?
34) The total number of Sales employees in all the a) 23550. b) 22800 c) 23450 d) 22750. e) 22950
companies together is approximately what percent of

Directions (36-40): Study the following information carefully and answer the questions given below.
The following line graph shows the number of peoples read various newspapers from different states.
900
840 850
800

700 660
700
600 630
600 560
450 550 510
500 Hindu
400
440 Indian Express
400 430
340 Others
300
250
200

100

0
Gujarat MP HP UP AP

Page 121 of 199

Subscribe The Xpress Video Course & Mock Test Package for Bank & Insurance Exams
If there are any suggestions/ errors in our PDFs Feel Free to contact us via this email: admin@exampundit.in
Ultra Practice Bundle PDF
IBPS PO Prelims - Quantitative Aptitude
36) In which state total number of peoples read a) 478. b) 468. c) 488. d) 498. e) 458
newspaper is minimum? 39) What is the difference between total persons read
a) HP Hindu newspaper from all the states together and
b) Gujarat total persons read other newspapers from all the
c) AP states together?
d) Both a and b a) 130. b) 170. c) 150. d) 140. e) 160
e) Both a and c 40) Find the ratio of number of peoples read Indian
37) Find the difference between number of peoples express from MP, HP, AP to the number of peoples
read Hindu & Indian Express in UP is what read Hindu from the same states together?
percentage of difference between no. of peoples read a) 151:149
the same in Gujarat? b) 151:153
a) 111.11%. b) 125%. c) 110%. d) 120%. e) 133.33% c) 151: 147
38) Find the average number of peoples read Indian d) 149:153
express newspaper? e) 147:153
Directions (41-45): Study the following information carefully and answer the questions given below.
The given line graph shows the number of students like three different subjects from five different classes.
70

60 58
54 54
50 52 46
46 48
48
40 36
34 36
36
30 32 32
28
20

10

0
A B C D E

Biology Physcics Chemistry

Page 122 of 199

Subscribe The Xpress Video Course & Mock Test Package for Bank & Insurance Exams
If there are any suggestions/ errors in our PDFs Feel Free to contact us via this email: admin@exampundit.in
Ultra Practice Bundle PDF
IBPS PO Prelims - Quantitative Aptitude
41) The total number of students like all the three number of students like Physics in all the classes
subjects in A is what percent more than that of the together?
total number of students like all the three subjects in a) 85.8% b) 84.5%. c) 83.6%. d) 82.3%. e) 80.9%
D? 44) If the ratio of the number of boys to girls in B and
a) 17.54%. b) 19.48%. c) 12.67%. d) 20.05% E is 45:28 and 19:10 respectively, then what is the
e) 14.98% difference between the number of boys in B and E?
42) What is the difference between the number of a) 12. b) 13. c) 14. d) 10. e) 16
students like Biology and Chemistry in B, C and D 45) What is the ratio of the number of students like
together? Chemistry in A, B and C together to the number of
a) 30. b) 40. c) 50. d) 35. e) 45 students like Biology and physics together in A, B
43) The total number of students like biology in all andC together?
the classes together is what percent of the total a) 49:151. b) 43:156. c) 47:155. d) 45:151
e) None of these
Directions (46-50): Read the information carefully and answer the following questions
The following line graph shows the number of WLS in various states in different years
300

250 100

200
75
150 34 80 30
39 68 40
100 60
52 80
65
50 51
40
28
0
Andhra Pradesh Rajasthan West Bengal Kerala Gujarat

2000 2010 2020

Page 123 of 199

Subscribe The Xpress Video Course & Mock Test Package for Bank & Insurance Exams
If there are any suggestions/ errors in our PDFs Feel Free to contact us via this email: admin@exampundit.in
Ultra Practice Bundle PDF
IBPS PO Prelims - Quantitative Aptitude
46) Find the sum of the number of WLS in Rajasthan 48) Find the ratio of average no. of WLS in 2020, to
in 2000 and number of WLS in Gujarat in 2010 the average no. of total WLS in West Bengal?
together is what percentage more/ less than the sum a) 255:269
of number of WLS in Rajasthan in 2010 and Gujarat b) 278:269
in 2000? c) 278:271
a) 27.27% less d) 278:255
b) 28.57%more e) 269:255
c) 42.86% less 49) In which state percentage increase of WLS from
d) 36.36% more previous year is always positive?
e) none of these a) Andhra pradesh
47) In the year 2021, Government planned to develop b) Gujarat
IT park by destroying 20% of total WLS in c) Rajasthan
Rajasthan, 40% of total WLS in Kerala and 30% of d) Gujarat&West Bengal
total WLS in Gujarat, then find the total no. of WLS e) Kerala & Rajasthan
in the year 2021? 50) Find the average number of WLS in the year
a) 664 2010?
b) 644 a) 65
c) 624 b) 60
d) 646 c) 75
e) 642 d) 70
e) 80

Line DI - Answer and Detailed Explanation


Answers:
Directions (1 – 5):
Company Total AC Split AC Window AC

Voltas 7296 3963 3333

Page 124 of 199

Subscribe The Xpress Video Course & Mock Test Package for Bank & Insurance Exams
If there are any suggestions/ errors in our PDFs Feel Free to contact us via this email: admin@exampundit.in
Ultra Practice Bundle PDF
IBPS PO Prelims - Quantitative Aptitude
Blue star 4890 2955 1935

LG 2784 1479 1305

Daikin 1917 870 1047

Haier 8430 5430 3000

Total 25317 14697 10620

1) Answer: A Total number of window AC sold by Haier = 3000


Total number of Split AC of Voltas = 3963 Required % change = (3000 – 2982)/3000 x 100 =
Voltas AC of type B = 3963 – 1713 = 2250 0.6%less
Total number of Window AC of Voltas = 3333 4) Answer: A
Voltas AC of type X = 3333 – 1683 = 1650 Revenue generated by LG from Split AC = 4/5 x 15000
Required % change = (2250 – 1650)/1650 x 100 = x (1479) = 177.48 lakhs
36.36% more Revenue generated by Blue Star from window AC = 3/4
2) Answer: C x 8000 x (1935) = 116.10 lakhs
Average number of AC sold by (LG + Daikin + Haier) = Total revenue = 177.48 + 116.10 = 293.58 lakhs
(2784 + 1917 + 8430)/3 = 4377 5) Answer: E
Average number of Window AC sold by (LG + Daikin + Total number of Split AC manufactured = 4/3 x 14697 =
Haier) = (1305 + 1047 + 3000)/3 = 1784 19596
Required difference = 4377 – 1784 = 2593 Total number of Window AC manufactured = 10/9 x
3) Answer: D 10620 = 11800
Total number of Window AC sold by (Daikin + Blue Total AC manufactured = 31396
star) = 1935 + 1047 = 2982
Directions (6 – 10):
For content developer A
QA question developed = 900
LR questions developed = 1000
Total question developed = 1900
Page 125 of 199

Subscribe The Xpress Video Course & Mock Test Package for Bank & Insurance Exams
If there are any suggestions/ errors in our PDFs Feel Free to contact us via this email: admin@exampundit.in
Ultra Practice Bundle PDF
IBPS PO Prelims - Quantitative Aptitude
Number of question selected in Mock test = 60% of 1900 = 1140
Similarly we can calculate the data for content developers
Content QA questions LR questions Total questions Questions
Developer developed developed developed selected in
Mock Test
A 900 1000 1900 1140
B 1250 1150 2400 1080
C 1000 1700 2700 1080
D 1500 1300 2800 1820
E 1600 1640 3240 1620
6) Answer: B 8) Answer: A
For content developer E, number of question selected = Total questions developed = 1900 + 2400 + 2700 + 2800
1620 + 3240 = 13040
Let the number of LR questions selected = 5a Total number of questions selected = 1140 + 1080 +
Number of QA questions selected = 80% of 5a = 4a 1080 + 1820 + 1620 = 6740
So, (5a + 4a) = 1620 Required % change = [(13040 – 6740)/13040] x 100 =
9a = 1620 48.3% (approx.)
a = 1620/9 = 180 9) Answer: D
Number of QA questions selected = 4a = 4 x 180 = 720 Average number of QA questions developed = (900 +
Required % = (720/1600) x 100 = 45% 1250 + 1000 + 1500 + 1600)/5 = 1250
7) Answer: C Average number of LR questions developed = (1000 +
For content developer D, total number of questions 1150 + 1700 + 1300 + 1640)/5 = 1358
selected = 1820 Difference between the required average = 1358 – 1250
Number of QA questions selected = 40% of 1820 = 728 = 108
Number of LR questions selected = 1820 – 728 = 1092 20% of total number of questions selected = 0.20 x (1140
Total revenue generated = 728 x 250 + 1092 x 325 = + 1080 + 1080 + 1820 + 1620) = 1348
5.369 lakhs Required % = (108/1348) x 100 = 8% (approx.)
Page 126 of 199

Subscribe The Xpress Video Course & Mock Test Package for Bank & Insurance Exams
If there are any suggestions/ errors in our PDFs Feel Free to contact us via this email: admin@exampundit.in
Ultra Practice Bundle PDF
IBPS PO Prelims - Quantitative Aptitude
10) Answer: E According to question,
Number of QA questions selected = 45% of 1080 = 486 486 x a + 594 x 50 = 1080 x 72.5
Number of LR question selected = 1080 – 486 = 594 486a = 78300 – 29700
Let the revenue generated per selected question from QA a = 48600/486 = Rs. 100
= Rs. a
Directions (11 – 15):
Player T-20 One Day Test Total
RavindraJadeja 132 72 96 300

AjinkyaRahane 156 68 120 344


Vijay Shankar 114 86 48 248
Ambati Rayadu 140 180 76 396
Total 542 406 340 1288
11) Answer: C Total matches played against Australia = (48 + 42 + 60 +
Total matches played by RavindraJadeja = 300 99) = 249
Matches played against West Indies and England = 300 – 13) Answer: B
41 = 259 Average of T-20 matches = 542/4 = 135.5
Matches played against England = 3/7 x 259 = 111 Average of One day matches = 406/4 = 101.5
12) Answer: A Difference of averages = 135.5 – 101.5 = 34
T-20 played by RavindraJadeja against Australia = 4/11 Required % change = (340 – 34)/340 x 100 = 90% less
x 132 = 48 14) Answer: D
Test matches by AjinkyaRahane against Australia = 35% Total required matches of AjinkyaRahane = 68 + 120 =
of 120= 42 188
T-20 matches played by Vijay Shankar against Australia Total required matches by AmbatiRayadu = 140 + 180 =
= 10/19 x 114 = 60 320
One day matches played by Ambati Rayadu against Required % = (188/320) x 100 = 58.75%
Australia = 55% of 180 = 99 15) Answer: E

Page 127 of 199

Subscribe The Xpress Video Course & Mock Test Package for Bank & Insurance Exams
If there are any suggestions/ errors in our PDFs Feel Free to contact us via this email: admin@exampundit.in
Ultra Practice Bundle PDF
IBPS PO Prelims - Quantitative Aptitude
Total required matches played by Vijay Shankar and =
Ambati Rayadutogether = 248 + 396 = 644
= 4216
Total matches played by all players together = 1288
19) Answer: A
Required ratio = 644: 1288 = 1:2
The average imports to USA in 2013, 2015, 2017 and
16) Answer: B
2018 together
Imports of India in 2015 = 2400 x = 2800 tons
=
Exports of USA in 2017 = 3800+ = 3800 +200
=
= 4000 tons
= 4100
Required percentage =
The average exports from India in 2014, 2016 and 2017
17) Answer: C together
Let N = Grain exported from India in 2012 = Grain
=
imported to USA in 2012
=
According to question,
= 4100
Required percentage =
20) Answer: D
545300 +41N = 562400 +38N Quantity of grain imported in 2013 = 6400 x 5600
tonnes
3N = 17100 Cost incurred in importing grain = 5600 x 62 = Rs
N = 5700 tons 347200
18) Answer: D Cost incurred in exporting grain = 616000 – 347200 =
( ) ( ) ( ) 268800
Required Average =
Cost per ton incurred to exporting grain =
= (5900 + 3350 + 3400)
Required Ratio = 62 : 42 = 31: 21

Directions (21 – 25):

Page 128 of 199

Subscribe The Xpress Video Course & Mock Test Package for Bank & Insurance Exams
If there are any suggestions/ errors in our PDFs Feel Free to contact us via this email: admin@exampundit.in
Ultra Practice Bundle PDF
IBPS PO Prelims - Quantitative Aptitude
Person Total questions Total Revenue
A 115 + 180 = 295 115 x 30 + 180 x 12.5 = 5700
B 140 + 150 = 290 140 x 10 + 150 x 25 = 5150
C 120 + 165 = 285 120 x 15 + 165 x 5 = 2625
D 125 + 95 = 220 125 x 10 + 95 x 18 = 2960
E 118 + 136 = 254 118 x 25 + 136 x 40 = 8390
Total 1344 24825
21) Answer: A Required % change = [(3200 – 2960)/2960] x 100 =
Total revenue generated by A = 5700 8.1%
Total revenue generated by E = 8390 25) Answer: E
Required % change = [(8390 – 5700) / 8390] x 100 = Average number of quant questions = (115 + 140 + 120
32% Less + 125 + 118)/5 = 123.6
22) Answer: B Average number of reasoning questions = (180 + 150 +
Average price per question by B = 5150/290 = 17.75 per 165 + 95 + 136)/5 = 145.2
question Required difference = 21.6
Average price per question by person D = 2960/220 = 26) Answer: B
13.45 Average of engineering students=
Required % = (17.75 / 13.45) x 100 = 132% = 25800
23) Answer: B 27) Answer: D
Total revenue by B = 5150 No. of medical students in 2016&2018 = 12800+26000 =
Revenue generated by C from Quant questions = 120 x 38800
15 = 1800 No. of engineering students in 2017&2020 = 28000
Required difference = 5150 – 1800 = 3350 rupees +25000 = 53000
24) Answer: D Required difference = 53000 – 38800 = 14200
Previous week revenue generated by person D = 2960 28) Answer: C
Next week Revenue generated by after interchanging = Total students in 2016 = 24000 +12800 = 36800
125 x 18 + 95 x 10 = 3200 Total students in 2017 = 28000+32000 = 60000
Page 129 of 199

Subscribe The Xpress Video Course & Mock Test Package for Bank & Insurance Exams
If there are any suggestions/ errors in our PDFs Feel Free to contact us via this email: admin@exampundit.in
Ultra Practice Bundle PDF
IBPS PO Prelims - Quantitative Aptitude
Total students in 2018 = 22000+26000 = 48000 = 2000
Total students in 2019 = 30000+37500 = 67500 33) Answer: B
Total students in 2020 = 25000+27800 = 52800 Number of male employeein Finance department in A =
In 2019, number of students is maximum i.e 67500 3/5 * 2250 = 1350
29) Answer: A Number of female employeesin Finance department in A
Diff. between no of engineering & medical students in = 2/5 * 2250 = 900
the year 2018 Number of urban male employeesin Finance department
= 26000 – 22000 in A = 5/9 * 1350 = 750
= 4000 Total number of employees in A = (2250 + 1750 + 2000
Diff. between no of engineering & medical students in + 2250) = 8250
the year 2016 Required percentage = [(900 + 750)/8250] * 100 = 20%
= 24000 – 12800 34) Answer: B
= 11200 Required percentage = (2000 + 1500 + 1500 + 2000 +
Required ratio is 4000 :11200 i.e 5:14 1750)/(2250 + 1750 + 2500 + 2250 + 1500) * 100
30) Answer: E = (8750/10250)*100
Number of engineering students in 2021 = 120% of = 85%
37500 = 45000 35) Answer: A
Number of medical students in 2021 = 75% of 24000 = Number of HR employees in F = 120/100 * (2250 +
18000 1750 + 2500 + 2250 + 1500) = 12300
Total students in 2021 = 45000+18000 = 63000 Number of IT employees in F = 125/100 * (1750 + 1500
31) Answer: C + 2000 + 1750 + 2000) = 11250
Required average = (2250 + 1750 + 2500 + 2250 + Required total = 12300 + 11250 = 23550
1500)/5 36) Answer: D
= 2050 No. of peoples read newspaper in Gujarat =
32) Answer: D 450+250+700= 1400
Required difference = (2250 + 1750 + 2000 + 2750) – No. of peoples read newspaper in MP = 430+600+840 =
(1750 + 1500 + 1500 + 2000) 1870

Page 130 of 199

Subscribe The Xpress Video Course & Mock Test Package for Bank & Insurance Exams
If there are any suggestions/ errors in our PDFs Feel Free to contact us via this email: admin@exampundit.in
Ultra Practice Bundle PDF
IBPS PO Prelims - Quantitative Aptitude
No. of peoples read newspaper in HP = 660+400+340 = = 600+400+510
1400 =1510
No. of peoples read newspaper in UP = 850+630+550 = Total persons read Hindu newspaper from MP, HP, AP
2030 = 430+660+440
No. of peoples read newspaper in AP = 440+510+560 = = 1530
1510 Required ratio 1510: 1530
In Gujarat as well as HP, the no. of newspapers readers’ i.e., 151:153
is minimum. 41) Answer: A
37) Answer: C Required percentage = {[(34 + 46 + 54) – (32 + 36 +
Required percentage = ×100 = 110% 46)]/(32 + 36 + 46)} * 100

38) Answer: A = 17.54%

Total number of peoples read Indian express 42) Answer: B

= 250+600+400+630+510 = 2390 Number of students like biology in B, C and D = 52 + 48


+ 36 = 136
Required average = =478
Number of students like Chemistry in B, C and D = 36 +
39) Answer: E
28 + 32 = 96
Total persons read Hindu newspaper from all the states
Difference = 136 – 96 = 40
together
43) Answer: D
= 450+430+660+850+440
Required percentage = [(46 + 52 + 48 + 36 + 32)/(54 +
=2830
58 + 54 + 46 + 48)] * 100
Total persons read other newspaper from all the states
= 82.3%
together
44) Answer: C
= 700+840+340+550+560
Number of boys in B = 45/73 * (52 + 58 + 36) = 90
= 2990
Number of boys in E = 19/29 * (32+48 + 36) = 76
Required difference = 2990 – 2830 =160
Difference = 90 – 76 = 14
40) Answer: B
45) Answer: E
Total persons readIndian express newspaper from MP,
HP, AP
Page 131 of 199

Subscribe The Xpress Video Course & Mock Test Package for Bank & Insurance Exams
If there are any suggestions/ errors in our PDFs Feel Free to contact us via this email: admin@exampundit.in
Ultra Practice Bundle PDF
IBPS PO Prelims - Quantitative Aptitude
Required ratio = (34 + 36 + 28):(46 + 52 + 48 + 54 + 58 = 140+147+105+ (28+52+39) + (51+68+34)
+ 54) = 664
= 98:312 48) Answer: D
= 49:156 Required ratio is ( ):( )
46) Answer: C
:
( ) ( )
Required percentage = ( )
×100 = 42.86% less
278:255
47) Answer: A 49) Answer: E
Total no. of WLS in Rajasthan in the year 2021 If percentage increase be positive, then the no. of WLS
= 80% of (40+60+75) = 140 in particular state will be more when compared to the
Total no. of WLS in Kerala in the year 2021 previous decade.
= 60% of (65+80+100) Therefore,
= 147 In Kerala and Rajasthan, the number of WLS is more
Total no. of WLS in Gujarat in the year 2021 when compared to prev.year
= 70% of (80+40+30) 50) Answer: B
= 105 Total number of WLS in the year 2010 =
Total number of WLS in 2021 from all the states 52+60+68+80+40 = 300
together Required average = 300/5 =60

Data Interpretation Practice Questions PDF Download


Download Quantitative Aptitude Practice Questions PDF

Pie DI
Directions (1-5): Study the following information carefully and answer the questions given below.
The given pie chart shows the number of mobile and laptops manufactured in 6 different companies.

Page 132 of 199

Subscribe The Xpress Video Course & Mock Test Package for Bank & Insurance Exams
If there are any suggestions/ errors in our PDFs Feel Free to contact us via this email: admin@exampundit.in
Ultra Practice Bundle PDF
IBPS PO Prelims - Quantitative Aptitude

Total number of mobile and laptops = 1200

12%
22%
A
B
16%
C
D
14%
E
15% F

21%

1) The ratio of the number of mobile and laptop a) 28


manufactured in A and E is 6:5 and 5:3 respectively. b) 26
The total number of laptop manufactured in A and E c) 24
together is approximately what percent of the total d) 22
number of mobile manufactured in A and E e) None of these
together? 3) What is the sum of the total number of laptop
a) 73% manufactured in A and B together?
b) 78% Statement I: The ratio of total number of Laptop
c) 80% manufactured in A to the total number of mobile
d) 68% manufactured in B is 4:2.
e) 82% Statement II: Total number of mobile manufactured in
2) What is the difference between the number laptop A is 20% of the total number of manufactured in D.
and mobile manufactured in B, if the number of Statement III: The number of laptop manufactured in B
laptop manufactured in B is 40% of the total number is 20% of the total number mobile manufactured in D.
of products manufactured in D? a) Only I and II
Page 133 of 199

Subscribe The Xpress Video Course & Mock Test Package for Bank & Insurance Exams
If there are any suggestions/ errors in our PDFs Feel Free to contact us via this email: admin@exampundit.in
Ultra Practice Bundle PDF
IBPS PO Prelims - Quantitative Aptitude
b) Only II and III Quantity I: If the ratio of the laptop and mobile
c) Either I or II and III are sufficient manufactured in C is 4:3 and the number of laptop
d) All I, II and III necessary to the answer the question manufactured in E is 25% more than that of number of
e) The question can’t be answered even with all I, II and mobile manufactured in C. What is the sum of the
III number of laptop manufactured in C and the number of
4) If the number of mobile manufactured in F is 25%, mobile manufactured in E?
what is the difference between the number of laptop Quantity II: What is the average number of products
manufactured in F and the total number of products manufactured in F, A, D and B together?
manufactured in C? a) Quantity I > Quantity II
a) 134 b) Quantity I ≥ Quantity II
b) 124 c) Quantity II > Quantity I
c) 154 d) Quantity II ≥ Quantity I
d) 144 e) Quantity I = Quantity II or Relation cannot be
e) None of these established
5)
Directions (6-10): Study the following information carefully and answer the questions given below.
The given pie chart shows the percentage of the ages of five different persons.

Total ages = 120

20% A
30% B
C
15%
D
10%
E
25%

Page 134 of 199

Subscribe The Xpress Video Course & Mock Test Package for Bank & Insurance Exams
If there are any suggestions/ errors in our PDFs Feel Free to contact us via this email: admin@exampundit.in
Ultra Practice Bundle PDF
IBPS PO Prelims - Quantitative Aptitude
6) The difference between the ages of C and D is F and H after 6 years is 25 years, then what is H’s age
equal to the F’s age 7 years ago. If the ratio of the 10 years ago?
ages of F to G after 5 years is 5:8 and H’s present age a) 20 years
is half of G’s age one year ago.What is the difference b) 12 years
between the average ages of H, D and E together and c) 16 years
the average ages of A and G together? d) 24 years
a) 10.5 years e) None of these
b) 12.5 years 9) Average ages of A, C and E is equal to the sum of
c) 15.5 years the ages of R and S. If the product of the ages of R
d) 17.5 years and D is 156 years, then what is the present age of S?
e) None of these a) 12 years. b) 15 years. c) 17 years
7) The difference between the ages of D and A is d) 19 years. e) 21 years
equal to present age of F. Ratio of the ages of F and G 10) The present age of F is 50% more than the
after 8 years is 2:3. Find G’s age 9 years ago? present age of G. If the average age of G, D and H is
a) 12 years 24 years and ratio of the ages of H to C is 7:5, then
b) 13 years what is the average age of F, G and H?
c) 15 years a) 27 years
d) 18 years b) 29 years
e) None of these c) 31 years
8)If the average ages of B, F and G is 18 years and the d) 33 years
E’s age is 50% more than G, then the average ages of e) None of these

Directions (11-15): Read the following instructions carefully and answer the following questions
The following pie-chart shows the salary distribution of Mr.X on various expenses.
Total salary of Mr.X = Rs.60000

Page 135 of 199

Subscribe The Xpress Video Course & Mock Test Package for Bank & Insurance Exams
If there are any suggestions/ errors in our PDFs Feel Free to contact us via this email: admin@exampundit.in
Ultra Practice Bundle PDF
IBPS PO Prelims - Quantitative Aptitude

Percentage distribution of various expenses


7.5%

5%

20%

30%
25%

12.5%
Rent Medicine Cloth Food Entertainment Travel

11) What is the ratio between money spent on food e) Rs.25400


and travel together to money spent on rent and 13) Money spent on travel is what percentage more or
medicine together? less than the money spent on entertainment?
a) 5:7 a) 50% more
b) 5:3 b) 44.44% less
c) 5:4 c) 25% more
d) 5:6 d) 33.33%less
e) 7:5 e) 30% more
12) What is total money spent on cloth and food 14) What is the difference between money spent on
together? travel & entertainment together and food & medicine
a) Rs.25050 together?
b) Rs.25500 a) Rs.25050
c) Rs.25000 b) Rs.25400
d) Rs.24500 c) Rs.25500

Page 136 of 199

Subscribe The Xpress Video Course & Mock Test Package for Bank & Insurance Exams
If there are any suggestions/ errors in our PDFs Feel Free to contact us via this email: admin@exampundit.in
Ultra Practice Bundle PDF
IBPS PO Prelims - Quantitative Aptitude
d) Rs.24050 a) 16.67%
e) Rs.24500 b) 12.5%
15) If the salary of Mr.X is increased by 25% also he c) 10%
increases expenditure on cloth by Rs.500, then find d) 11.66%
the percentage of amount he spend to buy cloth? e) 10.67%

Directions (16-20): Read the following instructions carefully and answer the following questions.
The following pie-chart shows the total peoples recovered from covid 19 in various countries.
Total peoples recovered from covid 19 = 12400

Total peoples recovered from Covid 19

30% 20%

25%

10%

15%
China India UK USA Russia

The following table shows Ratio of female to male


the ratio of male to female recovered from covid 19
recovered from covid 19 in
various countries Country
China 27:35
India 10:21
UK 15:16

Page 137 of 199

Subscribe The Xpress Video Course & Mock Test Package for Bank & Insurance Exams
If there are any suggestions/ errors in our PDFs Feel Free to contact us via this email: admin@exampundit.in
Ultra Practice Bundle PDF
IBPS PO Prelims - Quantitative Aptitude
USA 2:3
Russia 43:50
16) Find the average number of males recovered from e) 75%less
covid 19? 18) Find the ratio of sum of the female from India,
a) 1420.8 UK and Russia together recovered from covid 19 to
b) 1340.8 the male recovered from same countries together?
c) 1440.4 a) 181:263. b) 181:253. c) 253:181. d) 253:161
d) 1440.8 e) None of the above
e) 1404.8 19) If only 40% of total people who are affected by
17) No. of recovered female from USA is what covid 19 from India are recovered, then find the
percent (approx.) more or less than no. of recovered number of peoples not recovered from covid 19 in
males from china? India?
a) 65%less a) 1550. b) 1450. c) 1860. d) 2750. e) None of these
b) 60%more 20) Find the overall difference between male and
c) 70%less females recovered from covid 19?
d) 55%more a) 2004. b) 2006. c) 2002. d) 2010. e) 2008

Directions (21 – 25): Read the following information given below and answer the following questions
The pie chart given below shows the percentage distribution of number of students in class of 2018 – 2020 PGDM
batch in IIT-DELHI coming from different cities: Delhi, Mumbai and Bangalore. Total number of students in a batch
is 1600.

Page 138 of 199

Subscribe The Xpress Video Course & Mock Test Package for Bank & Insurance Exams
If there are any suggestions/ errors in our PDFs Feel Free to contact us via this email: admin@exampundit.in
Ultra Practice Bundle PDF
IBPS PO Prelims - Quantitative Aptitude

Percentage distribution of number of students in IIT-DELHI

15%
40%

45%

Delhi Mumbai Bangalore

The table given below shows the ratio of engineering background and non-engineering background students in IIT-
DELHI coming from three different cities.
City Engineers: Non Engineers
Delhi 5:3

Mumbai 2:7
Bangalore 4:1
21) Find the ratio of engineers from Delhi and non – 22) Find the difference between total engineers and
engineers from Mumbai respectively? non-engineers students in class of 2018 – 2020 PGDM
a) 4:7 batch from IIT-DELHI?
b) 7:5 a) 76
c) 5:7 b) 96
d) 3:7 c) 86
e) None of these d) 56

Page 139 of 199

Subscribe The Xpress Video Course & Mock Test Package for Bank & Insurance Exams
If there are any suggestions/ errors in our PDFs Feel Free to contact us via this email: admin@exampundit.in
Ultra Practice Bundle PDF
IBPS PO Prelims - Quantitative Aptitude
e) None of these a) 20% more
23) Find the average of engineers from Bangalore and b) 75% less
Mumbai together is how much % of non-engineers of c) 60% more
Delhi? d) 80% less
a) 73.33% e) None of these
b) 74.33% 25) Find the value of central angle corresponding to
c) 75.33% the numbers of students in Delhi and Bangalore
d) 76.33% together? (in degrees)
e) 79.33% a) 155o
24) Difference between the engineer’s students from b) 160o
Mumbai and Non- engineer’s students from c) 125o
Bangalore is how much % more or less than the non- d) 198o
engineers students from Mumbai? e) None of these
Directions (26 -30): Study the following information carefully and answer the questions given below.
The given pie chart shows the percentage of the vehicles in five different cities in 2018.

Total number of Vehicles = 4000

22% 20% Chennai


Bangalore
Delhi
15%
18% Mumbai
Hyderabad
25%

The given table shows the ratio of the number of Petrol to Diesel vehicles in different cities.

Page 140 of 199

Subscribe The Xpress Video Course & Mock Test Package for Bank & Insurance Exams
If there are any suggestions/ errors in our PDFs Feel Free to contact us via this email: admin@exampundit.in
Ultra Practice Bundle PDF
IBPS PO Prelims - Quantitative Aptitude
Cities Petrol:Diesel
Chennai 5:3
Bangalore 2:1
Delhi 3:2
Mumbai 5:3
Hyderabad 6:5

26) The number of petrol vehicle in Mumbai is d) Quantity I < Quantity II


approximately what percent of the number of petrol e) Quantity I ≤ Quantity II
vehicle in Delhi? 28) What is the difference between the average
a) 65% number of petrol and Diesel vehicle in all the cities
b) 70% together?
c) 75% a) 168
d) 80% b) 170
e) None of these c) 172
27) In 2019 the total number of vehicles in Bangalore d) 164
and Mumbai is increased by 20% and 25% e) 162
respectively compared to previous years and the ratio 29) Which of the following cities the number of Diesel
of the number of petrol to Diesel vehicle in Bangalore vehicles are equal?
and Mumbai in 2019 is 5:3 and 2:3 respectively. a) Chennai and Hyderabad
Quantity I: Sum of the number of petrol vehicle in b) Delhi and Mumbai
Bangalore and Mumbai together in 2019. c) Bangalore and Chennai
Quantity II: Sum of the number of Diesel vehicle in d) Hyderabad and Delhi
Bangalore and Mumbai together in 2019. e) Mumbai and Bangalore
a) Quantity I > Quantity II 30) In 2019, the total number of vehicles in Chennai is
b) Quantity I ≥ Quantity II increased by 25% compared previous year. If the
c) Quantity I = Quantity II number of petrol vehicle in Chennai in 2019 is equal

Page 141 of 199

Subscribe The Xpress Video Course & Mock Test Package for Bank & Insurance Exams
If there are any suggestions/ errors in our PDFs Feel Free to contact us via this email: admin@exampundit.in
Ultra Practice Bundle PDF
IBPS PO Prelims - Quantitative Aptitude
to the number of petrol vehicle in Hyderabad in 2018, b) 520
then find the number of Diesel vehicle in Chennai in c) 540
2019? d) 560
a) 500 e) None of these

Directions (31 – 35): Read the following information carefully and answer the following questions based on it.
The following Pie Chart shows the % distribution of students those who are writing Xavier’s Aptitude Test (XAT)
2020 from five different cities out of total students those are writing XAT – 2020.
Total number of students writing XAT exam in 2020 = 13500

Percentage Distribution of students From five


different cities

14%
28%

17%

15%
26%

A B C D E

The table given below shows the ratio of boys and girls writing XAT exam in the given cities.
City Boys: Girls
A 3:4
B 4:5
C 7:6

Page 142 of 199

Subscribe The Xpress Video Course & Mock Test Package for Bank & Insurance Exams
If there are any suggestions/ errors in our PDFs Feel Free to contact us via this email: admin@exampundit.in
Ultra Practice Bundle PDF
IBPS PO Prelims - Quantitative Aptitude
D 9:8
E 5:4

31) Find the difference between the total number of number of total students writing XAT exam in city A
boys writing XAT exam in all cities together to total and E together?
number of girls writing XAT exam in all cities a) 75.2%
together? b) 76.2%
a) 150 c) 77.2%
b) 75 d) 78.2%
c) 160 e) 79.2%
d) 80 34) Total number of boys writing XAT exam in City
e) None of these B and E together is approximately how much
32) The number of boys writing XAT exam in city F percentage more/ less than number of girls writing
is 16.66% more than number of girls writing XAT XAT exam in city C and D together?
exam in city A and the ratio of number of girls a) 29.8% more
writing XAT exam in city F to boys writing XAT b) 25.8% less
exam in city C is 4:15. Then find the total number of c) 26.8% more
students writing XAT exam in city F? d) 27.8% less
a) 3034 e) 30.8% less
b) 2024 35) Find the ratio of number of boys writing XAT
c) 3024 exam in city A to the total number of students writing
d) 2034 XAT exam in city A and city B together?
e) None of these a) 12:47
33) Find the average number of total students writing b) 17:43
XAT exam from City B and City D together is c) 13:43
approximately how much percentage of average d) 11:43
e) None of these

Page 143 of 199

Subscribe The Xpress Video Course & Mock Test Package for Bank & Insurance Exams
If there are any suggestions/ errors in our PDFs Feel Free to contact us via this email: admin@exampundit.in
Ultra Practice Bundle PDF
IBPS PO Prelims - Quantitative Aptitude

Directions (36-40): Study the following information carefully and answer the questions given below.
The following pie-chart shows the percentage distribution of total newspapers sold in different cities
Total newspaper sold = 52500

Total number of newspapers sold

26.66% 21.43%

10% 28.57%

13.33%

Mumbai Chennai Delhi Bangalore Gujarat

Total newspaper sold in a city = sum of the no. of TheHindu& Indian express newspaper sold in the city
The following table shows the number of Indian express newspaper sold in various cities
City Number of Indian express
newspaper sold
Mumbai 5250
Chennai 8000
Delhi 4300
Bangalore 2825
Gujarat 4500
36) Total number of Hindu newspaper sold in which b) Mumbai
city is second lowest? c) Delhi
a) Bangalore d) Chennai

Page 144 of 199

Subscribe The Xpress Video Course & Mock Test Package for Bank & Insurance Exams
If there are any suggestions/ errors in our PDFs Feel Free to contact us via this email: admin@exampundit.in
Ultra Practice Bundle PDF
IBPS PO Prelims - Quantitative Aptitude
e) Gujarat 39) What is the difference between the average
37) Find the ratio of number of Hindu newspaper number of Indian Express newspaper sold in all the
sold in Chennai and Indian express sold in Bangalore cities together and the average number of The Hindu
together to the number of Indian express sold in newspaper sold in all the cities together?
Mumbai and Hindu sold in Gujarattogether? a) 520
a) 393:590 b) 550
b)393:591 c) 440
c)393:592 d) 490
d) 590:393 e) 500
e) 591:393 40) If there are 25%, 20%, 30% of the Hindu
38) Number of Indian express newspaper sold in newspapers sold in Delhi, Bangalore and Gujarat
Gujarat is what percentage of number of The Hindu respectively are returned due to damage issue, then
newspaper sold in Mumbai? what is the number of damaged newspaper returned
a) 70% in these cities together?
b) 66.66% a) 4010
c) 50% b) 4030
d) 83.33% c) 4040
e)None of these d) 4020
e) 4100

Directions (41-45): Study the following information carefully and answer the questions given below.
Percentage of employees in different departments of an organization

Page 145 of 199

Subscribe The Xpress Video Course & Mock Test Package for Bank & Insurance Exams
If there are any suggestions/ errors in our PDFs Feel Free to contact us via this email: admin@exampundit.in
Ultra Practice Bundle PDF
IBPS PO Prelims - Quantitative Aptitude

F, 20%

A, 30%

E, 5%

D, 15%

B, 20%
C, 10%

The table shows the ratio of male to female in different age group

Below 35 years Above 35 years


Male: Female Male : Female
A 9:11 4:5
B 9:1 3:7
C 7:8 8:7
D 7:8 7:9
E 4:1 5:7
F 7:3 1:9

41) If the total number of employees in department A number of employees in department B above 35 years
is 255 and the total number of male employees below to the total number of employees in department B
35 years and females employees above 35 years is 131 below 35 years?
in department B, then find the ratio of the total a) 11:6
Page 146 of 199

Subscribe The Xpress Video Course & Mock Test Package for Bank & Insurance Exams
If there are any suggestions/ errors in our PDFs Feel Free to contact us via this email: admin@exampundit.in
Ultra Practice Bundle PDF
IBPS PO Prelims - Quantitative Aptitude
b) 10:7 c) 636
c) 12:5 d) 684
d) 12:7 e) None of these
e) None of these 44) If the number of female employees below 35 years
42) If the total number of employees in department D and the number of male employees above 35 years in
is 375 and the total number of male employees above department E are 40 and 75 respectively, then find
35 years of age and female employees below 35 years the number of employees in company A?
is 122 in department F, then what is the number of a) 2180
male employees below 35 years in department F? b) 2380
a) 255 c) 2250
b) 265 d) 2280
c) 252 e) None of these
d) 262 45) The total number of employees in department C is
e) None of these 160. Then what is the number of female employees
43) If the total number of employees in department B above 35 years in department D if total employees
below 35 years and above 35 years is 1320 and the below 35 years in department D are 96?
number of male employees below 35 years in the same a) 85
department is 360 then find the number of female b) 91
employees above 35 years in department B? c) 82
a) 644 d) 71
b) 645 e) None of these

Directions (46-50): Study the following information carefully and answer the questions given below.
The given pie chart shows the number of population in five different cities.

Page 147 of 199

Subscribe The Xpress Video Course & Mock Test Package for Bank & Insurance Exams
If there are any suggestions/ errors in our PDFs Feel Free to contact us via this email: admin@exampundit.in
Ultra Practice Bundle PDF
IBPS PO Prelims - Quantitative Aptitude

Total number of population = 15000

16% 18%

A
B
C
24% 20% D
E

22%

The given table shows the percentage of male population in five different cities.
Cities %Male population
A 45%
B 55%
C 65%
D 70%
E 60%
46) What is the difference between the number of 47) Ratio of the number of literate to illiterate
female population in A and D? population in B is 3:2 and the ratio of the number of
a) 400 male literate to male illiterate population in B is 3:2.
b) 405 What is the difference between the number of female
c) 410 illiterate and female literate population in B?
d) 415 a) 240
e) 420 b) 270

Page 148 of 199

Subscribe The Xpress Video Course & Mock Test Package for Bank & Insurance Exams
If there are any suggestions/ errors in our PDFs Feel Free to contact us via this email: admin@exampundit.in
Ultra Practice Bundle PDF
IBPS PO Prelims - Quantitative Aptitude
c) 300 a) 70.5%
d) 210 b) 72.5%
e) 180 c) 74.5%
48) What is the average number of male population in d) 76.5%
all the cities together? e) 68.5%
a) 1798 50) What is the ratio of the number of female
b) 1776 population in B to number of male population A?
c) 1794 a) 5:4
d) 1788 b) 8:7
e) 1782 c) 7:6
49) The number of female population in C and E d) 3:2
together is approximately what percent of the total e) None of these
number of population in B?

Pie DI - Answer and Detailed Explanation


Answers: Required percentage = (72+120)/(144+120) * 100
1) Answer: A =72.72% ~ 73%
Number of mobile manufactured in A = 22/100 * 1200 * 2) Answer: C
6/11 = 144 Number of laptop manufactured in B = 15/100 * 1200 *
Number of laptop manufactured in A = 22/100 * 1200 * 40/100 = 72
5/11 = 120 Total number of products manufactured in B = 14/100 *
Number of mobile manufactured in E = 16/100 * 1200 * 1200 = 168
5/8 = 120 Number of Mobile manufactured in B = 168 – 72 = 96
Number of laptop manufactured in E = 16/100 * 1200 * Required Difference = 96 – 72 = 24
3/8 = 72 3) Answer: A
From statement I,

Page 149 of 199

Subscribe The Xpress Video Course & Mock Test Package for Bank & Insurance Exams
If there are any suggestions/ errors in our PDFs Feel Free to contact us via this email: admin@exampundit.in
Ultra Practice Bundle PDF
IBPS PO Prelims - Quantitative Aptitude
The ratio of total number of Laptop manufactured in A to Total number of manufactured in C=21/100*1200=252
the total number of mobile manufactured in B is 4:2. Required Difference=252 – 108=144
So, Statement I alone is not sufficient to the answer the 5) Answer: A
question. From quantity I,
From Statement II, Number of laptop manufactured in C= 21/100 * 1200 *
Number of products manufactured in D = 15/100 * 1200 4/7 = 144
= 180 Number of mobile manufactured in C = 21/100 * 1200 *
Mobile manufactured in A = 20/100 * 180 = 36 3/7 = 108
Total number of manufactured products in A = 22/100 * Number of laptop manufactured in E = 125/100 * 108 =
1200 = 264 135
Number of Laptop manufactured in A = 264 – 36 = 228 Total number of manufactured in E = 16/100 * 1200 =
So, Statement II alone is not sufficient to the answer the 192
question. Number of mobile manufactured in E = 192 – 135 = 57
From Statement III, Required sum = 144 + 57 = 201
The number of laptop manufactured in B is 20% of the From quantity II,
total number mobile manufactured in D. Total number of products manufactured in A, B, D and F
So, Statement III alone is not sufficient to the answer the = 63/100 * 1200 = 756
question. Average = 756/4 = 189
From Statement I and II, Quantity I>quantity II
Number of mobile manufactured in B = 2/4 * 228 = 114 6) Answer: A
Total number of manufactured products in B = 14/100 * A = 20/100 * 120 = 24 years
1200 = 168 B = 15/100 * 120 = 18 years
Number of Laptop manufactured in B = 168 – 114 = 54 C = 25/100 * 120 = 30 years
Required total = 54 + 228 = 282 D = 10/100 * 120 = 12 years
4) Answer: D E = 30/100 * 120 = 36 years
Number of laptop manufactured in F’s age 7 years ago = 30 – 12 = 18 years
F=12/100*1200*75/100=108 F’s present age = 18 + 7 = 25 years

Page 150 of 199

Subscribe The Xpress Video Course & Mock Test Package for Bank & Insurance Exams
If there are any suggestions/ errors in our PDFs Feel Free to contact us via this email: admin@exampundit.in
Ultra Practice Bundle PDF
IBPS PO Prelims - Quantitative Aptitude
G age after 5 years = 30 * 8/5 = 48 years D + G + H = 24 * 3 = 72
G present age = 48 – 5 = 43 years H = 7/5 * 30 = 42 years
H = (43 – 1)/2 = 21 years G = 72 – 12 – 42 = 18years
Average ages of H, D and E = (21 + 12 + 36)/3 = 23 F = 3/2 * 18 = 27 years
years Required Average = (27 + 18 + 42)/3 = 29 years
Average ages of A and G = (43 + 24)/2 = 33.5 years 11) Answer: D
Difference = 33.5 – 23 = 10.5 years Required ratio = (30+7.5)% of 60000 : (20+25)% of
7) Answer: B 60000
F = 24 – 12 = 12 years = 37.5: 45
After 8 years F’s age = 12 + 8 = 20 years =5:6
After 8 years G’s age = 3/2 * 20 = 30 years 12) Answer: B
G age 9 years ago = 30 – 8 – 9 = 13 years Total money spent on cloth = 12.5% of 60000 =Rs.7500
8) Answer: C Total money spent on food = 30% of 60000 = Rs.18000
Average ages of F + G + B = 18 * 3 = 54 years Required sum = 7500+18000 =Rs.25500
Ratio of G and E = 100:150 = 2:3 13) Answer: A
G = 2/3 * 36 = 24 years Required percentage = ×100 = 50% more
F = 54 – 18 – 24 = 12 years 14) Answer: C
Average ages of H and F = 25 * 2 – 12 = 38 Required difference = (30+25)% of 60000 – (7.5+5)% of
H’s present age = 38 – 12 = 26 years 60000
H age 10 years ago = 26 – 10 = 16 years = 55% of 60000 – 12.5% of 60000
9) Answer: C = 42.5% of 60000
Sum of the ages of S and R = (30 + 24 + 36)/3 = 30 years = Rs.25500
R * D = 156 15) Answer: E
R * 12 = 156 New salary of the person = 125% of 60000 = Rs.75000
R = 13 years Expenditure on cloths = 12.5% of 60000 = Rs.7500
S = 30 – 13 = 17 years New expenditure on cloths = 7500+500 = Rs.8000
10) Answer: B

Page 151 of 199

Subscribe The Xpress Video Course & Mock Test Package for Bank & Insurance Exams
If there are any suggestions/ errors in our PDFs Feel Free to contact us via this email: admin@exampundit.in
Ultra Practice Bundle PDF
IBPS PO Prelims - Quantitative Aptitude
Percentage of expenditure on cloth = ×100 = 10.67%

Directions (16-20):
From pie-chart,
Peoples recovered from covid 19 in china = 20% of 12400 =2480
Number of males recovered from covid 19 in china = ×35 =1400
Therefore, number of female recovered from covid 19 in china
= ×27 = 1080
Similarly, values for all the countries are found and the results are tabulated.
Country No. of males No. of females Total
recovered recovered
China 1400 1080 2480
India 2100 1000 3100
UK 960 900 1860
USA 744 496 1240
Russia 2000 1720 3720
16) Answer: D Required ratio = 3620: 5060 i.e 181:253
Required average = = 1440.8 19) Answer: E

17) Answer: A Total peoples recovered from covid 19 in India = 3100

Required percentage = ×100 =65%less Peoples who have not recovered from covid 19 = ×60

18) Answer: B = 4650

Sum of the females recovered from India , UK, Russia 20) Answer: E

= 1000+900+1720 Total number of males recovered from covid 19

=3620 = 1400+2100+960+744+2000

Sum of the males recovered from India , UK, Russia = 7204

= 2100+960+2000 Total number of females who recovered from covid 19

= 5060 = 12400 -7204

Page 152 of 199

Subscribe The Xpress Video Course & Mock Test Package for Bank & Insurance Exams
If there are any suggestions/ errors in our PDFs Feel Free to contact us via this email: admin@exampundit.in
Ultra Practice Bundle PDF
IBPS PO Prelims - Quantitative Aptitude
= 5196 Required difference = 7204 – 5196 = 2008
Directions (21 – 25):
City Students Engineers Non – engineers
Delhi – 40% 40% of 1600 = 640 (5/8) x 640 = 400 240

Mumbai – 45% 45% of 1600 = 720 (2/9) x 720 = 160 560


Bangalore – 15% 15% of 1600 = 240 (4/5) x 240 = 192 48
21) Answer: C Required % = (176/240) x 100 = 73.333 %
Required ratio = 400: 560 = 5: 7 24) Answer: D
22) Answer: B Difference = 160 – 48 = 112
Total number of engineers = 400 + 160 + 192 = 752 Non-engineers from Mumbai = 560
Total number of non-engineers = 240 + 560 + 48 = 848 Required % change = ((560 – 112)/560) x 100 = 80%
Required difference = 848 – 752 = 96 less
23) Answer: A 25) Answer: D
Average number of engineers from Bangalore and Combined % distribution of Delhi and Bangalore
Mumbai together = (160 + 192)/2 = 176 together = 40 + 15 = 55%
Number of non-engineers in Delhi = 240 Required angle = 55% of 360 = 198o
Directions (26-30):
Cities Total vehicles Petrol Diesel
Chennai 800 500 300
Bangalore 600 400 200
Delhi 1000 600 400
Mumbai 720 450 270
Hyderabad 880 480 400
26) Answer: C Total number of vehicles in Bangalore in 2019 = 600 *
Required percentage = 450/600 * 100 = 75% 120/100 = 720
27) Answer: C Number of petrol vehicle in Bangalore = 720 * 5/8 = 450

Page 153 of 199

Subscribe The Xpress Video Course & Mock Test Package for Bank & Insurance Exams
If there are any suggestions/ errors in our PDFs Feel Free to contact us via this email: admin@exampundit.in
Ultra Practice Bundle PDF
IBPS PO Prelims - Quantitative Aptitude
Number of Diesel vehicle in Bangalore = 3/8 * 720 = Average number of petrol vehicle = (500 + 400 + 600 +
270 450 + 480)/5 = 486
Total number of vehicles in Mumbai in 2019 = 720 * Average number of Diesel vehicle = (300 + 200 + 400 +
125/100 = 900 270 + 400)/5 = 314
Number of petrol vehicle in Mumbai = 900 * 2/5 = 360 Required Difference = 486 – 314 = 172
Number of Diesel vehicle in Mumbai = 900 * 3/5 = 540 29) Answer: D
From quantity I, 30) Answer: B
Required sum = 450 + 360 = 810 In 2019 total number of vehicles in Chennai = 800 *
From quantity II, 125/100 = 1000
Required sum = 270 + 540 = 810 Number of petrol vehicle in Chennai in 2019 = 480
Quantity I = quantity II Number of Diesel vehicle in Chennai in 2019 = 1000 –
28) Answer: C 480 = 520
Directions (31 – 35):
Total students writing XAT exam in A = 28% of 13500 = 3780
Boys in city A = 3/7 x 3780 = 1620
Girls in City A = 4/7 x 3780 = 2160
Similarly, we find in all the cities
City Total students Boys Girls
writing XAT exam
A 3780 1620 2160
B 2025 900 1125
C 3510 1890 1620
D 2295 1215 1080
E 1890 1050 840
31) Answer: A Total number of girls writing XAT exam in all cities
Total number of boyswriting XAT exam in all cities together = 2160 + 1125 + 1620 + 1080 + 840 = 6825
together = 1620 + 900 + 1890 + 1215 + 1050 = 6675 Required difference = 150

Page 154 of 199

Subscribe The Xpress Video Course & Mock Test Package for Bank & Insurance Exams
If there are any suggestions/ errors in our PDFs Feel Free to contact us via this email: admin@exampundit.in
Ultra Practice Bundle PDF
IBPS PO Prelims - Quantitative Aptitude
32) Answer: C 34) Answer: D
Number of boys writing XAT exam in city F = 7/6 x Number of boys in B and E together = 900 + 1050 =
2160 = 2520 1950
Number of girls writing XAT exam in city F = 4/15 x Number of girls in C and D together = 1620 + 1080 =
1890 = 504 2700
Total students from city Fwriting XAT exam = 2520 + Required % change = [(2700 – 1950) / 2700] x 100 =
504 = 3024 27.8% less
33) Answer: B 35) Answer: E
Average students writing XAT exam in City B and City Number of boys in city A = 28% x 3/7 = 12%
D together = (2025 + 2295)/2 = 2160 Total students in city A and city B together = 28% +
Average studentswriting XAT exam in City A and City E 15% = 43%
together = (3780 + 1890)/2 = 2835 Hence required ratio = 12%:43% = 12:43
Required percentage = [2160/2835] x 100 = 76.2%
Directions (36-40):
From the given pie-chart,
Total newspaper sold in Mumbai = 21.43% of 52500 = (52500) = 11250

Total newspaper sold in Chennai = 28.57% of 52500 = (52500) = 15000


Total newspaper sold in Delhi = 13.33% of 52500
= (33.33% -20%) of 52500
= ( - ) of 52500

= (52500)
= 7000
Total newspaper sold in Bangalore = 10% of 52500 = 5250
Therefore,
Total newspaper sold in Gujarat = 52500 – (11250+15000+7000+5250)
= 14000

Page 155 of 199

Subscribe The Xpress Video Course & Mock Test Package for Bank & Insurance Exams
If there are any suggestions/ errors in our PDFs Feel Free to contact us via this email: admin@exampundit.in
Ultra Practice Bundle PDF
IBPS PO Prelims - Quantitative Aptitude
City Number of Indian Number of The Hindu Total newspapers
express newspaper sold newspaper sold sold
Mumbai 5250 6000 11250
Chennai 8000 7000 15000
Delhi 4300 2700 7000
Bangalore 2825 2425 5250
Gujarat 4500 9500 14000
36) Answer: C = 52500 – 24875
Number of the Hindu newspaper sold in Delhi is second = 27625
lowest i.e. 2700 Average of The Hindu newspapers sold in all the cities
37) Answer: A together
Required ratio (7000+2825): (5250+9500) =
9825:14750 = 5525
393:590 Required difference = 5525 – 4975 = 550
38) Answer: E 40) Answer: A
Required percentage = ×100 = 75% Number of damaged newspaper = 25% of (2700) + (20%
39) Answer: B of 2425) + (30% of 9500)
Total number of Indian express newspaper sold in all the = 675 + 485 + 2850
cities together = 4010
= 5250+8000+4300+2825+4500 41) Answer: A
= 24875 Total number of employees in the organization =
Average of no. of Indian express sold in all the cities
together
So, total number of employees in B =
=
Let the total number of employees in B below 35 years
=4975 and above 35 years be X and Y respectively.
Total number of The Hindu newspaper sold in all the Then,
cities together
Page 156 of 199

Subscribe The Xpress Video Course & Mock Test Package for Bank & Insurance Exams
If there are any suggestions/ errors in our PDFs Feel Free to contact us via this email: admin@exampundit.in
Ultra Practice Bundle PDF
IBPS PO Prelims - Quantitative Aptitude
B35 + A35 = 1320
B35 male employees = 360
9X + 7Y = 1310 -------(i)
9 = 360
Also X+Y = 170 ---------(ii)
10 =
Solving (i) & (ii), we get
X = 60 and Y = 110 A35 total employees = 1320 – 400 = 920

Required ratio = 110:60 A35 female employees =

= 11:6 44) Answer: D


42) Answer: C Total number of employees below 35 years in E =
Total number of employees in the organization = ( ) = 200

Total number of employees above 35 years in E =


Now, total number of employees in F = ( )= 180

Let the number of employees below 35 years and above So, the total number of employees in E = 200 + 180 =
35 years be X and Y in department F respectively. 380
So, X + Y = 500 ---------(i) Total number of employees in A =

3X + Y = 1220 --------(ii) 45) Answer: E

Solving (i) and (ii), we get Total number of employees in D =


X = 360 and Y = 140 So, the number of employees above 35 years in D = 240
Thus, the number of males below 35 years in F = – 96 = 144
The number of female employees above 35 years in D =

43) Answer: A
Let the employees below 35 year be B35 and those above
35 be A35

46) Answer: B

Page 157 of 199

Subscribe The Xpress Video Course & Mock Test Package for Bank & Insurance Exams
If there are any suggestions/ errors in our PDFs Feel Free to contact us via this email: admin@exampundit.in
Ultra Practice Bundle PDF
IBPS PO Prelims - Quantitative Aptitude

Cities Total Male population Female population


A 2700 1215 1485
B 3000 1650 1350
C 3300 2145 1155
D 3600 2520 1080
E 2400 1440 960
Difference = 1485 – 1080 = 405
47) Answer: B 48) Answer: C
Number of literate population in B = 3/5 * 3000 = 1800 Required average = (1215 + 1650 + 2145 + 2520 +
Number of illiterate population in B = 2/5 * 3000 = 1200 1440)/5 = 1794
Number of male literate population = 3/5 * 1650 = 990 49) Answer: A
Number of male illiterate population = 2/5 * 1650 = 660 Required percentage = [(1155 + 960)/3000] * 100 =
Number of female literate population = 1800 – 990 = 810 70.5%
Number of female illiterate population = 1200 – 660 = 50) Answer: E
540 Required ratio = 1350:1215 = 10:9
Difference = 810 – 540 = 270

Data Interpretation Practice Questions PDF Download


Download Quantitative Aptitude Practice Questions PDF

Caselet DI
Q.1. In Organization A, Ratio of female and male is 6:7. Female at Organization C is 750 more than Male at
Organization A. In Organization B, Male and Female are equal. Male at D is 250 more than the double of

Page 158 of 199

Subscribe The Xpress Video Course & Mock Test Package for Bank & Insurance Exams
If there are any suggestions/ errors in our PDFs Feel Free to contact us via this email: admin@exampundit.in
Ultra Practice Bundle PDF
IBPS PO Prelims - Quantitative Aptitude
Female at A. Female at D is 2550.Male and Female ratio at D is 11:15.Total Male are 5655. And Male at C is
double the Female at A.

1.20% Female of D are doing MBA and 25% Female c.760


of B are doing MBA. What is total no. of Female d.390
doing MBA from these organizations? e.none
a.655 4.Age of some people of every organization is below
b.743 30 and someare above 30. 12.5% of total employee
c.815 from B is above 30 and25% of total employee of D is
d.735 above 30. Then find the ratio of above 30 employee of
e.none D to the same of B?
2. What is the average (approx)Malein all a.123:67
theoraganizations together? b.221:61
a.5655 c.223:61
b.5699 d.234:67
c.5234 e.none
d.4567 5. 20% out of total Female and 25 % out of total Male
e.none at Chas resigned. What is total number of people who
3.Out of the Total people doing job at A in three are still working at C?
departments, marketing, accounts, and IT are in the a.2571
ratio of 4:3:2 respectively.Then total people in IT b.3456
department? c.3456
a.234 d.4532
b.450 e.none

Q. (6-10) In a college, there are four branches ECE, CE, ME, and EE. They are offering two types of courses,
M.Tech and B.Tech. Seat for different courses are different. Ratio of total seat capacity in ME and ECE is 7:6.
Page 159 of 199

Subscribe The Xpress Video Course & Mock Test Package for Bank & Insurance Exams
If there are any suggestions/ errors in our PDFs Feel Free to contact us via this email: admin@exampundit.in
Ultra Practice Bundle PDF
IBPS PO Prelims - Quantitative Aptitude
In ME seat capacity of M.Tech is 30. Total seat capacity of CSE is 60% of the seat capacity of EE. Total
B.Tech seats in allDeptartments together are 330. M.Tech seat at EE is 33.33% of B.Tech at EE. B.Tech and
M.Tech seat ratio in CSE is 3:1. Total students in CSE is 60. In ECE ,B.Tech:M.Tech is 5:1.
6. College want to increase 40% B.Tech seat in ECE c.3:4
and 20 seat in ME.Seat of the other two is same. Then d.31:9
total seat inB.Tech now is ? e.none
a.540 4.In which branch difference of B.Tech and M.Tech
b.234 seat capacity is lowest?
c.390 a.ME
d.240 b.ME&ECE
e.none c.ECE
7. In one year, 80% seat is filled up and inM.Tech d.CSE&EE
60% seat is filled up. Then what is the total no. of e.none
vacant seat in ECE? 5. College want to start a new branch Civil. In Civil
a.28 B.Tech seat is 10% less than B.Tech seats in ME and
b.32 M.Techseat is 20% more than M.Tech seat of ECE.
c.38 Then total seats in Civil are?
d.18 a.121
e.none b.123
8. Find ratio of B.Tech seat in ME and CSE together c.132
with M.Tech seats of ECE and EE together? d.143
a.31:3 e.none
b.37:12

Q (11-15) There are 2500 student in a college. All the students play different game. Each player either district
or state level player. 30% out of totalplayer plays Football. No of district level Cricket player is equal to the no
of district level Basketball player.Ratioof total Basketball players to the total Cricket player is 3:5. 10% out of
Page 160 of 199

Subscribe The Xpress Video Course & Mock Test Package for Bank & Insurance Exams
If there are any suggestions/ errors in our PDFs Feel Free to contact us via this email: admin@exampundit.in
Ultra Practice Bundle PDF
IBPS PO Prelims - Quantitative Aptitude
total of studentsare Hockeyplayers.Total Hockey players is half of total Volleyballplayers. Difference of
district levelVolleyball and State Volleyball player is 200.District Vollyballplayersare equal to District
Football players.Difference of District Cricket player and State Hockey is same as District Hockey player.
11. If the ratio of district Hockey player to the state a.33
Hockey players 1:9.then district Hockey players is b.25
what persent of district Cricket players? c.23
a.3.23 d.20
b.4.67 e.none
c.6..66 14 .Total Basketball players is what percent more or
d.5.61 less total District Football player.
e.none a.7.07
12. What is the ratio of total district Football player b.7.14
and district Cricket playars to the total State Volleyball c.6.19
player and state Basketball player? d.4.44
a.24:11 e. none
b.13:12 15.Avearge of Football,cricket,Volleyballplayers is?
c.11:19 a.632
d.11:23 b.234
e.none c.625
13. Out of total Volleyball player 28% is participate in d.631
college sports in some team. If one team has 7 player e. none
than how many team has participate?

Q. (16-20) A,B,C,D four students give exam in two subject Math and Reasoning. Total marks is 50. They got
different Marks indifferent Subjects. A got 30 marks in Math. B got 35 marks in reasoning. C is got 33.33%
marks morethan A got in Math. Ratio of Reasoning marks of A and Math marksof D is 7:5. Where total

Page 161 of 199

Subscribe The Xpress Video Course & Mock Test Package for Bank & Insurance Exams
If there are any suggestions/ errors in our PDFs Feel Free to contact us via this email: admin@exampundit.in
Ultra Practice Bundle PDF
IBPS PO Prelims - Quantitative Aptitude
marks Agets is 65.D got 5 marks more in Math than Reasoning. Average of marks C got is 25.Total age of
A,B,C, and D is 96.Math marks of D is age of D. Age of C is one more than double of his Reasoning marks.

16. Ratio of age A:B is 12:13. Than what is the age of b.42.44
B? c.43
a.21 d.44.44
b.24 e.none
c.26 19. What is ratio of total Math marks of all and total
d.25 Reasoning marks of all together?
e.none a.37:39
17. Ratio of average marks of C and B is 5:6.Then B b. 31:43
get in Math is ? c.43:47
a.25 d. CND
b.24 e. non of these
c.23 20. Marks of Cin Math is what percent of age of D?
d.23 a.120
e.21 b.124
18. Total marks of A is what percent more and less c.136
than total makrs of D? d.160
a.41.44 e.none

Q.(21-25) Vitamin C tablet sell in shop at P in March is 330. Where in April at Q is is 480. At R in April total
sell is 10% more than total sell in march at P. Total sell at R in March andApril is 783. Average sell at Q is
400.And sell in April at P is 25% more than the sell in March at Q. Total sell in March at P,Q,R and T is 1324
and total sell in April 25% more than in March at all the shop.
21. Total sell of P is what percentage of total sell of Q a.91.34
in both month? b.89.99
Page 162 of 199

Subscribe The Xpress Video Course & Mock Test Package for Bank & Insurance Exams
If there are any suggestions/ errors in our PDFs Feel Free to contact us via this email: admin@exampundit.in
Ultra Practice Bundle PDF
IBPS PO Prelims - Quantitative Aptitude
c.91.25 24. In June at P 20% and at Q 25% sell increase than
d.95.55 April. Then what is total sell in June at Pand Q?
e.none a.1236
22. What is average sell in April all the shop together? b.1080
a.413.75 c.1452
b.411.23 d.2596
c.422.1 e. none
d.456.7 25. In P, per piece price is Rs 5 where in R is Rs 4.
e.none What is total amount of vitamin C sell in March
23. What is ratio of total sell in R to the T in both inthese two shop?
month? a.3102
a.89:79 b.2345
b.78:79 c.6545
c.23:79 d.4564
d.87:74 e.none
e.none

Q (26-30) In city X, students prepare for JEE is 250.In Y ratio of student Prepare for JEE and NEET is 3:2.
Average student prepare for JEE and NEET in Z is 230. In Z , students prepare for JEE is 10 less than NEET
in Y. Where at Y, JEE student is 50 more than NEET student atZ.Averageof total student in X is 235.
26. What is the ratio of total student from X and Y? 27. In which city total student (JEE+NEET) is
a.43:21 maximum?
b.43:87 a.Y
c.47:52 b.X&Y
d.47:32 c.X
e.none d.Z
e.none
Page 163 of 199

Subscribe The Xpress Video Course & Mock Test Package for Bank & Insurance Exams
If there are any suggestions/ errors in our PDFs Feel Free to contact us via this email: admin@exampundit.in
Ultra Practice Bundle PDF
IBPS PO Prelims - Quantitative Aptitude
28. What is difference between total JEE student from b.564
X and Y togehter and total NEET student from Z and c.578
Y? d.456
a.99 e. none
b.97 30. JEE student at X is what percent more or less than
c.95 NEET at Y?
d.92 a.23.78
e.none b.20.19
29. In CITY W, only JEE student is their and its 20% c.24.56
more than total student of X. What is total student at d.23.45
W? e.none
a.567

Q (31-35) In city M, 500 people use whatsapp and the ratio of people useinstagram and facebook is2:3. In city
N 33.33% more people use facebook than same in M and 50 less people use instagramin N than whatsapp use
in M. Totalpepopleuse instagram in M ,N, O is 1020 andtotal people use whatsapp, facebook,instagram is 700
at O. Which is equal to the total no of pepole use whatsapp and instagram in M. and ratio of people use
facebook and whatsapp in N is 4:5.
31. Facebook use in O is half of instagram use in O. b.23:34
than what is whatsapp user in O? c.21:89
a.234 d.21:31
b.211 e.none
c.245 33. Facebook use in M is what percent of whatsapp
d.243 use in N?
e.none a.24
32. What is ratio of total people M and N? b.67
a.20:27 c.44
Page 164 of 199

Subscribe The Xpress Video Course & Mock Test Package for Bank & Insurance Exams
If there are any suggestions/ errors in our PDFs Feel Free to contact us via this email: admin@exampundit.in
Ultra Practice Bundle PDF
IBPS PO Prelims - Quantitative Aptitude
d.60 35.In another city P, facebook use is 30% more than
e.none facebook use M, instagram use 10% more than
34. Instagram user in M is what percent more or less instagram use in O, and whatsapp use 25% more than
than instagram user in O? whatspp use in N. then total user in P is?
a.34 a.1234
b.45.94 b.1131
c.56.53 c.1422
d.45.67 d.1311
e.none e.none

Q(36-40) In city A and B there are four types of people living i.e. Writer, Painter, Singer,Dancer.Writer of city
B is 20% More than the writer of A. Ratio of painter in city A and B is 7:5. Dancer at A is 14 less than painter
at Band dancer at B is 28 less than the painter at A. Total dancer is 78. Difference of singer A and B is 12.
Ratio of singer of A and B is 17:13. Writer at B is 48.
36. What is ratio of total writer and painter in A with 38. Find Ratio of total people of A with total people of
the total dancer and singer in B? B?
a. 112:12 a.191:167
b.111:76 b.193:197
c.110:81 c.193:199
d.111:132 d.197:179
e.none e.none
37.Total writer is what percent of total singer? 39. Total painters in both A and B is how much
a.97.78 percent more or less than the total singer?
b.98.81 a.29
c.93.23 b.33.33
d.92.45 c.12.2
e.none d.25
Page 165 of 199

Subscribe The Xpress Video Course & Mock Test Package for Bank & Insurance Exams
If there are any suggestions/ errors in our PDFs Feel Free to contact us via this email: admin@exampundit.in
Ultra Practice Bundle PDF
IBPS PO Prelims - Quantitative Aptitude
e.none b.23
40. What is difference between writer, painter, dancer c.45
at A together with the painter, dancer and singer at B? d.15
a.13 e.none

Q(41-45) In 2019, Ibps vacancy is 4420 which is 25% more than the CGL vacancy in 2018.In 2018 railway
vacancy is 12.5% less than the Railway vacancy in 2019. Total vacancy in 2019 is 15220 which is 25% more
than the total vacancy of 2018.Total vacancyof IBPS and Railway in 2019 is 11900.
41.What is average(approx) vacancy of 2018 in exam? c.1432:143
a.4521 d.1411:2805
b.7895 e.none
c.4237 44. What is the difference between Railway vacancy in
d.5463 2018 and CGLvacancy in 2019?
e.none a.897
42. In 2020, IBPS vacancy decrease 25% than 2019 b.987
IBPS vacancy, where CGL is increase 25% than same c.322
in 2019, and Railway vacancy is same as 2018. What d.935
is total vacancy in 2020? e.none
a.19210 45.In IBPS vacancy in 2019,after some time vacancy
b.12345 is decrease by 25%, later it increase by 20%,Then
c.21345 what is different of vacancy in first and now?
d.13145 a.442
e.none b.332
43. What is ratio of total IBPS vacancy (2018+2019) c.334
to the total Railway vacancy (2018+2019)? d.443
a.1324:1456 e.none
b.1321:1797
Page 166 of 199

Subscribe The Xpress Video Course & Mock Test Package for Bank & Insurance Exams
If there are any suggestions/ errors in our PDFs Feel Free to contact us via this email: admin@exampundit.in
Ultra Practice Bundle PDF
IBPS PO Prelims - Quantitative Aptitude

Q(46-50) In college A , total teachersare 300,out of this 70% are asstprofessor,rest of is lab asst. In college B
total number of teachers is half of the teacher of A. At B, 80% is asstprofessor.Total teacher at C is 10 less
than the no of asst professor at A. In college C, lab asst is equal to the lab asst of B. In college D, total lab asst
is 40, which 25% of total teacher.
46. What is difference between total asst professor of a.43
A and B together and total lab asst of C and D b.65
together? c.76
a.432 d.75
b.342 e.none
c.260 49. What is average of asst professor of all the college
d.234 together?
e.none a.123
47. In college D, 60% asst professors are female b.155
where was 20% lab asstare female. So no of total c.134
female in college D is? d.123
a.80 e.none
b.67 50. Lab asstof A is how much more or less to the Lab
c.56 asst of D?
d.68 a.123
e.none b.125
48. In college B, out of the asst professors some have c.122
Phd, some have not. So the ratio of phd holder with the d.123
non Phd holder is 3:5. So the no of Phd holder in e.none
college B is?

Page 167 of 199

Subscribe The Xpress Video Course & Mock Test Package for Bank & Insurance Exams
If there are any suggestions/ errors in our PDFs Feel Free to contact us via this email: admin@exampundit.in
Ultra Practice Bundle PDF
IBPS PO Prelims - Quantitative Aptitude
Caselet DI - Answer and Detailed Explanation
Q(1-5) 2. ANS A
Let, male at A is 7x, female 6x. total male all the organization is (945+1220+1620+1870)
Female at C is 7x+750. = 5655
Male at D is 12x+250 and female is 2550. So the average is 5655 4 =1413.75
Now according the question, 3. ANS D
Or, Total people of A is 810+945=1755

Or, So people work in IT is =1755 = 390.

Or, 4. ANS B
Putting the value of x we get total people of B is 1220 +1220 =2440
female at A =6*135=810, 12.5% is above 30.
male at A = 7*135=945, So, above 30 age is 2440*12.5/100 =305
Female at C =7*135+750=1695, Total people of D is 2550 +1870 =4420
male at C is 810*2 =1620, Above 30 is 25 % , so above 30% is 4420*25/100 =1105
male at D = 12*135+250= 1870. So the ratio of above 30 age of D and B is 1105:305 =
Now total male is 5655. So male at B is [5655 – 221:61
(945+1620+1870)] = 1220. 5. ANS A
City Female Male In C total female is is 1695. 20% resigned. So working is
A 810 945 80%.
B 1220 1220 80% of 1695 is 1695*80/100 = 1356
C 1695 1620 Total male resigned is 1620. 25% resigned.75% is still
D 2550 1870 working.

1. ANS -C Total male working is 1215.

From B 25% female doing MBA. i.e 1220*25/100=305 Total people working is = 2571.

From D 20% female doing MBA i.e. 2550*20/100= 510. Q(6-10)

Total female doing MBA from these is 305 +510 =815. Let,

Page 168 of 199

Subscribe The Xpress Video Course & Mock Test Package for Bank & Insurance Exams
If there are any suggestions/ errors in our PDFs Feel Free to contact us via this email: admin@exampundit.in
Ultra Practice Bundle PDF
IBPS PO Prelims - Quantitative Aptitude
Total seat at ME dept is 7x. and Ece is 6x. 80% is fill up, so the vacant B.tech seat is 20.
M.tech seat in ME is 30, B.tech seat in ME is 7x-30. Total M.tech seat is 20.
B.tech seat : M.tech seat in ECE dept is 5:1. 60% is fill up,
Let, 5x is B.tech seat and x is M.tech seat. So vacant M.tech seat is 20*40/100 = 8
Now if Seat in CSE is 60. So EE seat is 100. So total vacant seat in ECE is 20+8=28.
B.tech seat in CSE is= 60*3/4 =45 [B.tech:M.tech ratio 8. ANS D
3:1 in CSE] Total B.tech in ME and CSE is 110+45 =155
M.tech in CSE is =60-45=15 Total M.tech in ECE and EE is 20+25 =45
In EE, M.Tech seat is 33.33% of B.tech Seat. Total seat So the respective ratio is 155:45 = 31:9.
is 100 so B.tech is 75 and M.tech seat is 25. 9. ANS B
Total B.Tech seat in college is 330. In ece difference is 20, ME is 20, IN cse is 30, in ee is
So, 330= 5x +7x-30+45+75 50.
Or, 12x=240, x=20 So difference lowest in both ME and ECE.
Putting the value we get all the department B.tech and 10. ANS B
M.tech Seat. Civil b.tech seat is 10% less than b.tech in ME.
Department B.tech M.tech b.tech seat is 110*90/100 =99
ECE 100 20 m.tech seat is 20% more than m.tech in ece .
ME 110 30 so m.tech seat is 20*120/100=24
CSE 45 15 total seat in civil is =123.
EE 75 25 Q(11-15)
6. ANS C We can use District football player as DF, state football
40% b.tech seat increase in ece so total B.tech is now player as SF, District cricket player as DC, state cricket
140. 20 seat increase in ME. player as SC, District Hockey player as DH, state hockey
Total seat increase in B.tech is 60. player as SH, District volleyball player as DV, state
Total b.tech seat is now 330+60= 390. Volleyball player as SV, District basketball player as
7. ANS A DB, state basketball player as SB.
Total B.tech seat in ECE is 100 30% of total student is play football= 2500*30/100 =750

Page 169 of 199

Subscribe The Xpress Video Course & Mock Test Package for Bank & Insurance Exams
If there are any suggestions/ errors in our PDFs Feel Free to contact us via this email: admin@exampundit.in
Ultra Practice Bundle PDF
IBPS PO Prelims - Quantitative Aptitude
Now DC =DB. So, 10x=250
Let total basket ball player is 3x and total cricket ball x=25
player is 5x. DH=25 ,DC=375.
10% hockey player, 250 student play Hockey,. Than required percent is *100 =6.66%
Total Volleyball player is 500
12. ANS A
SO, 8x =2500 -750-500-250
Total DF+DC =350+250=600
X=1000
Total SB+SV=125+150= 275
Total cricket player is 625 AND TOTAL Basketball
So the ratio is 600:275 =24:11
Player is 375.
13. ANS D
Now
Total Volleyball player participate in college sports is =
DC – SH=DH, DC=SH+DH, DC=250[Total hockey
500*28/100 = 140
player is 250]
If one team has 7 players than total team is
DC=DB=250, so SB=125
140/7 =20
Ds=625-250 =375
14. ANS B
DV-SV=200 AND DV+SV =500
Total basketball player is 375. Total DF is 350.
Solving these two DV=350, SV=150
25 more than DF.
DV=DF
So the required percentage =
DF=350 so DS= 400.
15. ANS C
Sports Total District State
Football 750 350(DF) 400(SF) Average football , cricket, volleyball player is = (

Cricket 625 250(DC) 375(SC) )


Hockey 250 Q(16-20)
Volleyball 500 350(DV) 150(SV) A get 30 marks in math. C got 33.33% more.
Basketball 375 250(DB) 125(SB) So C get in math, 30*133.33/100 =40

11. ANS C Let, Reasoning marks of A is 7x and math marks of D is

Let , 5x.

DH =x SH=9x Now 7x=65-30, x= 5

Page 170 of 199

Subscribe The Xpress Video Course & Mock Test Package for Bank & Insurance Exams
If there are any suggestions/ errors in our PDFs Feel Free to contact us via this email: admin@exampundit.in
Ultra Practice Bundle PDF
IBPS PO Prelims - Quantitative Aptitude
So D get in math is 25. In reasoning get 20. We don’t know the math marks of B. so we cannot get
Average of marks of C is 25. Total marks 50. So he tget the total marks of math.
in reasoning is 10. So cannot determine.
Age of D is 25. Age of C is 10*2+1=21 20. ANS D
Marks of math is 40 and age of D is 25
MATH REASONING AGE Required percentage is = (40/25)*100=160%
A 30 35 Q(21-25)
B 35 Total sell in march at P is 330. I At R in march 10%
C 40 10 21 more than this.
D 25 20 25 So at R in march total sell is 363. In april total sell at R is
16. ANS C 783-363=420
Total age of A+B is 96-46 =50 Total at Q is 800. So at Q in march 320.
Let, A age is 12x, B age is 13x. In april at P total sell is 320*125/100 =400
25x =50 , x=2 Total sell in march is 1324 , at t , sell in march is 1324-
So B age is 26. 330-320-363= 311
17. ANS A Total sell in April is = 1324*125/100 = 1655
Average marks of C is 25. So at T in April total sell is 1655-400-480-420=355
Ratio of average marks of C: average marks of B =5:6 MARCH APRIL
So average marks of B is 30. P 330 400
Total marks of math and reasoning is 60, Q 320 480
so math marks is 60-35 =25 R 363 420
18. ANS D T 311 355
Total marks of A is 65. Total marks of D is 45. 21. ANS C
So A get 20 Marks more. Total sell of P in both month is 330+400 =730
So required percent is = 44.44% Total sell of Q in both month is =320+480 =800
So required percent is (730/800)*100 =91.25%
19. ANS D
22. ANS A

Page 171 of 199

Subscribe The Xpress Video Course & Mock Test Package for Bank & Insurance Exams
If there are any suggestions/ errors in our PDFs Feel Free to contact us via this email: admin@exampundit.in
Ultra Practice Bundle PDF
IBPS PO Prelims - Quantitative Aptitude
Total sell in April is =400 +480+420+355 =1655 Z 198 262
So the average is 1655/4 =413.75 26. ANS C
23. ANS D Total student from X is 250+220=470
Total sell in R is 363+420=783 Total student from is Y is 312+208 =520
Total sell in T is = 311+355= 666 So required ratio is = 470:520 = 47:52
So required ratio is = 783:666 =87:74. 27. ANS A
24. ANS B At X total student is = 470
Sell at P in june is =400*120/100 =480 At Y total student is =520
Sell at Q in june is =480*125/100 =600 At Z total student is =198+262=460
Total sell in June is 1080. So maximum student is at Y.
25. ANS A 28. ANS D
Total amount sell at P in march is 330*5 =1650 Total jee student from X and Y is 250+312=562
Total amount sell at R inmarch is 363 *4= 1452. Total Neet student from Y and Z is 208+260=470
So total amount is 3102. So the difference is 562-470 = 92
Q(26-30) 29. ANS B
Let, At Y , jee student is 3x and neet student is 2x. Total student at X is 470.
At z total student is 460. And jee student is 2x-10 and So total student at W is =( 470*120/100) = 564
neet is 3x -50 30. ANS B
So 2x-10+ 3x -50 = 460 Jee at X is 250 and neet at Y is 208.
Or, 5x =520 x=104 So the difference is 42.
Jee student at y is 312 and neet student is 208. So required percentage is = (42/208)*100 =20.19%
At z jee student is 198 and neet student is 262. Q(31-35)
At X, jee student is 250 , total student 470. Let,
So neet student at X is 220. Instagram use in M is 2x and facebook use in M is 3x.
city jee Neet At N 33.33% more people use facebook than use
X 250 220 facebook in M.
Y 312 208 Facebook use in N is = 3x*133.33/100 = 4x

Page 172 of 199

Subscribe The Xpress Video Course & Mock Test Package for Bank & Insurance Exams
If there are any suggestions/ errors in our PDFs Feel Free to contact us via this email: admin@exampundit.in
Ultra Practice Bundle PDF
IBPS PO Prelims - Quantitative Aptitude
Instagram use in N is 500-50 =450 So Total user of facebook in P =300*130/100 =390
According to the question, Total user of whatsappin P =500*125/100 =625
700= 500+2x, x=100 Total user of instagram in P is = 370*110/100 = 407
Instagram use is 200, facebook use 300 in M. Total people at P is 407+625+390 =1422.
Facebook use is 400, whatsapp is 500 in N. Q(36-40)
Total instagram use in three city is 1020 Writer in B is 48.which is 20% more than writer in A.
So, 200 + 450+ Instagram in O =1020 So writer in A is 48*100/120 = 40.
Instagram in O =370 Let, painter in A is 7d and painter in b is 5d.
CITY FACEBOOK INSTAGRAM WHATSPP Dancer in a =5d-14 dancer in B =7d-28
M 300 200 500 According to question,
N 400 450 500 5d-14+7d-28=78,
O 330-a 370 a Or, 12d=120, d=10
31. ANS C In A painter is 70, dancer is 36.
instagram user in o is 370. In B painter is 50, dancer is 42.
Facebook user is 185. Now singer in A is 17c and singer in B is 13c
So whatsapp user is 330-185 =245 17c-13c=51 ,c=3
32. ANS A Singer in A is 51 and singer in B is 39.
Total people of M is 300+200+500 =1000 CITTY WRITER PAINTER DANCER SINGER
Total people of N is 400+450+500 =1350 A 40 70 36 51
So respective ratio is = 1000:1350 = 20:27 B 48 50 42 39
33. ANS D 36. ANS C
So required percentage is = *100 =60% Total writer and painter in A is 40+70 =110
Total dancer and singer in B is 42+39 =81
34. ANS B
So respective ratio is = 110:81.
Instagram in M is 200 and in O is 370.
37. ANS A
So the difference is 170.
Total writer is 40+48=88
Required percentage is (170/370)*100 = 45.94%.
Total singer is 51+39=90
35. ANS C
Page 173 of 199

Subscribe The Xpress Video Course & Mock Test Package for Bank & Insurance Exams
If there are any suggestions/ errors in our PDFs Feel Free to contact us via this email: admin@exampundit.in
Ultra Practice Bundle PDF
IBPS PO Prelims - Quantitative Aptitude
So the percentage is (88/90)*100 =97.78% EXAM 2018 2019
38. ANS D IBPS 2635 4420
Total people at A is 40+70+36+51 = 197 RAILWAY 6545 7480
Total people at B is 48+50+42+39 =179 CGL 3536 3320
So respective ratio is 197:179. 41. ANS C
39. ANS B total vacancy in 2018 is 2635+6545+3536 =12716
Total painter is 70+50 =120 so the average is 12716/3 = 4238.66
Total singer is 51+39 =90 42. ANS A
Painter is 120-90 = 30 more than singer. In 2020 ibps vacancy is 25% decrease,
So respective percentage is =(30/90)*100 =33.33% so the vacancy is 4420*75/100 =3315
40. ANS D cgl vacancy is 25% increase 2020.
Total no of writer, painter, dancer in A is 40+70+36 So cgl vacancy is 7480*125/100 =9350
=146 So total vacancy in 2020 is 3315+9350+6545 =19210
Total no of painter dancer , singer at B is 50+42+39 43. ANS D
=131 Total ibps vacancy is 4420+2635 =7055
So the difference is 146- 131 = 15 Total railway vacancy is 6545+7480=14025
Q (41-45) So the ratio is 7055:14025 =1411:2805
IBPS vacancy in 2019 is 4420. 44. ANS B
CGL vacancy in 2018 is 25% less ibps vacancy 2019 So the difference 7480-6545=935
So, vacancy is= 4420*100/125 = 3536. 45. ANS A
Total cgl vacancy in 2019 is 15220-11900=3320 Ibps 2019 vacancy is 25% decrease than it increase 20%.
Railway vacancy in 2019 is 11900 – 4420 = 7480. So,now vacancy is =4420*75/100 *120/100 =3978
Railway vacancy in 2018 is 7480*87.5/100=6545 So the difference is 4420-3978 = 442.
Total vacancy is 2018 is 15220*100/125 = 12716 Q(46-50)
Ibps vacancy in 2018 is =12716-6545-3536 =2635 Total teacher in college A is 300.
Out of this 70% are asst professors
So asst professors at A is 210, and lab asst is 90.

Page 174 of 199

Subscribe The Xpress Video Course & Mock Test Package for Bank & Insurance Exams
If there are any suggestions/ errors in our PDFs Feel Free to contact us via this email: admin@exampundit.in
Ultra Practice Bundle PDF
IBPS PO Prelims - Quantitative Aptitude
Total teacher at B is 150. So the difference is 330-70=260.
In B 80% is asst professor 47. ANS A
So asst professor at B is 120 60% asst professor is female at D.
So lab asst is 30. So female asst professor at D is = 120*60/100 =72
Lab asst in C is 30. 20% lab asst at D is female.
total teacher at C is 210-10 =200 Female lab asst is 40*20/100 =8
so asst professors at C is 170. Total female teacher is 80.
IN college D ,lab asst is 40, which is 25% of total 48. ANS D
teacher. At B, out 120, phd holder is 3x and non phd holder 5x.
So total teacher is 40*100/25 =160. So, 8x=120
Asst professor at D is 120. x=15
COLLEGE ASST LAB TOTAL So total non phd holder is 5*15 =75
PROFESSOR ASST 49. ANS B
A 210 90 300 Total asst professor in all the college
B 120 30 150 =(210+120+170+120)= 620
C 170 30 200 So the average is 620/4 =155
D 120 40 160 50. ANS B
46. ANS C Lab asst at A is 90 and Lab asst at D is 40.
Total asst professor at A and B is 210+120=330 So the difference is 50.
Total lab asst at C and D is 40+30 =70 So required percentage is 50/40*100 =125%

Caselet DI Practice Questions PDF Download


Download Quantitative Aptitude Practice Questions PDF

Page 175 of 199

Subscribe The Xpress Video Course & Mock Test Package for Bank & Insurance Exams
If there are any suggestions/ errors in our PDFs Feel Free to contact us via this email: admin@exampundit.in
Ultra Practice Bundle PDF
IBPS PO Prelims - Quantitative Aptitude
Mixed DI
Direction (1-5) Study the charts and answer the question.
Total number of employees =3600. Percentage of employees working in different department is given in the pie chart.
Percentage of female in respective departments is given in the table.

Accounts HR Marketing IT
Departments Percentage of female

Accounts 40%
10%
30% HR 60%

35% Marketing 30%

IT 55%
25%

1. What is the total number of females working in HR 3. What is respective ratio of number of males
and IT departments together? working in HR department to the number females in
a.678 marketing department?
b.789 a.18:13
c.670 b.11:18
d.768 c.20:21
e.738 d.17:21
2. How many males work in the IT department? e.17:18
a.132 4. What is the difference of total male in all
b.144 departments of the company and the total number of
c.168 female in Accounts, HR, and IT departments
d.162 together?
e.156 a.789
b.882
Page 176 of 199

Subscribe The Xpress Video Course & Mock Test Package for Bank & Insurance Exams
If there are any suggestions/ errors in our PDFs Feel Free to contact us via this email: admin@exampundit.in
Ultra Practice Bundle PDF
IBPS PO Prelims - Quantitative Aptitude
c.567 a.42.85
d.890 b.85.23
e.568 c.53.32
5. Number of male in IT department is what percent d.41.23
of the number of female in marketing department? e.56.23

Direction (6-10) Study the charts and answer the question.


Number of candidates appearing three different exams from four different cities given in bar graph and percentage of
student passed the exams given in line graph.
900
780
800 720 700
700 600
560 580
600
470 450
500
400 340
300 250
204 200
200
100
0
P Q R S

IBPS CGL RAILWAYS

Page 177 of 199

Subscribe The Xpress Video Course & Mock Test Package for Bank & Insurance Exams
If there are any suggestions/ errors in our PDFs Feel Free to contact us via this email: admin@exampundit.in
Ultra Practice Bundle PDF
IBPS PO Prelims - Quantitative Aptitude
80

70

60

50
IBPS
40
CGL

30 RAILWAYS

20

10

0
P Q R S

6. Number of candidates who passed IBPS from state d.75:34


P was approximately what percent of number of e.23:97
candidates who passed Railways from S? 8. What is the total number of candidates passed
a.232 from Q in all the exams?
b.231 a.234
c.261 b.768
d.245 c.916
e.240 d.456
7. What is the respective ratio between the number of e.780
candidates who appeared for CGL from R and the 9. What is the difference between the total number
number of candidates who passed in Railways from candidates passed CGL from Q and R together and
R? the total number candidates passed Railways from P
a.23:79 and S together?
b.53:89 a.456
c.55:57 b.434
Page 178 of 199

Subscribe The Xpress Video Course & Mock Test Package for Bank & Insurance Exams
If there are any suggestions/ errors in our PDFs Feel Free to contact us via this email: admin@exampundit.in
Ultra Practice Bundle PDF
IBPS PO Prelims - Quantitative Aptitude
c.678 a.596
d.567 b.567
e.345 c.467
10. What is number of candidates passed Railways d.460
exams from all the state together? e.356

Direction (11-15) Study the graphs and answer the question.


Percentage distribution of runs score by M S Dhoni in three formats T20, Test, and ODI is given in Pie chart. Total
runs (T20+Test+ODI) score by M S Dhoni is 18000.
In table ratio between runs score in India and the outside of India is given.

Percentage of Runs
T20 Test ODI

15%

25%
60%

runs score in India : runs score outside of India


T20 4:5
Test 8:7
ODI 7:5
11. What is respective ratio between the runs score in a.3:2
India in T20 format and the runs score in outside b.4:5
of India in Test format? c.1:2

Page 179 of 199

Subscribe The Xpress Video Course & Mock Test Package for Bank & Insurance Exams
If there are any suggestions/ errors in our PDFs Feel Free to contact us via this email: admin@exampundit.in
Ultra Practice Bundle PDF
IBPS PO Prelims - Quantitative Aptitude
d.9:2 14. Runs scores in Test formats is what percent more
e.4:7 than the runs score in T20 formats?
12. Runs score in India in ODI format is what percent a.55%
of runs score in India in T20 format? b.66.66%
a.345% c.33.33%
b.123% d.70%
c.567% e.88.88%
d.525% 15. What is difference between the runs score in ODI
e.569% in outside of India and the runs score in test in
13. What is average runs score outside of India in all outside of India?
three formats? a.2400
a.2100 b.2350
b.2400 c.1400
c.2700 d.4300
d.2000 e.2100
e.2345

Direction (16-20) Study the graphs and answer the question.


The following line graph shows the total number of Members in five different parties. And the Table shows the
percentage of the females out of the total members in each Party.

Page 180 of 199

Subscribe The Xpress Video Course & Mock Test Package for Bank & Insurance Exams
If there are any suggestions/ errors in our PDFs Feel Free to contact us via this email: admin@exampundit.in
Ultra Practice Bundle PDF
IBPS PO Prelims - Quantitative Aptitude

Members
4000
3400
3500
3200
3000
2500 2800
2500

2000 2100
Members
1500

1000

500

0
A B C D E

Party Percentage of
female
A 40%
B 71.42%
C 62.5%
D 60%
E 42.85%

Note: Take approximate value which is derived from the Table


16. Find the ratio of the total number of members of c.17:19
A and B together to the total number of female d.17:29
members of C and E together? e.23:16
a.21:23
b.23:29

Page 181 of 199

Subscribe The Xpress Video Course & Mock Test Package for Bank & Insurance Exams
If there are any suggestions/ errors in our PDFs Feel Free to contact us via this email: admin@exampundit.in
Ultra Practice Bundle PDF
IBPS PO Prelims - Quantitative Aptitude
17. Find the difference between the number of male 18. Total number of male members of A and E is
members of A and the number of the female members approximately what percent of the total number of
of D? female members of B and D?
a.132 a.66.66%. b.71.23%. c.87.57%. d.78.89%. e.71.34%
b.234 19. What is average of the number of male members
c.256 of A, C, and E?
d.140 a.1234.66. b.1433.33. c.1221.11. d.1123.33. e.1890.33
e.123 20. The numbers of male members of C is what
percent more or less than the male numbers of B?
a.100%. b.34.56%. c.36.87%. d.78.56%. e.60%

Direction (21-25) Study the graphs and answer the question.


Bar-graph shows the number of people in five different cities and table shows the percentage of male in five cities
and the ratio of literate and illiterate people in five different cities.

Number of people
3500
3000
3000
2500 2400
2500
2000
2000

1500

1000

500

0
P Q R S

Number of people

Page 182 of 199

Subscribe The Xpress Video Course & Mock Test Package for Bank & Insurance Exams
If there are any suggestions/ errors in our PDFs Feel Free to contact us via this email: admin@exampundit.in
Ultra Practice Bundle PDF
IBPS PO Prelims - Quantitative Aptitude
City Percentage of males Literate : Illiterate
P 60% 2:3
Q 50% 3:1
R 40% 5:1
S 30% 2:1

21. Literate people from city P is what percent more a.1692. b.1420. c.1650. d.1120. e.1320
than the illiterate people from city Q? 24. What is the ratio of total females from city S and
a.70%. b.45%. c.60%. d.50%. e.100% T to the total illiterate males from city P and city R?
22. Find the ratio of between the numbers of female a.2:3. b. Cannot be determine. c.4:5. d.7:2. e.4:9
from city R to the number of male from city S? 25. Find the difference between number of female
a.1:2. b.2:3. c.5:9. d.5:2. e.2:1 from city Q and number of male from city R?
23. What is the average of literate people in five a.200. b.450. c.310. d.250. e.220
cities?

Direction Q (26-30) – Study the graphs and answer the flowing question.
Four containers has different amount (in liter) mixture (milk +water).
Bar graphs shows the total amount of mixture and table graph show the ratio of milk and water in respective
container.

Total Quantity(in liter)


80
60
60
40
40 30 25
Total Quantity(in liter)
20

0
P Q R S

Page 183 of 199

Subscribe The Xpress Video Course & Mock Test Package for Bank & Insurance Exams
If there are any suggestions/ errors in our PDFs Feel Free to contact us via this email: admin@exampundit.in
Ultra Practice Bundle PDF
IBPS PO Prelims - Quantitative Aptitude
Container Ratio of milk and water
P 2:1
Q 4:1
R 5:3
S 3:2
26. In container P, how much water should be added c.23:13
to it so that the ratio of milk and water becomes 2:3? d.17:11
a.20 e.13:29
b.23 29. What is the difference of amount of Milk in
c.29 container S and container R?
d.21 a.9
e.19 b.11
27. In container Q, 5 liter water is added to the c.19
mixture. What is the new ratio of water and milk? d.18
a.5:6 e.13
b.3:5 30. 50% of mixture S taken out and 2 liter water
c.2:1 added to the mixture. What is the new ratio of milk
d.1:2 and water?
e.3:2 a.9:7
28. 60% of mixture R and 40% of mixture P is mixed b.7:3
together in containers T. what is the ratio of milk and c.3:5
water in container T? d.3:2
a.7:11 e.7:9
b.5:4
Direction Q (31-35) Study the charts and answer the questions.
Length (in meter) of five trains given in bar graph and line graph shows the speed (m/sec) of those trains.

Page 184 of 199

Subscribe The Xpress Video Course & Mock Test Package for Bank & Insurance Exams
If there are any suggestions/ errors in our PDFs Feel Free to contact us via this email: admin@exampundit.in
Ultra Practice Bundle PDF
IBPS PO Prelims - Quantitative Aptitude

Length of the train(in meter)


300

250

200

150

100

50

0
V W X Y Z

Length of the train(in meter)

Speed of the trains(m/sec)


50
40
40
30
30 25
20
20 15

10
0
V W X Y Z

Speed of the trains(m/sec)

31. Train V and train W is running towards each c.14


other. In how many sec they cross each other? d. 6 3/4
a.12 e.8 2/3
b.16
Page 185 of 199

Subscribe The Xpress Video Course & Mock Test Package for Bank & Insurance Exams
If there are any suggestions/ errors in our PDFs Feel Free to contact us via this email: admin@exampundit.in
Ultra Practice Bundle PDF
IBPS PO Prelims - Quantitative Aptitude
32. Train Z cross a platform in 30 sec. How much e.32
time train Y needs to cross the same platform? 34. Train Y and Train Z running in same direction.
a.6 In how many sec they pass one another?
b.9 a.32.3
c.8.44 b.29.33
d.12.33 c.19
e.3 d.25
33. Train X cross bridge A in 16 sec and Train W e.21.3
cross bridge B in 20 sec. then find the difference of 35. What is average length of train V, X and Z?
length of bridge A and bridge B? a.233.3
a.30 b.256.12
b.45 c.245
c.20 d.250
d.29 e.213.3

Direction Q (36-40) – Study the graphs and answer the flowing question.
Total monthly income of five persons is given in bar graphs and in table ratio of expenditure in food, rent, and bill is
given.

Income
Q

O
Income
N

0 5000 10000 15000 20000 25000

Page 186 of 199

Subscribe The Xpress Video Course & Mock Test Package for Bank & Insurance Exams
If there are any suggestions/ errors in our PDFs Feel Free to contact us via this email: admin@exampundit.in
Ultra Practice Bundle PDF
IBPS PO Prelims - Quantitative Aptitude

Person Food : Rent : Bill


M 2:3:1
N 3:4:2
O 5:4:1
P 3:2:3
Q 5:4:2
36. Find the ratio between the amounts spends in rent b.2400
by M and the amounts spend in rent by N? c.2000
a.1:1 d.2100
b.3:2 e.2222
c.2:1 39. Amount spend by P in rent is what percent of the
d.5:4 amount spend by M in bill?
e.3:4 a.120%
37. What is the average amount spends in bill by O, P b.200%
and Q together? c.130%
a.2346 d.122%
b.1234 e.145%
c.3400 40. Amount spend by Q in food is what percent more
d.4000 or less than the amount spend by P in rent?
e.2340 a.150%
38. What is the difference between the total amounts b.120%
spend by N in food and bill and the total amounts c.180%
spend by P in food and bill? d.210%
a.1800 e.235%

Direction Q (41-45) Study the graphs and answer the following question.

Page 187 of 199

Subscribe The Xpress Video Course & Mock Test Package for Bank & Insurance Exams
If there are any suggestions/ errors in our PDFs Feel Free to contact us via this email: admin@exampundit.in
Ultra Practice Bundle PDF
IBPS PO Prelims - Quantitative Aptitude
In table total number of student in five classes and ratio of number of boys and girls is given and in line graphs
average age boys and girls in respective class is given.
Class Total student Boys : Girls
I 60 7:5
J 45 4:5
K 35 5:2
L 55 6:5

Chart Title
25

20

15

10

0
I J K L

Average age boys Average age of girls

41. What is the average age of total students of class 42. What is the difference between total age of all the
I? boys in class K and the total age of girls in class J?
a.14.36 a.35
b.21.32 b.29
c.12.32 c.12
d.23.3 d.25
e.19.75 e. 21
Page 188 of 199

Subscribe The Xpress Video Course & Mock Test Package for Bank & Insurance Exams
If there are any suggestions/ errors in our PDFs Feel Free to contact us via this email: admin@exampundit.in
Ultra Practice Bundle PDF
IBPS PO Prelims - Quantitative Aptitude
43. Number of boys in class L is what percent of c.212:231
number of boys in class I? d.239:213
a.44.22% e.235:234
b.74.32% 45. What is the total no of girls in all the class
c.85.71% together?
d.66.66% a.85
e.33.33% b.65
44. What is the ratio of total age of the class J and c.78
total age of the class L? d.67
a.290:233 e. 56
b.187:214

Direction Q (46-50) - Study the pie charts and answer the flowing question.
Distributions of total number of voters from five different cities and distribution of female voters from total female
among these cities is given.
Total number of voters =24000
Ratio of males and females =7:5

Percentage voters
P Q R S T

10%
25%
30%
15%
20%

Page 189 of 199

Subscribe The Xpress Video Course & Mock Test Package for Bank & Insurance Exams
If there are any suggestions/ errors in our PDFs Feel Free to contact us via this email: admin@exampundit.in
Ultra Practice Bundle PDF
IBPS PO Prelims - Quantitative Aptitude

Percentage of female
voters

15%
30%
20%
10% 25%

P Q R S T

46. The number of male voter in city P is what c.38:15


percent of total number of voter in the same city? d.65:17
a.32% e.32:31
b.40% 49. What is the difference of total number female
c.55% voters of city Q and T?
d.60% a.2300
e.50% b.1000
47. What is average number of female voters in city c.2100
R, S, T? d.1600
a.2400 e.1200
b.2100 50. The number of male voter of Q is what percent of
c.1800 male voter of S?
d.1500 a.21.15%
e.1200 b.25%
48. What is ratio of male voter of city R to the female c.33.33%
voter of city T? d.14.28%
a.5:39 e.16.67%
b.3:4

Page 190 of 199

Subscribe The Xpress Video Course & Mock Test Package for Bank & Insurance Exams
If there are any suggestions/ errors in our PDFs Feel Free to contact us via this email: admin@exampundit.in
Ultra Practice Bundle PDF
IBPS PO Prelims - Quantitative Aptitude
Mixed DI - Answer and Detailed Explanation
Solution (1-5):-
Common Explanation-
Total number of employee in the company is 3600.
30% of total employee is on accounts department.
So, total employee in accounts department is 3600*30/100 =1080.
Out of the total employee 40% is female in accounts departments.
So female employee in accounts is 1080*40/100=432
Now male employee in accounts department is 1080-432 =648.
Like this way we can calculate number of male, female employee of other department also which is given in the
table.
Department Total employee Number of female Number of male
Accounts 3600*30/100 1080*40/100 1080-432
=1080 =432 =648
HR 3600*25/100 900*60/100 900-540
=900 =540 =360
IT 3600*10/100 360*55/100 360-198
=360 =198 =162
Marketing 3600*35/100 1260*30/100 1260-378
=1260 =378 =882

1. Answer : E = (3600*10)/100 =360


Total female in HR and IT department is Number of female is 360*55/100 =198
540+198 =738 So, number of male is 360-198 =162
2. Answer : D 3. Answer : C
Total number of employee in IT department is Respective ratio

Page 191 of 199

Subscribe The Xpress Video Course & Mock Test Package for Bank & Insurance Exams
If there are any suggestions/ errors in our PDFs Feel Free to contact us via this email: admin@exampundit.in
Ultra Practice Bundle PDF
IBPS PO Prelims - Quantitative Aptitude
= (number of male in HR department: number of Total female in accounts, HR, IT is =
female in marketing department) (432+540+198) =1170
=360:378 [value from the table] So the respective difference is (2052-1170) =882
=180:189=20:21 5. Answer: A
4. Answer : B So the required percentage is =
Total male in all the department is = (162/378)*100=42.85%
(648+360+162+882) =2052
Solution (6-10)
Common explanation
IBPS CGL Railways

Number Number of Number of


Appeared student Appeared student Appeared student
passed passed passed

560*60/100 470*60/100 204*50/100


P 560 470 204
336 282 102

720*50/100 580*70/100 250*60/100


Q 720 580 250
360 406 150

780*70/100 450*60/100 340*60/100


R 780 450 340
546 270 204

700*40/100 600*75/100 200*70/100


S 700 600 200
280 450 140

6. Answer: E Number of candidates appeared from R in CGL is 450


So the required percentage is (336/140)*100 =240% Number of candidates passed from R in Railways is 204
7. Answer: D So required ratio is = 450:204=75:34

Page 192 of 199

Subscribe The Xpress Video Course & Mock Test Package for Bank & Insurance Exams
If there are any suggestions/ errors in our PDFs Feel Free to contact us via this email: admin@exampundit.in
Ultra Practice Bundle PDF
IBPS PO Prelims - Quantitative Aptitude
8. Answer: C Total number of candidates passed from P and S in SBI
Total Number of candidates passed from Q is is =102+140 =242
=360+406+150 =916 So the difference is = 676-242 = 434
9. Answer: B 10. Answer: A
Total number of candidates passed from Q and R in CGL Total number of candidates passed Railways is =
is = 406+270 =676 102+150+204+140=596
Solution (11-15)
Common explanation
Runs score by M S Dhoni in T20 format is =18000*15/100=2700
In T20 format out of the total runs score in India is =2700*4/9=1200
Runs score in outside of India in T20 format is =2700*5/9=1500
Others formats runs also calculate in same way are given in the table.
Format Total runs Runs in India Runs in outside of India
T20 18000*15/100 2700*4/9 2700*5/9
=2700 =1200 =1500
ODI 18000*60/100 10800*7/12 10800*5/12
=10800 =6300 =4500
TEST 18000*25/100 4500*8/15 4500*7/15
=4500 =2400 =2100

11. Answer: E Runs score in India in ODI format is 6300


So the respective ratio is Runs score in India inT20 format is 1200
= (runs score in India in T20 format: runs score in So the required percentage is (6300/1200) *100 = 525%
outside of India in Test format) 13. Answer: C
=1200:2100 [value taken from the table] Total runs score in outside of India is
=4:7 =(1500+4500+2100) =8100
12. Answer: D

Page 193 of 199

Subscribe The Xpress Video Course & Mock Test Package for Bank & Insurance Exams
If there are any suggestions/ errors in our PDFs Feel Free to contact us via this email: admin@exampundit.in
Ultra Practice Bundle PDF
IBPS PO Prelims - Quantitative Aptitude
So the average runs score in outside of India is So ODI (4500-2700) =1800 score than Test format.
=8100/3=2700 So the required percentage =(1800/2700)*100 =66.66%
14. Answer b 15. Answer a
Runs score in outside of India in Test is 2700 So the respective difference is 4500-2100 =2400
Runs score in outside of India in ODI is 4500
Solution (16-20)
Common explanation
Total members in party A is 2500
Number of female members in party A is (2500*40/100) =1000
Number of male members in party A is (2500-1000) =1500
Like same way we can calculate the other parties also which is given in table.

Party Total members Female members Male members


A 2500 2500*40/100 2500-1000
=1000 =1500
B 2100 2100*71.42/100 2100-1500
=1500 =600
C 3200 3200*62.5/100 3200-2000
=2000 =1200
D 3400 3400*60/100 3400-2040
=2040 =1360
E 2800 2800*42.85/100 2800-1200
=1200 =1600

16. Answer: E Total number of female members in C and E is


Total number of members in A and B is 2500+2100 =2000+1200=3200
=4600 So the respective ratio is = 4600:3200 =23:16

Page 194 of 199

Subscribe The Xpress Video Course & Mock Test Package for Bank & Insurance Exams
If there are any suggestions/ errors in our PDFs Feel Free to contact us via this email: admin@exampundit.in
Ultra Practice Bundle PDF
IBPS PO Prelims - Quantitative Aptitude
17. Answer: D So required percentage is = {(3100/3540)*100} =87.57%
Male members in A is 1500 19. Answer: B
Male members in D is 1360 Total male members in A, C, E party is
So the respective difference is (1500-1360) =140 1500+1200+1600 =4300
18. Answer: C So the average male members in this three parties is
Total male members in A and E party is 1500+1600 4300/3 =1433.33
=3100 20. Answer: A
Total female members in B and D party is 1500+2040 Number of male members in C is 1200 and Number of
=3540 male members in C is 600.
So in party C numbers of male members is 1200-600 So required percentage is = (600/600)*100=100%
=600 more than number of male members in B party.
Solution (21-25) So the total litterte people in all the city together is =
21. Answer: E (1000+2500+1500+1600) =6600
Number of literate people from P is =2500*2/5=1000 So the average litterte people is =6600/4=1650
Number of illiterate people from Q is =2000*1/4=500 24. Answer: B
So required percentage is = {(1000- We cannot know the data about city T.
500)/500}*100=100% 25. Answer: A
22. Answer: D Female from Q is = 2000*50/100 =1000
Female from R is 3000*60/100 =1800 Male from R is = 3000*40/100 = 1200
Male from S is 2400*30/100=720 So the respective difference is (1200-1000) = 200
So respective ratio is 1800:720 =5:2 Solution – (26-30)
23. Answer: C 26. Answer: A
Litterte people from P =2500*2/5=1000 In container P total quantity of mixture is 30 liter.
Litterte people from Q =3000*5/6=2500 Out of 30L amount of milk is 30*2/3=20L and water is
Litterte people from R=2000*3/4=1500 30*1/3= 10L
Litterte people from S=2400*2/3=1600 Let, X liter water should added to the water to make the
ratio.

Page 195 of 199

Subscribe The Xpress Video Course & Mock Test Package for Bank & Insurance Exams
If there are any suggestions/ errors in our PDFs Feel Free to contact us via this email: admin@exampundit.in
Ultra Practice Bundle PDF
IBPS PO Prelims - Quantitative Aptitude
Or, Now quantity of mixture left in container is 60*50/100
=30L
Or, 2x+20=60
In 30L, milk is 30*3/5 =18L and water is 30*2/5=12L
Or, x = (60-20)/2=20 liter
2L water is added in container, so amount of water is
27. Answer: D
12+2=14L
In container Q total quantity of mixture is 25L.
New milk and water ratio is 18:14 = 9:7
Milk is 25*4/5=20 and water is 25*1/5=5
Solution (31-35)
Now 5L water is added. So the quantity of water is now
31. Answer: E
5+5=10
When two train running opposite direction and cross
Now ratio of water to milk in mixture is 10:20 =1:2
each other, they have to cross sum their length.
28. Answer: C
Sum of length of train V and W is = (280+240) =520 m
60% mixture from R is 60*40/100 =24 L.
And relative speed is sum of their speed i.e. (40+20)
In 24L of mixture amount of milk is (24*5/8) =15L and
=60m/sec
amount of water is (24*3/8) =9L
So the time taken to cross each other is 520/60=8 2/3sec
40% mixture from P is 30*40/100 =12L, out of that
32. Answer: D
amount of milk is 12*2/3 =8L and amount of water is
Train cross a platform in 30 sec.
12*1/3 =4L.
Let, the length of the platform is L.
So, In container T total milk is (15+8) = 23L
So, {(260+L)/15} = 30
In container T total water is (9+4) = 13L
Or, L=450-260=190m
So in container T, milk and water ratio is 23:13
This 190 m long platform train Y cross in
29. Answer: B
{(190+180)/30} =37/3 =12.33 sec
In container S, total quantity of mixture is 60.out of that
33. Answer: C
amount milk is
Let the length of bridge A is p and length of bridge B is
= (60*3/5) =36L
q.
In container R, amount milk is 40*5/8 =25
Bridge A cross by train X in 16sec
So the required difference is (36-25) =11
So, (220+p)/25=16, p=400-220=180
30. Answer: A
Bridge B cross by train W in 20sec
50% of mixture taken out from container S.
Page 196 of 199

Subscribe The Xpress Video Course & Mock Test Package for Bank & Insurance Exams
If there are any suggestions/ errors in our PDFs Feel Free to contact us via this email: admin@exampundit.in
Ultra Practice Bundle PDF
IBPS PO Prelims - Quantitative Aptitude
So, (240+q)/20 =20, q=400-240=160 So time taken to cross one another is 440/15 =29.33sec
(p – q) =180-160 =20 35. Answer: A
34. Answer: B Sum of total length of train V, X, and Z is 280+220+200
Train Y and Z running same direction so the relative =700
speed is 30-15=15m/sec So average length is 700/3 =233.3
Total distance they have to cover is 180+260=440
Solution (36-40)
Common explanation
Person Total income Spend in food Spend in rent Spend in bill
M 12000 12000*2/6 12000*3/6 12000*1/6
=4000 =6000 =2000
N 18000 18000*3/9 18000*4/9 18000*2/9
=6000 =8000 =4000
O 20000 20000*5/10 20000*4/10 20000*1/10
=10000 =8000 =2000
P 16000 16000*3/8 16000*2/8 16000*3/8
=6000 =4000 =6000
Q 22000 22000*5/11 22000*4/11 22000*2/11
=10000 =8000 =4000
36. Answer E Total amount spend by N in food and bill is 6000+4000
So the required ratio is 6000:8000 =3:4 =10000
37. Answer D Total amount spend by p in food and bill is
Total amount spend in bill by O, P and Q is 6000+6000=12000
=2000+6000+4000=12000 So the difference is 12000-10000=2000
So the average amount spend in bill is 12000/3 =4000 39. Answer B
38. Answer C So required percent is= {(4000/2000)*100} =200%
40. Answer A

Page 197 of 199

Subscribe The Xpress Video Course & Mock Test Package for Bank & Insurance Exams
If there are any suggestions/ errors in our PDFs Feel Free to contact us via this email: admin@exampundit.in
Ultra Practice Bundle PDF
IBPS PO Prelims - Quantitative Aptitude
Q spends 10000 in food and P spends 4000 in rent. No of boys in class L is 30 and girls in class L is
So Q spend (10000-4000) = 6000 more than P spends in 55*5/11=25
rent. So the total age of the student of class L is
So the required percent is (6000/4000)*100 =150% = (30*19) + (20*25) = 1070
Solution (41-45) So required ratio is =935:1070=187:214
41. Answer E 45. Answer A
No of boys in class I is=60*7/12 =35 Girls in class I =60*5/12 =25
Total age of boys is = 35*21=735 Girls in class J =45*5/9 =25
No of girls in class I is =60*5/12 =25 Girls in class K =35*2/7 =10
Total age of girls is =18*25=450 Girls in class L =55*5/11 =25
Total age of 60 student in class is =735+450=1185 So total no of girls in all the class is =25+25+10+25 =85
So the average age of the student class is 1185/60=19.75 Solution (46-50)
42. Answer D Total female 24000*5/12 =10000
No of boys in class K is 35*5/7 =25 Total male is 24000*7/12=14000
So the total age of boys in class K is =18*25 = 450 46. Answer E
No of girls in class J is 45*5/9=25 Total number of voter in city P is 24000*25/100 =6000
So the total age of girls in class J is =25*19 = 475 Number of female in city P is 10000*30/100=3000
So the required difference is (475-450) = 25 Number of male is 6000-3000
43. Answer C So required percentage is (3000/6000)*100 =50%
No of boys in class L is 55*6/11=30 47. Answer D
No of boys in class I is 60*7/12 =35 Total number of female voter in city R,S,T is
So required percent = (30/35)*100 = 85.71% (10%+20%+15%) =45%
44. Answer B So the average female voter is 45%/3=15%
No of boys in class J is 45*4/5 20 and no girls in class J So average female voter these three city is
is 25 10000*15/100=1500
So total age of the student of class J is 48. Answer C
= (20*23) + (19*25) = 935 Total voters in city R is 24000*20/100 =4800

Page 198 of 199

Subscribe The Xpress Video Course & Mock Test Package for Bank & Insurance Exams
If there are any suggestions/ errors in our PDFs Feel Free to contact us via this email: admin@exampundit.in
Ultra Practice Bundle PDF
IBPS PO Prelims - Quantitative Aptitude
Total female voters in city R is 10000*10/100 =1000 50. Answer A
Total male voters in city R is 4800-1000=3800 Total number of voter in Q is =24000*15/100 =3600
Total female voters in city T is 10000*15/100 =1500 Total number of female voter in city Q is
So required ratio is 3800:1500 =38:15 =10000*25/100 =2500
49. Answer B Total number of male voter in city Q is =3600-2500
In city Q female voter is 25% =1100
In city T female voter is 15% Total number of voter in city S is =24000*30/100 =7200
So the difference of female in city P and T is (25%-15%) Total number of female voter in city S is
= 10% =10000*20/100 =2000
So the difference in number of female voter these two Total number of male voter in city S is 7200-2000 =5200
cities is So required percentage is =(1100/5200)*100 =21.15%
=10000*10/100 =1000

Data Interpretation Practice Questions PDF Download


Get More Quantitative Aptitude Practice Questions PDF

Click Here For More IBPS PO Prelims Bundle PDF

THE COMPLETE Static GK Capsule for Upcoming Exams

The COMPLETE Static Banking Awareness PDF

Click Here to Join Our What’s App Group & Get Instant Notification on

Study Materials & PDFs

Click Here to Join Our Official Telegram Channel


Page 199 of 199

Subscribe The Xpress Video Course & Mock Test Package for Bank & Insurance Exams
If there are any suggestions/ errors in our PDFs Feel Free to contact us via this email: admin@exampundit.in

You might also like